X



トップページ物理
1002コメント428KB
特殊相対性理論でわからないことあったら、俺ができる限り教えるスレ
レス数が1000を超えています。これ以上書き込みはできません。
0001ご冗談でしょう?名無しさん
垢版 |
2022/06/19(日) 02:18:05.28ID:YoyAPKCg
特殊相対性理論をだいたいは理解できたので質問があったら聞いてください
0004ご冗談でしょう?名無しさん
垢版 |
2022/06/19(日) 02:40:45.93ID:???
双子のパラドックスって要は、慣性系では時間の遅れはお互い様ってことだけど、加速系になると加速したほうの時間の流れが相対的にゆっくりになるから双子のパラドックスは解消されるよねって話だった気がする
0007ご冗談でしょう?名無しさん
垢版 |
2022/06/19(日) 07:38:19.64ID:ufbZM2bu
相対性理論は間違っていますが何か?
0009poem
垢版 |
2022/06/19(日) 10:19:10.55ID:/VEIMdzI
このスレ建てようと思った考えあってこのクソスレ(未)建てたんじゃない?

>>1
一昨日かなスレ「光速度垂直成分問題(光速不変垂直適用)【思考実験】」自分が建てた直後にこのように建てたから、たぶんこのスレの核のこれ質問して欲しいんじゃない?

質問・光速度に水平の300Mm/sは光速度不変の法則が適用されて光速度不変になるけど、光速度に垂直の0m/sに光速度不変の法則が適用されて零速度不変になるのか

ついでに
質問・なんで光は水平の300Mm/s光速度、垂直の0m/s零速度、物体の速度に影響されない不変なのか。光も物体も同じ速度なのにお互いに影響されず不変になる物理的原理は。光の不変速度の正体は?

ついでに
質問・スレ「光速度垂直成分問題(光速不変垂直適用)【思考実験】」で出てきた3つほどの思考実験について結論の否定や設定の問題などレビューを

ついでに
質問・相対性理論が0m/s零速度にも適用できて光の全て(360°)に適用できるとなると相対性理論の修正点はあるか


自分はここでは一般人の立場で参加する
1はここら辺質問して欲しいんじゃない?
上はなるべく一般人を逸脱してない質問した
0011ご冗談でしょう?名無しさん
垢版 |
2022/06/19(日) 10:46:10.73ID:???
一般相対性理論の内容は難解でまだ理解できてないのですが、特殊相対性理論に関しては、光速度不変の原理と相対性原理という2つをおさえていれば、誰でも簡単に理解可能だと思います。
0017ご冗談でしょう?名無しさん
垢版 |
2022/06/19(日) 11:02:47.84ID:???
もしかしてE=mc^2に運動量も入れろってことかな
万有引力の法則の相対論バージョンを作ろうとして、質量とエネルギー運動量の等価性を用いてエネルギー運動量テンソルを使うわけだし
0018ご冗談でしょう?名無しさん
垢版 |
2022/06/19(日) 11:05:06.65ID:???
私は特殊相対論に関しては大学数学を使って厳密に理解しているわけではないですが、特殊相対性理論の思考実験(同時性の不一致、空間の縮み、時間の遅れなど)については、ある程度理解できたと思ったのでこのようなスレを書かせていただきました。
0020ご冗談でしょう?名無しさん
垢版 |
2022/06/19(日) 11:11:44.07ID:???
>>18
そんなものを解説してるサイトなんて山ほどあるし、
物理板なんて聞いてもいないのにわけわからんこと教えてくる人ばかりだけど、わざわざキミ個人に聞くメリットってなんかあんの?
0023poem
垢版 |
2022/06/19(日) 11:25:51.95ID:???
>>12
pdf見た
こんな問題解ける人が日本の理系上位大学の全員なら物理学が今の時点でも数十年以上進んでて物理学の進歩ペース高速で、今みたいに頭打ちじゃない気がする。
今専門の立場にいる人には頭打ちじゃないかもしれないけど知らない一般人素人からすると新しい発見・教科書の更新が少ないから頭打ちに見えるんだけど、
こんな問題解ける頭のいい理系上位大学とさらに化物のトップ理系大学の頭の良さの人がゴロゴロいて頭打ち…少なくとも一般人への物理学の進歩のお溢れが頭打ちなの、なんでなのか不可思議で疑問。まるで扱ってる現代物理学が間違っているかのよう。間違ってないなら不可思議。

とかキチガイレスしてみる
0025poem
垢版 |
2022/06/19(日) 11:31:40.36ID:???
>>23書く前レス止まってたから進まないと思ってたけどレスが進んでタイミング悪すぎ
0027poem
垢版 |
2022/06/19(日) 11:56:43.07ID:???
スレ主はどんな質問想定してたの?
0028ご冗談でしょう?名無しさん
垢版 |
2022/06/19(日) 13:08:20.32ID:???
じゃあむかし話題になった車輪の問題

鉄道の車輪は回転数が増すほど速度が増すので、速度が光速に近づけば近づくほど回転数は増していくはず
でもそれを地上から見た場合、列車が光速に近づけば近づくほど時間の流れが遅くなるから車輪の回転もゆっくりになるはず
相対論的に考えて、地上から見た車輪はどうなっているのでしょうか?
0031ご冗談でしょう?名無しさん
垢版 |
2022/06/19(日) 13:24:37.66ID:???
>>27
キミはたぶん「光速不変」の意味を間違えてるだけだ。

質点の「速度」の意味はベクトルだからxyz3成分があるが、「速さ」はスカラーで1つ。
「光速不変の原理」の「光速」の意味は光の「速さ」だから一つの定数c
光の進行方向が「不変」の意味でもないから間違えないように。
0032ご冗談でしょう?名無しさん
垢版 |
2022/06/19(日) 13:32:40.23ID:???
>>28
>>29
>特殊相対論ってざっくり言えば等速直線運動

基本的には正しい、特殊相対論では車輪の形状は簡単だが回転数は計算が難しい。
0033ご冗談でしょう?名無しさん
垢版 |
2022/06/19(日) 13:45:53.88ID:???
数式だけで考える「数学思考」の人は理工系で使いものにならない。

常に物理解釈が出来る「物理思考」の人は将来有望な人材。(copyright)
0035ご冗談でしょう?名無しさん
垢版 |
2022/06/19(日) 13:49:17.61ID:???
>>29
慣性系Aのx,y,z,t座標において螺旋運動を描く質点の世界線を、慣性系Bにローレンツ変換する
それだけのことなんだけど、扱えないと?
0036ご冗談でしょう?名無しさん
垢版 |
2022/06/19(日) 14:20:52.71ID:???
>>32
車輪の回転数は簡単だろ。
列車の速度がv、車輪の半径がrの場合、車輪の回転数は、
 非相対論なら v/2πr
 特殊相対論なら v/2πr / γ
0043ご冗談でしょう?名無しさん
垢版 |
2022/06/19(日) 15:11:52.20ID:???
>>42
列車の慣性系 c>>速さで車輪の円周が回転してれば 円周が2πrと見なせるが

光速に近い速度で車輪の円周がどうなるか、ローレンツ変換で物理解釈できるか?

できないなら単なるコピペ。
0044ご冗談でしょう?名無しさん
垢版 |
2022/06/19(日) 15:23:32.80ID:???
>>43
車輪の回転数求めるのに車輪の円周なんか計算する必要ねーよ。
あと「ローレンツ変換で物理解釈」ってなんやねん?
車輪とレールの接点がどうなるかだけ考えてみ。
0045ご冗談でしょう?名無しさん
垢版 |
2022/06/19(日) 15:33:58.60ID:???
>>44
>車輪とレールの接点がどうなるかだけ考え

そういう奴を単純思考という

特殊相対論のローレンツ変換は「車輪とレールの接点」だけじゃダメなんだよ!
車輪全体で矛盾が起きたら無効だ
0046ご冗談でしょう?名無しさん
垢版 |
2022/06/19(日) 15:37:37.14ID:???
>>45
矛盾が起きるってんなら矛盾を示してみ。
おまえが別のどんな複雑な方法で計算してみようが構わんが、結局答えは v/2πr / γ になるはずだから。
0050ご冗談でしょう?名無しさん
垢版 |
2022/06/19(日) 15:57:30.35ID:???
>>48
「車輪とレールの接点がどうなるかだけ考えてみ。」で、まだわからない程度の奴は使い物にならない。
懇切丁寧にパワポで図解資料とか作らんと分からんか?
0051ご冗談でしょう?名無しさん
垢版 |
2022/06/19(日) 16:06:56.62ID:???
>>49
>∫0 2π √ (r sinθ)^2 +(r/γ cosθ)^2 dθ

(1)その式で線路と列車の速度vで円周の長さに直してくれ、簡単だろ

(2)その式がどういう物理解釈で出てきたのかな?
0053ご冗談でしょう?名無しさん
垢版 |
2022/06/19(日) 16:18:31.35ID:???
>>51
(1) ∫[0, 2π] r√((sinθ)^2 +(√(1- v^2 /c^2) cosθ)^2) dθ
(2) 水平方向が 1/γ に収縮した楕円の円周の計算だ。
特殊相対論だと明確に言ってんのにそれ以外の物理的解釈もなにもあるか。

で、それを聞いてなにをしたいねん?と、こっちが聞いとんねん。
0055ご冗談でしょう?名無しさん
垢版 |
2022/06/19(日) 16:30:07.05ID:???
>>53
>(2) 水平方向が 1/γ に収縮した楕円の円周の計算だ。

そういう意味なら線路系から見た円周だから、列車系からみれば車輪の円周は2πrだな

その場合、線路と接点部の車輪の長さ儉はローレンツ変換でどうなるのか?
光速に近づくとローレンツ収縮するはずだが、円周と矛盾しないのか?
0056ご冗談でしょう?名無しさん
垢版 |
2022/06/19(日) 16:40:00.25ID:???
>>55
線路系から見た車輪回転数の話してんだから、列車系から見たらまた違うの当たり前でしょうが。
列車系から見たら列車が動いてないんだから相対論も関係ない。
もうずっと馬鹿なことしか言ってないよな。
0057ご冗談でしょう?名無しさん
垢版 |
2022/06/19(日) 16:48:24.82ID:???
>>56
最初に戻す誤魔化しで逃げるのか

>>55 のつづき、
列車の慣性系で円周(2πr)の車輪が一定回転して線路との接点部だけローレンツ収縮する??
矛盾してるだろが
0058ご冗談でしょう?名無しさん
垢版 |
2022/06/19(日) 16:52:45.74ID:???
>>57
最初に物すも何もずっとその話題だわ。
じゃなかったらおまえはさっきからずっと何についてはなしてんねん?

列車系から見たらローレンツ収縮するのはレールでしょう。
なにが矛盾やねん?
0059poem
垢版 |
2022/06/19(日) 16:57:27.81ID:???
>>43と他の人

単に車輪の進む方と接地する方と電車自体の空間と時間が伸び縮みするから、車輪の全体の式を表せても部分で見ればその式を変形しないと出ないんじゃない?
0060ご冗談でしょう?名無しさん
垢版 |
2022/06/19(日) 16:59:08.63ID:???
>>58
>列車系から見たらローレンツ収縮するのはレールでしょう。
遂にデタラメを喚きだしたか

レールだけローレンツ収縮してるのなら滑ってるだけで車輪は回転しない。
そんなことも知らんのか
0062poem
垢版 |
2022/06/19(日) 17:04:52.07ID:???
>>31
光速不変が速さについてのみで速度についてでない

=水平成分のみの速度は変わらないが垂直成分の速度は変わる(垂直の向きには不変は掛からずスライドする。垂直の速度には不変が適用されない)

電磁場なら涙型になる(=観測者から見て、場を出す物のある時刻に放たれた最前線ラインの場が同心円にならない)
→この問題って問題じゃなく正常なの?
0064ご冗談でしょう?名無しさん
垢版 |
2022/06/19(日) 17:11:42.95ID:???
>>63
もちろん車輪はローレンツ収縮しないのかと聞かれればもちろんするが、
いびつに縮むことになるので、ある程度のとこまでは張力でがんばる。
そりゃ限界超えたら壊れるだろう。
お前が聞きたかったのはそういう話なの?
0065ご冗談でしょう?名無しさん
垢版 |
2022/06/19(日) 17:15:59.90ID:???
>>63
もちろんローレンツ収縮でいびつに縮んで車輪半径 r が変わるかもしれないが、それなら変形後の r を使って計算しとけ。
0066ご冗談でしょう?名無しさん
垢版 |
2022/06/19(日) 17:27:44.40ID:???
>>64
>いびつに縮むことになるので、ある程度のとこまでは張力でがんばる。
変な方向に話がコロコロ変わる

特殊相対論のローレンツ変換だけからは、回転する車輪が2πrのままで歪むとか

回転数の式も簡単に出て来ないという意味だよ。
0067poem
垢版 |
2022/06/19(日) 17:32:59.98ID:???
>>66
というか、皆車輪が歪むって思ってるけど、車輪の周りの時間空間が歪むんだからね。車輪が歪んだって車輪自体が歪むわけでなく車輪が破壊されたりしない。周りの空間が歪んで見かけ上歪むだけで、何なら平べったくなっても円運動してる。見る観測者によって平べった型が変わる。
0068poem
垢版 |
2022/06/19(日) 17:34:58.16ID:???
>>67
あくまで相対性理論が正しいならの立場のツッコミね。
0069ご冗談でしょう?名無しさん
垢版 |
2022/06/19(日) 17:36:32.61ID:???
>>66
変な方向に話をコロコロ変えてんのはおまえだろ。
車輪が縮んで変形するなんて言い出すなら、はじめからどれくらい縮むか前提として入れとけ。
0071ご冗談でしょう?名無しさん
垢版 |
2022/06/19(日) 17:50:29.25ID:???
>>69
お前かその仲間が、「車輪とレールの接点がどうなるかだけ考えてみ。」
と言い出しただろ

特殊相対論のローレンツ変換が前提なら車輪がローレンツ収縮するのが当然
で矛盾が起こる
そんだけ
0072poem
垢版 |
2022/06/19(日) 17:53:26.46ID:???
>>70
>>67に考えないと、光速近くで必ず物体が粒子に分解されるってなるでしょ。しかも相対的に自分は分解されてなく、互いに違う分岐世界で自分は無傷で相手は分解されて光速度で運動してる互いを見るってなる
0073ご冗談でしょう?名無しさん
垢版 |
2022/06/19(日) 17:57:51.54ID:???
>>71
だから、車輪とレールの接点がどうなるかだけ考えてみ。
そしたら車輪の回転数は v/2πr / γ とわかるだろ。

もちろん車輪の外周はリム部分が支えるのでローレンツ収縮の計算通りには縮まない。
少しは縮むってなら縮んだ後の半径を r として計算しろ。
0074ご冗談でしょう?名無しさん
垢版 |
2022/06/19(日) 18:07:30.85ID:???
>>73
>車輪とレールの接点がどうなるかだけ考えてみ。

お前は自分のやり方が理論と矛盾してないか全く考えないでやるタイプだろ

ならおしまい。
0075ご冗談でしょう?名無しさん
垢版 |
2022/06/19(日) 18:12:16.01ID:???
>>74
特殊相対論の基本通りの話なので矛盾があるはずがないのだが、相対論に矛盾があるって話か?

あるんなら具体的に。
ないんなら疑問は解決したということでおしまい。
0076ご冗談でしょう?名無しさん
垢版 |
2022/06/19(日) 21:27:26.54ID:???
加速する前後2台のロケットの話もあったね
あ、けど等速直線運動についてしか述べられないとか言ってたらそれも無理か
0077ご冗談でしょう?名無しさん
垢版 |
2022/06/20(月) 10:54:35.45ID:???
>>76
>加速する前後2台のロケット・・・等速直線運動についてしか述べられないとか言ってたらそれも無理か

大間違い!

2台のロケットのパラドックスは直線上の運動が基本だから、加速度運動の各瞬間を
一定速度状態と見なし、特殊相対論の大局的なローレンツ変換が可能なのだよ。

双子のパラドックスの計算方法も同様であり、どちらも論理的矛盾が無い。
0078poem
垢版 |
2022/06/20(月) 10:58:00.64ID:???
>>76
加速ぐらいなら疑問に思う人たくさんいるから計算してると思うから、前例あるでしょ?特殊相対性理論の原本の範疇になくても、定式化は済んでるんじゃない?
0079ご冗談でしょう?名無しさん
垢版 |
2022/06/20(月) 11:17:30.34ID:???
スレ主が等速直線運動しか答えられないらしいから、スレ主にそういうのテーマ聞くのは無理だなって話だろ
いちいち偉そうに否定するとこじゃない
0080poem
垢版 |
2022/06/20(月) 11:51:29.06ID:???
ちなみにだけど
加速度の瞬間の速さが光速度を超えないように空間時間が歪むなら

0.000000000000…0000001の時間内に1m/sまで加速して時間内に加速が止まる加速度が300Mm/s^2を余裕で超えるものとかってのは実現できたりする?速度は300Mm/sが上限だけど、加速度の上限だと何m/s^2?
0081ご冗談でしょう?名無しさん
垢版 |
2022/06/20(月) 11:59:22.79ID:???
>>80
キミが経験による低速運動のマインドコントロールが解ければ疑問病も治るだろう

速さが1m/sとか1000m/s の物理計算が正しくても、非常に大きい約300000km/sの光速で
も同じだと考えるのは物理的に正しいとは言えない、物理実験の事実から認めることだ。

光速不変を簡単な例にすれば、慣性系で
・真空中で静止した電球を発光させれば球面状に光速cで広がる。
・真空中で一定速度の電球を発光させれば球面状に光速cで広がる。
(同じなのは速度だけで、波長や強さは方向で異なるから間違わないように)

キミが矛盾してると考えたなら、経験による低速運動計算のマインドコントロールが
解けてないということだ。
運動の計算方法も根本的に変える必要があるということ。
0082poem
垢版 |
2022/06/20(月) 12:05:56.23ID:???
>>81
80は垂直問題の件とは関係ない一般人の質問なんだけど
0084ご冗談でしょう?名無しさん
垢版 |
2022/06/20(月) 12:16:06.63ID:???
>>82
キミは
>>81 などの特殊相対論の基本を理解してから、計算方法も根本的に変えないと

>>80 の、他人が読むと意味不明のスレになってしまう。
0088ご冗談でしょう?名無しさん
垢版 |
2022/06/20(月) 12:24:20.82ID:???
てか登記って絵文字に対しては粘着アスペってまったくいわないよね?

なんだかんだで絵文字と絡むの楽しみにしてるんだろう
こんなところに顔出さないで電磁気スレでじゃれあってきたらいいと思うよ
0089ご冗談でしょう?名無しさん
垢版 |
2022/06/20(月) 12:31:53.95ID:???
>>88
アスペ症はコミュ障害だが計算可能のレベルに分類されている。

絵文字荒らしはアスペ症より酷い自閉症、式などの計算も物理解釈も出来ないレベル
人工無能に近い。
0096poem
垢版 |
2022/06/20(月) 15:18:29.63ID:???
で登記さんの結論は
「垂直0m/s成分には光速不変が適用されない」
「ある時刻に出た360°電磁場はいろんな観測者から見て同心円にならない。いろんな観測者と電磁荷の相対速度差により光速近くなら涙型になる、低速なら僅かに歪んだ同心円になる」
って主張ね

登記さんは色々計算できる人だから、この結論を主張するのも
「垂直成分に不変が適用される計算をすると、理論的に矛盾し反証される実際にやった計算」
「垂直成分に不変が適用されない計算をすると、理論的に突っ込みどころなく証明される実際にやった計算」
計算あってのものだよね。

それを長文で書かずに、>>81小学生に言うようなはぐらかした回りくどい論点も本質も説明していない謎理屈じゃ、証明・反証したことならないから、ちゃんと書いて欲しいな

難しい勉強わからないからpoemさんはそれをわからないけど、
他のわかる人はわかるから、他の人がポツポツ呟いて登記さんが何を計算してたか何を書いていたか溢してくれるだろうから、他の人の翻訳通訳で少しはpoemにもわかると思うから、書く意味はある。書いて欲しい。安心して書いてくれ。書くもの書いてくれなきゃ何も少しもわからない。
0097ご冗談でしょう?名無しさん
垢版 |
2022/06/20(月) 15:53:19.40ID:???
今日のpoemはいつもと比べたら日本語としてなんとなく読めるから言うけど、
登記はそんなこと主張してないで。
誰と間違えてんのか知らんけど登記は計算もしてないし、他の人も登記の翻訳なんかできへんで。
0098ご冗談でしょう?名無しさん
垢版 |
2022/06/20(月) 16:16:18.57ID:???
>>96
既に手遅れだったか、自分の説に凝り固まってしまったようだな。
絵文字爺と同じで
年を取ってからでは脳の回路網が可塑化して、外部からの情報を自分の説に捻じ曲げる
らしい。

Strike while the iron is hot. (鉄は熱いうちに打て)
0099poem
垢版 |
2022/06/20(月) 16:25:25.05ID:???
>>97
助言TNKSなんだけど
>>98
の本人は96の主張みたい

今更だけど98って登記さんでいいんだよね?騙りじゃなく
0101ご冗談でしょう?名無しさん
垢版 |
2022/06/20(月) 16:30:09.45ID:???
誰でも年取れば脳が固まるが、特殊相対論の基本概念が解った後で固まらないと
会話・問答が成りたない。
0102ご冗談でしょう?名無しさん
垢版 |
2022/06/20(月) 16:47:45.37ID:hJGhFS0b
>>101
特殊相対論の基本概念とは?
0103ご冗談でしょう?名無しさん
垢版 |
2022/06/20(月) 17:06:22.79ID:???
>>102
特殊相対性原理と光速不変の原理の物理的内容だよ.

それが解ってればローレンツ変換による多数の数式を忘れても調べれば直ぐ見つかる。
0104poem
垢版 |
2022/06/20(月) 18:04:48.92ID:???
>>100
なんか>>98が登記さんって、これ絵文字さんに見えてきたんだけど

電磁気スレから登記さんの長文乱れまくってて、絵文字さんなら長文書いたら乱れまくってるだろうし、どちらかわからない

絵文字さんなら説明してくれるだけいつもと違っていいんだけど、登記さんなら計算できるからお願いしたとおり詳しく反証・証明してほしい

もうマークつけないとどちらがどちらかわからない
0105ご冗談でしょう?名無しさん
垢版 |
2022/06/20(月) 18:34:17.83ID:???
>>104
まずpoemは>>98見て最初は自分に賛同してると思ったわけでしょう?
普通ではなかなかあり得ない。
どうやらきみの脳は一旦思い込んだら超強力な認知バイアスかかるみたい。
「登記さんは自分に賛同してる」って思い込んだら、辻褄を合わせるため>>98もだんだん絵文字さんに見えてくる。

そういう傾向は人間だれしもあるんだけどpoemは特に顕著。
自分の思い込みに合わせるように認識を片っ端から歪めてしまうからいろいろ間違って理解しちゃってると思うよ。
人間の脳って不思議だね。
0106ご冗談でしょう?名無しさん
垢版 |
2022/06/20(月) 23:22:47.98ID:cRe+OKM4
>>88
馬鹿野郎!🔥
こっちは絡み合いたくなんかねーよ断じて!🌸(´Д`)🌷(゜д゜)🙄
単に物理学で嘘ついてる現場見たら放置できねんだわ。
…まぁ、ボクは絵文字でわないンだけどネ🌀(o´エ`o)b
0107ご冗談でしょう?名無しさん
垢版 |
2022/06/20(月) 23:57:05.01ID:cRe+OKM4
ちょ物質 ♨275-#956 を覚えているかおまいら。
ここも大概あれに近い感覚を想起しそうなんだが…
言ってもいないこと捏造されるし、たぶんあの何でも万能!
(本人による錯覚)教えたがりの知ったかぶり紛れ込んでるし…😭
ちょ物 ♨276はよ☺
0108poem
垢版 |
2022/06/21(火) 07:15:13.28ID:???
今度は絵文字さんごめん
>>104を書いたの
登記さんってある程度前ちょっとスレで見てた時はこんなこと言わず長文も乱れてなかったから、ここ最近の電磁気スレは他人が混ざってるのかと思ったし、ここの98とか登記さんって皆が言うだけで別人の可能性考えてたけど、直近の電磁気スレのレスと比較したら登記さんだね

絵文字さんが長文書いたら乱れまくるだろうなと予想してたし、乱れまくる人他に知らないのと、乱れまくる人でこのタイミングで書く人絵文字さんしかありえないから、登記さんじゃなければ絵文字さんかなって予想して104を書いた。ごめん絵文字さん
0109poem
垢版 |
2022/06/21(火) 07:31:41.31ID:???
>>105
98を見て賛同してると思ったとか言ってるけど違う。可能性2つ

>まずpoemは>>98見て最初は自分に賛同してると思ったわけでしょう?
>普通ではなかなかあり得ない

1.
>>96 の主張はpoemさんの主張(光速不変垂直適用する)に反論で賛同と反対。105の賛同が98を見て96の始め数行のことなら、これも違う。賛同してない。


2.
自分が98を見て登記さん(敵)じゃなく絵文字さんかなと思ったのを登記さんは98を見た自分が絵文字さん(味方)が書いて自分に賛同してる、と自分が思ったと思ってるなら、こちらの意味でも105の賛同が98の書き込みを敵でなく味方と思った、ってことを言いたいなら、絵文字さんも(味方)ではない(敵)だから賛同してない
0110poem
垢版 |
2022/06/21(火) 07:36:16.93ID:???
というか
光速度垂直成分問題(光速不変垂直適用)【思考実験】
の話題は荒れる元か?
鬼門なのかね
それならこの話題もうここでしない。この話題取り下げで

話戻して
0111ご冗談でしょう?名無しさん
垢版 |
2022/06/21(火) 07:45:23.43ID:???
ちがうちがう
なんちゃら問題だから荒れるんじゃなくて、きみが何話したとしても荒れるの
言ってる意味がほぼわからないから
0112ご冗談でしょう?名無しさん
垢版 |
2022/06/21(火) 08:04:18.39ID:???
てか近頃どのスレもpoemと登記と絵文字が荒れてるだけで、それ以外の人は極めてクールだと思うんだが
0113ご冗談でしょう?名無しさん
垢版 |
2022/06/21(火) 09:17:40.51ID:IkNOLP5p
結局のところ、相対性理論とやらは誰にも理解できていないという客観的事実は隠すことはできないのであった。
0115ご冗談でしょう?名無しさん
垢版 |
2022/06/21(火) 10:13:42.57ID:???
特殊相対論における光速不変は、光がどの方向に進もうがその速度の大きさが不変というだけであって
光源の進行方向に対して平行や垂直方向に分解した各成分が不変などとは言ってない。
光速不変垂直適用とやらはそもそも出発点が間違ってるだけなので問題にさえなっていない
0116ご冗談でしょう?名無しさん
垢版 |
2022/06/21(火) 11:19:48.93ID:???
光速不変の原理を中坊でも分かるように図に書いてやる、その意味だけは分かるだろ
電球○から発した4方向の光子で説明しよう
(1)電球○静止の場合
      ・光子
      ↑
  光子・←○→・光子
      ↓
      ・光子
(2)電球○→方向に速度vで運動してる場合
      ・光子
      ↑
  光子・←○→・光子
      ↓
      ・光子
(1)(2)どちらの場合でも4方向の光子の速さはすべて同じc定数である。

注意:光速不変は物理実験(MM実験など多くの)の事実を基にしてる。
   光子は質量が有る物質ではないから、物質運動の計算式は使えない(禁止)!
   それでも小中高校の計算式を使い矛盾だと言う人は「相間」になるしかない。
(copyright)
0118ご冗談でしょう?名無しさん
垢版 |
2022/06/21(火) 12:16:35.65ID:???
>>116
が光速不変の基本だが、意味が理解出来た人は次のステップアップが必要だ。

(1)恒星-太陽系を慣性系として観測する

   ・(遠方)恒星からの光子の速度c
   ↓
   ●→地球の公転速度v

(2)地球上を慣性系として恒星からの光子を観測する

    ・(遠方)恒星からの光子の速度c
    ↙ (/)
   ● (地球静止)

光子の方向が↙に変化する、これが光行差現象である。
(2)で「光速不変」は ↙の方向の速度がcであるという意味である!
 垂直成分↓の速さではない!

ここまで理解できた人は「光速不変」マスターだ。 めでたしめでたし
(copyright)
0119ご冗談でしょう?名無しさん
垢版 |
2022/06/21(火) 13:05:30.43ID:???
いっつも思うけどこれ誰宛なん?
ライティングがかなり苦手みたいだから、そういうの得意な人をメンターに付けたりスクールに通った方が良いと思うんだが。
0121ご冗談でしょう?名無しさん
垢版 |
2022/06/21(火) 13:16:29.36ID:IkNOLP5p
私宛です。ごめんなさい
0126ご冗談でしょう?名無しさん
垢版 |
2022/06/21(火) 14:05:23.77ID:???
相互コミュニケーションが早いのはこっちだろう
そのメリットに対してデメリットは不要なノイズが多いこと
だからここでは長文は嫌われる
0127ご冗談でしょう?名無しさん
垢版 |
2022/06/21(火) 17:31:49.53ID:Mh8N4qMv
>>112
そうですね、まるで【お通夜みたい】ですよね!💩🤣
0128ご冗談でしょう?名無しさん
垢版 |
2022/06/21(火) 17:39:27.65ID:Mh8N4qMv
>>108
ポエムちゃんは、何が起きても「自分が悪いんだ」と思いこむ性格なの?
そして登キチガイは全て他人に責任転嫁する異常者🤣
…木下登記違い…っていいな🌸(🌀´Д`)🌷
0129poem
垢版 |
2022/06/21(火) 19:13:51.75ID:???
>>128
荒らしで
知障の
トップだからね
poemさんは
0130poem
垢版 |
2022/06/21(火) 19:20:16.91ID:???
>>128
あと馬鹿
頭悪い
精神幼稚

poemさんザ底辺
0131ご冗談でしょう?名無しさん
垢版 |
2022/06/21(火) 20:27:39.29ID:Mh8N4qMv
>>129
だから卑下すんなっての!🐤ヾ🌸( ゚∀゚)ノ🌷
0133ご冗談でしょう?名無しさん
垢版 |
2022/06/22(水) 15:07:53.55ID:???
>>132
無意味が嫌ならおまえ自身が自分で工夫して意味のあるスレにしてくれ
おまえは誰かに呼ばれてきたお客さんじゃないんやで
0138poem
垢版 |
2022/06/28(火) 13:46:27.79ID:???
何の話だっけ

電車の車輪がどうの話か
0139poem
垢版 |
2022/06/28(火) 13:50:03.83ID:n2/q32Am
レス番
>>75まで車輪の話
>>80まで加速の話
0140poem
垢版 |
2022/06/28(火) 13:53:52.21ID:n2/q32Am
レス番
>>81から登記が自分の別スレの垂直問題スレチに突っ込んで脱線した

ちな>>80の自分の質問、普通の小中学生で疑問に思った子がするような普通の質問だと思うから荒らしじゃなくて問題ないよね?
0141poem
垢版 |
2022/06/28(火) 14:05:33.75ID:n2/q32Am
読み返したら
車輪の話スタート
>>27>>28

その前に>>9で自分が垂直問題に触れてたんだね。
それに登記が>>31で返信してて自分が見てなかった
9はスレ建ての意図が垂直問題かなと思ったから質問したんだけど、スレ主の意図は垂直問題は関係ないみたいで
31はまあ9に対する返信だからそこで辞めれば問題なかった。返信は垂直スレですればいいし、登記さん返信くれてたけど、こちらにも書いてるのこのスレで垂直問題議論したかったのかな

スレ主と思しきレスは等速問題しか解説できない(加速とか車輪は解説できない)
0142poem
垢版 |
2022/06/28(火) 14:09:11.26ID:n2/q32Am
あ、>>62で31に対して返信してる
この時気づいたのか
スレチ返信してたのか
0143ご冗談でしょう?名無しさん
垢版 |
2022/06/28(火) 14:17:55.34ID:???
>>脳治療薬ほすぃ脳治療技術ほすぃ脳治療日々経験ほすぃ
>>ネットで自論言って破壊を求める脳治療ほすぃ民

相対論スレとかにスレすると糖質脳が破壊されて正常に戻ると思い込んでるらしい

だから妄想スレを書き込むと、誰かに糖質脳を破壊レスしてもらい正常に戻るらしい

他人が立てたスレも「破壊を求める脳治療ほすぃ民」の仲間だと勘違いして荒らす
0144poem
垢版 |
2022/06/28(火) 14:23:36.32ID:n2/q32Am
62で返信してたから81で返信返されたのか。そのまま脱線

>>77以前の加速の計算できるのか、って他の人の議論状態から>>78加速くらい前例あるでしょ。ってのは繋がる(ちなみに、絵文字さんみたいにネットで調べれば出てくるんじゃない?って想像した。加速度くらいなら等速と同じくもう解決済みで纏めたpdfでもあるんじゃないかって。)
>>79「上から目線に否定するなスレ主は等速しか解説できないって言ってんだから」上から目線に見えたのか。加速ぐらいは解決済みかと思っただけなんだが

そっから自分の>>80が脈絡ぶち切れてるな。加速の質問だから脱線してないけど脈絡がぶち切れてるから脱線したのかな
0145poemしゃん
垢版 |
2022/06/28(火) 14:25:15.01ID:n2/q32Am
>>143
>他人が立てたスレも「破壊を求める脳治療ほすぃ民」の仲間だと勘違いして荒らす

これちがう
0148poem
垢版 |
2022/06/28(火) 14:29:38.19ID:n2/q32Am
>>143
脳治療して欲しぃ民は脳を破壊されたくて破壊行動とってしまって、破壊力の腕と長文を振り回す。症状。発作
0149poem
垢版 |
2022/06/28(火) 14:30:53.19ID:n2/q32Am
>>147
poemさんが相対性理論解説できるわけないから建てるわけないじゃん

別☆人
0150poem
垢版 |
2022/06/28(火) 14:34:10.76ID:n2/q32Am
>>146
今喋ってる意図はスレの軌道修正
0151ご冗談でしょう?名無しさん
垢版 |
2022/06/28(火) 14:34:23.39ID:???
>>148
そうか、誰かのレスがお前の糖質脳を破壊して治してくれると思い込んでるんだな

病院で治療しなければ治らない諦めろ
0152poem
垢版 |
2022/06/28(火) 14:36:26.68ID:n2/q32Am
ともかく選択肢はこんなかんじ?

・スレ主が解説できる等速問題議論
・始めの車輪問題議論
・後の加速問題議論
・別の相対性理論問題
0153ご冗談でしょう?名無しさん
垢版 |
2022/06/28(火) 14:45:22.37ID:9Y71MA/r
相対論は矛盾に見えるから糖質を悪化させるだけかもしれないな
0154ご冗談でしょう?名無しさん
垢版 |
2022/06/28(火) 14:54:24.46ID:v+qS5nuq
文系なんだけど最近特殊相対性理論学んでて同時刻の相対性についてなんだが、
仮に長さLの透明なバスの中に観測者AとL/2のところに光源を乗せて光速に近い速度vで一定に移動しているとする、そして外ではそのバスを観測しているBがいるとする。
ほんで特殊相対性原理と光速度不変の原理ってあると思うんだけど、
特殊相対性原理で「全ての慣性系において物理法則は同じ形に表される」ってあるじゃん?
そしたら光源が一瞬だけ光がバスの両側に発されたとするとAからしたら同時に光がバスの両端に着く。
ほんで観測者Bからしたら光速度不変の原理で、(バスが右側に動いているとすると)バスの左側に行った光は左端に早くついて、右側の光は右端にそれよりもおそく着く、って習ったんだ。

ここで一つ疑問に思ったんだけど、
バスの中で光を発したとしても光速度不変の原理で、Aから見てもBと同じように、左側に早くついて右側に遅く着くはずじゃないか?仮に両端に同時についてるのならA側の速度はbからみてv+cで光速超えちゃうじゃん。
0155poem
垢版 |
2022/06/28(火) 15:02:08.67ID:n2/q32Am
>>154
AがBに対して+c
BがAに対して-c
とすると

Aから見て後方に進む光はBと併走してBから遠ざからない
Bから見て前方に進む光はAと併走して電車の壁につかない
0156poem
垢版 |
2022/06/28(火) 15:03:25.10ID:n2/q32Am
>>155
※時間の伸び縮みのみ変化する場合

空間の伸び縮みでどうなるのかは知らない
0157poem
垢版 |
2022/06/28(火) 15:04:34.38ID:???
>>156
空間の伸び縮みで電車の壁につくし、Bから遠ざかると思うけど。
時間のみなら併走する
0158ご冗談でしょう?名無しさん
垢版 |
2022/06/28(火) 15:31:33.02ID:v+qS5nuq
ごめん。AもBも止まってる。
0159poem
垢版 |
2022/06/28(火) 16:12:37.53ID:n2/q32Am
>仮に両端に同時についてるのならA側の速度はbからみてv+cで光速超えちゃうじゃん。

AがBに報告する光の速度はA+c
Bが光を直接観測する速度は+c
0160poem
垢版 |
2022/06/28(火) 16:16:25.22ID:n2/q32Am
>>158
ならAから見てももBから見ても光の位置同じ。Aの隣にBがいて一緒に見てるんだから。一緒に見てなくてAがBの隣から+cの速度で前に遠ざかってるなら上。
0161ご冗談でしょう?名無しさん
垢版 |
2022/06/28(火) 16:21:06.82ID:n2/q32Am
>>154
>仮に長さLの透明なバスの中に観測者AとL/2のところに光源を乗せて光速に近い速度vで一定に移動しているとする、そして外ではそのバスを観測しているBがいるとする。

と書いて
>>158
>ごめん。AもBも止まってる。

154の問題文(Aがバスに乗ってバスが光速近くで進んでる)と言ってるのに158(AはBの隣から移動してない)と言ってるのAはバスの後方の壁すり抜ける霊体か
0162poem
垢版 |
2022/06/28(火) 16:22:59.45ID:n2/q32Am
>>159
自分の速度込みで自分の見た+cを報告
0163poem
垢版 |
2022/06/28(火) 16:28:28.72ID:???
>>161
Aが電車に留まらない=床との摩擦もないし
0164poem
垢版 |
2022/06/28(火) 16:39:51.72ID:n2/q32Am
相対性理論の問題と言えば
・群速度
・角度で移動する物
が光速超える問題

等速問題、車輪問題、加速問題よりこっち議論して欲しい本音

ま、等速、車輪、加速の話でいいけど。

むしろ普通の学者は光速超える物問題について考えるの興味ない人多いだろうからね。等速、車輪、加速の計算は学者楽しいんだろう
0167ご冗談でしょう?名無しさん
垢版 |
2022/06/28(火) 20:58:57.85ID:v+qS5nuq
AとBは止まってるて言うのは
Aはバスの上で止まってる(Aからしたら止まってる)。
Bは単純に止まってるってことや
誤解招く言い方ですまん
0169ご冗談でしょう?名無しさん
垢版 |
2022/06/28(火) 21:51:48.86ID:???
あんたが学んでいるという「同時刻の相対性」とは何なのか?
たぶんAとBでは同時が異なるという説明のはずなんだが、あんたの主張ではAもBも同時は不変だよね
あんた何学んでるの?
0170ご冗談でしょう?名無しさん
垢版 |
2022/06/28(火) 22:48:19.95ID:v+qS5nuq
>>169
Aからしたら光は同時に到達してBからしたら同時ではない。
即ち「離れた2地点で起こる事象が同時かどうかは、それをみる座標系による。」ってのはもちろん理解してる。
俺が言いたかったのは
音源の動いてる方に音を発しても音の速さには影響ないけど、光の時は速さに影響あるのって聞きたかった。
0171ご冗談でしょう?名無しさん
垢版 |
2022/06/28(火) 22:51:00.72ID:v+qS5nuq
訂正: 「速さ」→「速度」
0173ご冗談でしょう?名無しさん
垢版 |
2022/06/28(火) 23:41:14.71ID:???
>>154「A側の速度はbからみてv+cで光速超えちゃうじゃん。」
これが聞きたいポイントだな。
Bから見て、Aは右に速度vで動いてる、光は左に速度cで動いてる。
足したら c+v じゃんてことだな。
うん足し算したらそりゃ c+v になるよ。

だから何っていうか、その値に物理的な意味はなんもない。
相対論で相対速度を計算するときは、単純に2つの速度を足し算しても意味のある値は出てこない。
0174ご冗談でしょう?名無しさん
垢版 |
2022/06/28(火) 23:48:18.68ID:v+qS5nuq
>>172
やったよ。

バスは一定の速度vで動いてるんだから右側に向かってる光はBから見るとv+cで光速度不変の法則に矛盾してない?って言いたかった。
それで今光源の速度に関わらず光の速度が一定なら
バスの右側に向かってる光はv+cではなくcなのだから結局Aから見てもBから見てもcだから同時に着くはずでは?ってことが言いたかった。
0175ご冗談でしょう?名無しさん
垢版 |
2022/06/28(火) 23:55:07.55ID:???
>>174
光の速度はAから見てもc、Bから見てもcだ。
一方バスはAから見れば止まってるんだから、光はバスの両端に同時につく。
Bから見ればバスは右に動いてるんだから、光は左の壁に先にぶち当たるでしょう。
0176ご冗談でしょう?名無しさん
垢版 |
2022/06/28(火) 23:58:35.82ID:v+qS5nuq
>>173
何で意味がないんだ?
v+cが引っかかる
0177ご冗談でしょう?名無しさん
垢版 |
2022/06/29(水) 00:03:54.64ID:???
>>176
逆に何の意味があるの?
Aから見た光の速度はc、Bから見た光の速度もcっていってんのに、
じゃあその v+c って何から見た何の速度よ?
0178ご冗談でしょう?名無しさん
垢版 |
2022/06/29(水) 00:09:46.35ID:PCDi/rkg
>>175
AからみてもBから見てもcなら二人から見ても同時に着くはずじゃない?
0179ご冗談でしょう?名無しさん
垢版 |
2022/06/29(水) 00:14:40.04ID:???
>>178
光の速度は同じなんだけど、バスの速度が違うでしょう。
バスに乗ってるAから見ればバスは止まって見えるってのはわかるか
0181ご冗談でしょう?名無しさん
垢版 |
2022/06/29(水) 00:31:27.17ID:PCDi/rkg
>>179わかる

バスが一定速度で動いていてAは車内が止まってるのと同じ状態に見えるからボールをv1で投げるとそのボールはAから見たらv1、Bからはv1+v
だけどこの場合車内が止まっているのと同じ状態なら
Aは同時刻に光が到達するんだよね? 
なら結局これだとBから見てv+cで光速度不変の原理を守れてないくないか?
0182ご冗談でしょう?名無しさん
垢版 |
2022/06/29(水) 00:41:40.01ID:???
>>181
> ボールはAから見たらv1、Bからはv1+v
Bから見たらv1+vぴったりではないってのが相対論。

> なら結局これだとBから見てv+cで
Bから見てもcってのが相対論。

なんでそうなるか、もうちょっと勉強した人でないと説明するのは難しいけどそういうもんだと思って。
0183ご冗談でしょう?名無しさん
垢版 |
2022/06/29(水) 01:04:08.37ID:PCDi/rkg
>>182
わかった。先生に聞いてみるよ。

Bから見てもcってことは、Bから見たらバスはv=0に見えるからv+c=cってこと?
0185ご冗談でしょう?名無しさん
垢版 |
2022/06/29(水) 01:14:06.21ID:PCDi/rkg
さーせん。色々ありがとうございました。
0186poem
垢版 |
2022/06/29(水) 03:24:10.47ID:IIwMLQhy
>>185
車内カメラと速度違反カメラがあったとして
バス内の車内カメラは光が両壁に同時に着いてるとhddに記録されるかWi-FiでALSOCに送られる
Bの傍の速度違反メーターがバス内の後ろ壁に光が着弾する地点に設置されてたら前壁に光が着弾してない様子がhddもしくはWi-Fiか有線でPOLICEに送られる
互いの映像は食い違ってる
ALSOCが自分の映像を信じれば光はAに対して-c←A→+c
POLICEが自分の映像を信じれば光はBから見て-c←B→+c
もしALSOCがPOLICEの映像を信じれば光はAから見て-c-b←A→-b+c
もしPOLICEがALSOCの映像を信じれば光はBから見て-c+a←B→+a+c
と報告速度が合成される
0187poem
垢版 |
2022/06/29(水) 04:12:30.75ID:IIwMLQhy
>>185
貴方が言う合成速度は速度差を持つ観測者の数だけ観測者から見た光の位置が観測者の報告を聞いて纏めると分裂するのをわかってないだけ
相対性理論では分裂するのが光でなく観測者の数だけ時間空間が分裂する。…と今相対性理論の方を対象的に書こうとして相対性理論が何言いたいか今更わかった。

相対性原理と光速不変原理を両立すると両者に従う者の間で位置ズレを起こす。
これを相対性原理の観測者の置かれてる観測者の数の無限個の時間空間が分裂して光子は分裂せず1個のままなのか
光速不変原理の光子が観測者の数だけ無限個に分裂して観測者同士の置かれてる時間空間は分裂せず1個のままなのか
2つの方法のうち一方を取らなければならない
そのうち時間空間(観測者の方)が分裂して、分裂した無限個の時間空間を1個に合成すると時間空間の歪みとして見えると言う理論が相対性理論なんだね

相間だからどうでもいいけど
0188ご冗談でしょう?名無しさん
垢版 |
2022/06/29(水) 04:14:24.01ID:IIwMLQhy
>>187追記

相対性原理と光速不変原理を両立すると両者に従う者(相対性原理(観測者)、光速不変原理(光))の間で位置ズレを起こす
0189ご冗談でしょう?名無しさん
垢版 |
2022/06/29(水) 04:15:25.18ID:IIwMLQhy
>>188
()←括弧内追記

追記じゃない訂正。ごめん
0190poem
垢版 |
2022/06/29(水) 06:44:01.78ID:IIwMLQhy
186と187から
位置ズレにより観測者の時間空間だけが歪むんじゃなく
観測者ごとに世界の事象の事実も歪んでるんだね。相対性理論を信じると
0191poem
垢版 |
2022/06/29(水) 07:00:11.73ID:IIwMLQhy
スレチだけど
シュレディンガーの猫の分岐世界(とエヴェレットの多世界解釈)嘘だね。観測者ごとに分裂してる世界の事象の事実が1つに合成されて歪んだ事象の事実になってて、観測者集めて摺り合わせたら歪んだ世界の事象の情報になるだけで、分岐世界(多世界)になるためには分裂したままじゃなきゃいけない。合成するなら分岐しない。

相対性理論は異なる速度差の観測者の時間空間を光を絶対(1個)にして観測者一人ごとに1個の時間空間が分裂したのを、ローレンツ変換で1個の歪んだ時間空間として合成するもので、この計算だから

・相対性理論を信じない…観測者の時間空間が分裂したまま合成しない…分岐世界(多世界)
・相対性理論を信じる…観測者の時間空間が分裂したのを合成して歪んだ時間空間にする…分岐世界(多世界)にならない

分岐世界(多世界)・シュレディンガーの猫は相対性理論的に嘘

擦り合わせたら情報が噛み合わない、事象の事実が歪んで、宇宙の違う運動・観測者の数だけ食い違って報告される、宇宙全体で事象の事実が絶対でないのが相対性理論の世界論、世界観だね
0192poem
垢版 |
2022/06/29(水) 07:01:44.81ID:IIwMLQhy
>>191
186と187からそうなる

スレチだからこちらに脱線注意
0194ご冗談でしょう?名無しさん
垢版 |
2022/06/29(水) 13:32:43.23ID:???
病院で精神病のお医者さんにこのスレッドをコピーして読んでもらえば

キミがお医者さんに言葉で自分の症状を説明できなくとも、適切な治療をしてくれる
0195poem
垢版 |
2022/06/30(木) 11:09:41.25ID:FrZVSvR9
>>194
自分が知障寄りのギリ健ギリ障なのは認めてるけど

上は相信(健常)が正しいならという立場で小中学生が習う相対性理論の本の内容をいくらか掘り下げた相信のそのままがそうなる内容を書いてる

自分は相間(障害)で自分の論は光遅度説、時間s物理が時刻=現在=空間の位置=今の宇宙の全ての記録の時間連続を物理に組み込む物理量だと思ってて、光が時間連続の物理だから光は普通の運動と物理的に違うから相対性理論を作る必要がそもそもなく相対性理論なくても自明だ、という論と光速不変垂直問題の論から相間が本心で、上の相信書き込みは思う物を棚に挙げて議論の流れに竿刺すために相信の理論や考えを考えてる

>病院で精神病のお医者さんにこのスレッドをコピーして読んでもらえば
>キミがお医者さんに言葉で自分の症状を説明できなくとも、適切な治療をしてくれる

書いてる日本語の文章は頭の壊れてる障害がわかる病院行きレベルだろうけど
病院行きレベルの日本語だけど相信の立場に立って、相信の理論を肯定し相信の考え(小中学生が本で習うレベルのもの)を深めて書いてるのだから、内容自体はこの日本の普通の健常な大多数の頭いい老若男女の共通の信じてる論理の深く掘り下げてなかった部分を書いてるに過ぎない。だから日本語は知障だが内容は健常ド正中。

でもこの健常ド正中論を日本語で見える形に書いたの読むと健常者の論理が障害に見えるね。

すると194の引用は194を含めた健常の老若男女の大多数の人にブーメランなんじゃないか?
相対性理論信じてる人の考え至るべきものを書いてる内容だから相対性理論信じてる人はやがてこの内容の理屈に到達する。相対性理論信じてる人は読んで障害に見える考えに到達する。ブーメランなんじゃないか
0197ご冗談でしょう?名無しさん
垢版 |
2022/06/30(木) 16:56:56.54ID:???
相信が健常とか何を言ってるんだ
相間だろうが相信だろうが理解の代わりに信じる奴は馬鹿でしかない
理解してればどちらにもならん
まあ、この事さえ理解せんのだろうな
0198ご冗談でしょう?名無しさん
垢版 |
2022/07/01(金) 13:44:06.71ID:???
同じ事象を異なる観測者が観測すれば異なる観測値になるのは
相対論だろうがガリレイ変換だろうが同じことだと思うが、
これを時間空間が分裂とか表現するマヌケはpoemだけ。
これのどこが健常ド正中なんだか
0199ご冗談でしょう?名無しさん
垢版 |
2022/07/01(金) 16:07:03.32ID:PenHPNjK
ホーキングの虚数時間からの宇宙生成でおもいついたが
虚数時間の世界があるなら宇宙誕生時だけではなく今もあるだろうし、突然消滅するほうがおかしい
時間は時空として扱われるし、それなら虚時空もいまもあるだろ?
虚時空を前提した理論をとりあえず作るといいことある?
0200ご冗談でしょう?名無しさん
垢版 |
2022/07/01(金) 16:10:56.16ID:PenHPNjK
虚時間は、1次元数直線が平面的になるだけで想像しやすいが・・
虚時空は、何から何になるのか、よくわからん
そもそも相対性理論の時空の扱い方もよくわからん
0201ご冗談でしょう?名無しさん
垢版 |
2022/07/01(金) 16:16:17.53ID:PenHPNjK
時間のみを虚時間に拡張したのち、空間と融合したやつを虚時空にもできるし、
虚時間と虚空間から、虚時空を作るのもできるし
直接、時空から虚時空を作るのもできそうだが
それぞれ別物になりそうだが、どれが適切なのかわわからん
0202ご冗談でしょう?名無しさん
垢版 |
2022/07/01(金) 16:31:58.05ID:BNanq6h2
妄想は限りなく縮退する
0203ご冗談でしょう?名無しさん
垢版 |
2022/07/01(金) 16:39:37.45ID:PenHPNjK
ホーキングの虚時間宇宙のやり方がそもそもわかってないしな
とくに最初から拡張したわけではなく X^2 = -1 という計算になったとき
X = i と解いただけなのか?
なぜビックバンの特異点がなくせるのか?
0204poem
垢版 |
2022/07/01(金) 17:16:47.28ID:iuog+AId
そもそも虚数の実在性スレ
虚数が数学的にどこに位置してるのか
それが物理的にどこに実在しているのか
0205poem
垢版 |
2022/07/01(金) 17:38:20.71ID:???
>>196-198
虚数が物理的にどこに実在するか
虚数の物理の具体例リストアップおね
電子の抵抗が実数抵抗に、中性微子の抵抗の親戚が虚数抵抗として出るとか、色んな物理の計算上出る虚数リストアップして?1人か2人か3人かわからないけど一番この板の学者として実力あって頭良さそうだから
0207poem
垢版 |
2022/07/01(金) 18:55:26.32ID:???
>>196
は一言コメントくれたけど
>>197-198
は一言もコメントしないの?
色んな虚数出てくるのリストアップしてくれたら虚数、虚時間虚空間の議論に入れるけど
虚時間虚空間の話題はいらないか。頭のいい学者になるとこの手の興味なくなるのかな。
一言もコメントしないのそういうことだよね
じゃ次の話題
0208ご冗談でしょう?名無しさん
垢版 |
2022/07/01(金) 19:19:50.50ID:???
おまえ普通の実時間とは別に虚時間があると思ってないか?
ホーキングの虚時間なんて、普通の実時間に虚数単位iを掛けただけのもので
ただの変数変換に過ぎない。実質同じものだぞ
(t,x,y,z)のかわりに(it,x,y,z)を使うと、ミンコフスキー時空の問題がユークリッド空間の問題に置き換わって
見通しが良くなる(こともある)というだけのもの。昔の教科書にはよく見られたが最近はほとんど見ない。
0210ご冗談でしょう?名無しさん
垢版 |
2022/07/02(土) 05:30:36.26ID:yWOJkNvI
>>208
ホーキング論文じたいがわからないので、わからないが
虚時間宇宙・空間がそれだとして0秒時点で切り替わる理由がわからんが
両方どっちも存在しているが、虚時間宇宙・空間は無になってなんの影響を与えなくなってて、考えなくていいだけ?
0211ご冗談でしょう?名無しさん
垢版 |
2022/07/02(土) 05:49:26.64ID:yWOJkNvI
ざっくりいうと虚時間宇宙空間は、あの世みたいな物といえるとおもうが
相対性理論も、量子論も、量子重力対応もかなり正しいと思えるが
現世とあの世を同時に考えた空間なら、量子論=相対論の一致を証明できるのか?
素粒子の生成消滅、量子もつれも、あの世に行ってるか、影響の可能性
0212ご冗談でしょう?名無しさん
垢版 |
2022/07/02(土) 13:54:12.09ID:???
実時間が存在すれば虚時間も存在する
片方を考えれば他方は自動的に満たされるから分かりやすい方で考えれば良い
トンネル効果の途中では虚時間しか存在しないが実時間での結果だけで良いなら
虚時間を考える必要はない
0214ご冗談でしょう?名無しさん
垢版 |
2022/07/02(土) 15:13:54.27ID:yWOJkNvI
ホーキング論文ざっくりみて分かる気がしないので諦めた
0215ご冗談でしょう?名無しさん
垢版 |
2022/07/02(土) 18:30:00.90ID:???
>>213
どっちも関係ないじゃん
1つ目はトイモデルでインフレーションが終わる時に3次元球になる確率が大きい事
2つ目はカーブラックホールでエントロピーを求めたもの
0216ご冗談でしょう?名無しさん
垢版 |
2022/07/02(土) 19:11:27.13ID:yfPtHudr
ところで「チンポがシコシコする」という日本語表現は、学術的に正しいと言えるのか?

チンポ「を」シコシコするのではなくて、チンポ「が」シコシコする。この場合、「チンポ」は主語となる。

オブジェクト指向で言う「集約」は2種類あって、全体(俺)と部分(チンポ)が繋がっている場合と、
全体(俺)と部分(チンポ)が別々になっている場合とが考えられる。けれども「チンポ」はそれ自体
が独立した生き物であり、所有者の意思とは無関係に、自ら勃起して「シコシコする」。
例えば寝てる時にエロい夢みて朝起きてみたらチンコが勃起して射精してたとか。

違うか?

「胸がドキドキする」は良いが、「チンポがシコシコする」はダメな理由を、50字以内で述べろ!
0217ご冗談でしょう?名無しさん
垢版 |
2022/07/02(土) 21:56:53.38ID:???
 
日本の最大の弱みは
 人脈(人間関係)至上主義!
 
政界でも企業内でも
 人間関係だけの人脈コジキ営業職
が幅を効かして有能な人材を押しのける
 
企業内では
 人間関係だけの人脈コジキ営業職
が威張り腐って、貴重な技術職は不遇になる 
   
だから優秀な技術職が 
 中韓に引き抜かれて技術パクリされる 
そして日本国内に残るのは社会的生産性ゼロの 
 人間関係だけの人脈コジキ営業職だけになる
   
「やーどーもどーも」だけで注文をもらう
 客のクツの裏をナメて仕事を恵んでもらう
人間関係だけの人脈コジキ営業職が出世して
 優秀な技術職が薄給で冷遇されてれば
日本の科学技術力が衰退するのは当然だ
 
そして行き着く先の最悪ケースは
 【KDDI】のシステムトラブルだ!  
本来なら社長にすべき優秀な技術職を
プライドさえ捨てれば馬鹿でも出来る  
 人間関係だけの人脈コジキ営業職
よりも冷遇している
 世界でもマレにみるアホ企業KDDI
 
0218ご冗談でしょう?名無しさん
垢版 |
2022/07/02(土) 23:56:48.08ID:???
やぁ登記=888八兵衛公さん、こんな所でもご活躍でんなぁ!?🐤
ところであんた、いったい何のために物理板にずっと帰省してるの?(‘д‘ )🌷
0219ご冗談でしょう?名無しさん
垢版 |
2022/07/03(日) 04:52:23.60ID:lagd+UGL
>>215
"マルチバースの難題"を解決したホーキングの「遺言」 「最後の論文」に書かれていたこと

論文の題名は「永遠のインフレからのスムーズな脱出?」(A Smooth Exit from Eternal Inflation? )。

「有名宇宙論」への異議申し立て
まず、これまでの宇宙論の定説というか、人気学説として「永遠のインフレーション」というものがあったことを押さえておきたい。
広大な宇宙全体としてインフレが止まったわけではなく、急膨張が止まったのは、ごくごく一部であり、その他の領域は永遠にインフレが続いているという仮説だ。

われわれの宇宙も、無数に誕生したポケット宇宙の1つにすぎない。ポケット宇宙という言い方が気に入らなければ、「マルチバース」とよんでくださってもかまわない。
だが、この人気学説には大きな弱点があった。
無数のポケット宇宙が、それぞれ異なる物理的な特性をもっていて、重力が強くてブラックホールだらけになったり、重力が弱くて星や惑星が形成されなかったりして、ようするに「なんでもあり」になってしまうのだ。

どんな宇宙でも可能だとしたら、われわれの宇宙も偶然こうなっただけで、「どうして今のような物理法則になったのか」を問うことができなくなってしまう。
もちろん、それが真理なのであれば仕方ないが、ホーキングとハートグは、この人気学説が気にくわなかった。
永遠のインフレーション学説、言い換えるとマルチバース学説は、なんでもありなのだから、検証不可能ではないか。
それが、最後の論文の出発点だった。

従来の「永遠のインフレーション」学説は、主にアインシュタインの一般相対性理論を活用し、そこにちょっぴり量子論の効果を入れ込んでいたが、ホーキングとハートグは、最初からホログラフィー原理で計算をしてみた。
すると、宇宙は全体として、有限・スムーズだという答えになった!
なんだか、わかったような、わからないような解説で申し訳ないが、結論はわかりやすい。

【結論】 ホログラフィー原理を応用して、時間を投影して計算した結果、これまでのような「なんでもあり」のマルチバースではなく、ほとんどのマルチバースが似ている可能性が出てきた。
https://gendai.ismedia.jp/articles/-/55816
0220ご冗談でしょう?名無しさん
垢版 |
2022/07/03(日) 04:53:53.94ID:lagd+UGL
>>215
虚時間宇宙論は、最後の論文ではなくそれ以前のやつか
0221ご冗談でしょう?名無しさん
垢版 |
2022/07/03(日) 05:02:10.99ID:lagd+UGL
無境界仮説と最後の論文は関係はしてるが、別だったか


「宇宙の始まり」を消し去ったホーキングの「すごい世界観」
https://gendai.ismedia.jp/articles/-/55317?imp=0


ホーキング博士、最後のセオリー:彼が多元的宇宙について考えていたこと
「無境界仮説」と「多元的宇宙」の関係
ホーキングの有名な、それでいて実証不可能な業績のひとつに、1983年にカルフォルニア大学サンタバーバラ校の物理学者ジェームズ・ハートルと共同で提唱した「ハートル・ホーキングの無境界仮説」という概念がある。
無境界仮説によると、もし時間をさかのぼってビッグバン前にたどり着けたとすれば、そこには空間や時間の概念自体が曖昧になってしまう場があるという。

そこは「特異点」のようなはっきりとした点ではなく、むしろ北極を見渡せばどの地点が正しく北極点なのかわからなくなるように、境界のない、スムーズなものなのだとホーキングは形容する。

「時空の過去には境界が存在しなかったというのが、宇宙初期の物理的なコンディションであったはずです」と、ホーキングは理論を説明する動画で語っている。
無境界仮説の要となっているコンセプトは、原子のように小さな特異点は、粒子のようにも波のようにも振る舞うことができるという量子論的な概念だ。「無境界仮説は量子重力を基にしたビッグバン・モデルなのです」

しかし量子論を用いた仮説は途方もない可能性をも内包する。今回の論文の共著者で、ベルギーのルーヴェン・カトリック大学の物理学教授であるトーマス・ハートグ博士は、『WIRED』日本版の取材に次のように説明する。
「わたしたちは徐々に、無境界仮説のモデルはひとつの宇宙だけではなく、無限の宇宙をつくり出してしまうことに気づきはじめました。それらは並行して存在する宇宙の集合体のことで、マルチヴァース(多元的宇宙)と呼ばれています」
https://wired.jp/2018/04/10/hawking-last-theory/
0222ご冗談でしょう?名無しさん
垢版 |
2022/07/03(日) 05:17:03.60ID:aqjfull4
何を言っても実証不可能な戯言。あるいは何でも言える
0223poem
垢版 |
2022/07/03(日) 11:17:05.14ID:???
虚時間の前に負の時間空間についてpoem説

正の数字…順順序(順序が正しい)
負の数字…逆順序(順序が逆)

+n+n+n…=×m←順順序
÷M=-N-N-N…←逆順序

この宇宙空間、3次元全方向…正の空間…正しい順序の空間。
この宇宙空間に負の空間(逆の順序の空間)は存在しない

しかし空間の右を正の数mとしたら左は負の-数mになる。

負の空間がないのに、負の空間の計算になるのは何故か?

→負が逆順序だから。

左向きは右向きに対して逆順序だから負として空間の計算に出る

→しかし注意点は負として計算に出るからと言って、左が右に対し負の空間というわけではないこと。あくまで負の空間と同じ逆プロセスであるだけで、負の空間の表出ではない。

poemさん何言いたいかっていうと
虚時間、虚空間、の物理はこの宇宙の物理の計算に出てくるの、虚の計算の仕組み、なんのプロセスを表すかを研究しなきゃいけない。
その相対性理論やインフレションで虚の計算がでるのはほんとの虚空間虚時間でなく虚の空間時間と同じプロセスの正の空間時間の物理かもしれないし、虚空間虚時間が関わってるかもしれないし

虚って何だろうね。
もしかして逆順序のさらに逆順序かな…。考えられるのそれしかないかな…
0224poem
垢版 |
2022/07/03(日) 11:38:02.41ID:???
>虚って何だろうね。
>もしかして逆順序のさらに逆順序かな…。考えられるのそれしかないかな…


負(逆順序)の 計算には問題があってね

順順序(+)×順順序(+)=順順序(+)
順順序(+)×逆順序(-)=逆順序(-)
逆順序(-)×順順序(+)=逆順序(-)
逆順序(-)×逆順序(-)=順順序(+)

逆順序の逆順序が元に戻って順順序になってしまう問題なんだ
逆向きを逆からみたら正しい向きになる。当然なんだけどね。
でも順順序の物は何個掛けても符号そのままなのに、逆順序の物は何個も掛けたら奇数偶数で符号が入れ替わっちゃう。
それに、これは順順序の人の世界を基準にしたから、であって、逆順序に住まう人の世界を基準にしたら、逆順序を何個掛けても符号が入れ替わらない、順順序を何個も掛けたら奇数偶数で符号が入れ替わる、って逆の視点になるじゃん

つまり順順序、逆順序とかそういうのも、順順序の世界の主観でしかないわけなんだよね。

そして順序は逆順序2つで順順序になる。
逆順序から見たら順順序2つで逆順序になりそうだけど、
もし逆順序から見て何順序2つで逆順序になるものがあったら?
その何順序2つで、逆順序の世界の人から見た視点で、本当に逆順序になるかわからないけど、
順順序の世界の人から見た視点で、逆順序世界の現象が、何順序2つで逆順序に戻ったように見えたなら、それは逆順序の未知の逆順序。
逆順序の未知の逆順序が逆順序から見て、逆順序に戻らないかもしれないし、あわよくば逆順序の未知の逆順序は順順序の逆順序から見たものかもしれない。

…虚数って、やっぱり逆順序とかそういう解釈とかになりそうかな?それしか解釈なさそうなような。

虚数が負から見た正の数字か
それとも虚数は正とは別の場所にある負の世界の未知の逆か

…まあでも、虚数とは逆順序の逆順序とわかっても、何も数学的正体の深いところまでは解明できないよね。浅いところ削れたところでそこから深いところには到達できない。そして物理的な実在…は虚数の物理は異なる虚数の世界だから、正のこの宇宙に実在はしないけど、物理的なプロセスの正体、正のこの宇宙にプロセスとして出てくる虚数の正体と意味がわかればいいね
0225ご冗談でしょう?名無しさん
垢版 |
2022/07/04(月) 13:37:57.22ID:???
>>219
問題はそれが重力だけのトイモデルってとこ
他の物理法則を高次元のコンパクト化で説明しようとすると
内部空間が無限に存在してやはりマルチバースになってしまう
0227ご冗談でしょう?名無しさん
垢版 |
2022/07/05(火) 23:54:57.68ID:Zr0zVtbQ
あたらしいめちゃくちゃすごい素粒子を見つけた
0229ご冗談でしょう?名無しさん
垢版 |
2022/07/06(水) 13:58:20.40ID:ONL05Uhd
なにそれ?
0232ご冗談でしょう?名無しさん
垢版 |
2022/07/07(木) 18:16:19.46ID:AcO0+b+H
どっちにしろ間違っておる
0234ご冗談でしょう?名無しさん
垢版 |
2022/07/08(金) 04:31:24.83ID:UseS4b6a
なにそれ?
0236ご冗談でしょう?名無しさん
垢版 |
2022/07/08(金) 08:46:32.91ID:UseS4b6a
なんだ、言葉のお遊びか。
0239poem
垢版 |
2022/07/09(土) 16:05:38.99ID:???
ゴミ提供
アキレスと亀(パラドックス)の話
亀先行、アキレス後行
・アキレスと亀の速度比を100:1にして計算すると→追いつけない→パラドックス
・アキレスと亀を速度差で100‖1にして計算すると→追いつける→パラドックスでない

・後者の時間経過は普通の1秒刻み
・前者の時間経過は追いつく極限まで秒を無限に分割

・後者のパラドックスでない→亀とアキレス、先行後行の差分の時間で追いつく
・前者のパラドックス→亀とアキレスは常に1:100の距離にいる。追いつくまで比が1:100=0:0.000000000000000…=0:0になる時間掛ける(等差時間でなく、等比時間)。無限に分割した無限に掛けた等比時間の先で追いつく。その等比時間で無限の時間は等差時間で僅かな時間。

↓↓
相対性理論の光と普通の速度の追いかけっこがまさに光=アキレス、普通の速度=亀、で相対性理論の言ってるのは、時間は等差時間の絶対時間しかないなんて嘘で、普通の速度と光の速度の追いかけっこ、普通の速度の状態により等比時間の相対時間が実在する、という理論

↓↓
光をアキレス、普通の速度を亀として相対性理論を考えてみよう
↓↓
と言うゴミテーマ
↓↓
乗る?暴言の応酬のネタとしてはゴミ過ぎて優良じゃない?
0242poem
垢版 |
2022/07/10(日) 00:12:27.58ID:???
>>240
まじ?手垢なのか…
0243poem
垢版 |
2022/07/20(水) 15:05:40.15ID:???
>>198
0198 ご冗談でしょう?名無しさん 2022/07/01 13:44:06
同じ事象を異なる観測者が観測すれば異なる観測値になるのは
相対論だろうがガリレイ変換だろうが同じことだと思うが、
これを時間空間が分裂とか表現するマヌケはpoemだけ。
これのどこが健常ド正中なんだか



こっから話し合い再開できる?
0244poem
垢版 |
2022/07/20(水) 15:25:51.61ID:???
>>197
>相信が健常とか何を言ってるんだ
>相間だろうが相信だろうが理解の代わりに信じる奴は馬鹿でしかない
>理解してればどちらにもならん
>まあ、この事さえ理解せんのだろうな

>理解してればどちらにもならん

今気づいたんだけど、本当に物理できる学者は相対論「信じても、疑ってもいない」ってこと?
本当に頭いい学者は相対論を高度に理解してても、相対論が正しいか間違ってるかの、原理レベル(相対論から→(相対性原理・光速不変原理、という立脚する原理の意味でなく)→ローレンツ変換から予言される時空の歪みの起こるという原理のこと)の理論的証明ができないから─できないのがわかるから、信じもしないし疑いもしない、静観するって言ってるの?

つまり
・実力ある物理学者「相対論の予言する時空現象を証明できない。故に相対論が正解か不正解か、証明できない」
・相対論信じる学者←(相対論も物理を本当の意味で理解できない、ただ全て肯定して受け容れるだけの考えなし)
・相対論疑う学者(人)←(相対論や物理を理解する段階にも至らない、自分が頭良いと自惚れてるだけの勘違い)
・相対論信じても疑ってもいる学者←(まとも)
・相対論信じてなく疑ってもいない学者←(優秀)

197は優秀…これは真だろうけど
そういうことを言ってるのかな197は。
相対論を静観する人間は健常だけど、信じる疑うで論争する人は全員障害、って
0246poem
垢版 |
2022/07/20(水) 15:29:17.54ID:???
すると>>197は観測値は変わる物だからそこから相対論が正しいとも証明できないし、間違いとも反証できない。(→poemのしてた反証に対して反論)

ってことかな?
0247poem
垢版 |
2022/07/20(水) 15:30:30.80ID:???
>>245
そういう意味でしょ?
予想したの違う?
0248poem
垢版 |
2022/07/20(水) 16:21:44.74ID:???
ちなみに観測値が分裂するじゃん。相対論計算すると。バラバラな点を記録すると。バラバラな点まで歪みとした連続の曲線になるんだろうから、無限の分裂の合成が歪みじゃん。世界が分裂するか光が分裂するかじゃん
0250ご冗談でしょう?名無しさん
垢版 |
2022/07/20(水) 16:33:47.73ID:???
同じ事象を異なる観測者が観測すれば異なる観測値になるのは
相対論だろうがガリレイ変換だろうが同じことだと思うが、
これを分裂とか歪みとか表現するマヌケは相変わらずpoemだけ。
病院行け
0251poem
垢版 |
2022/07/20(水) 16:42:26.82ID:???
異なる観測値の時点で観測値が観測者により分裂してるじゃん(異なる測定)
0252poem
垢版 |
2022/07/20(水) 16:47:15.05ID:???
測定した目盛りの真実と物理的真実が同じじゃなく、測定した目盛りは物理的実体でなく架空で、測定した観測値が分裂する=物理が(歪みの離散点で)分裂するわけでなく、測定と物理は=架空と真実だ、って言うなら、測定して実証、実験して実証という物理の手続きは理論の証明にならなくなるけど、そう思うの?
0253poem
垢版 |
2022/07/20(水) 16:51:06.92ID:???
ここの学者、どこでも学者みんな、物理量は物理的実体でなく架空、単位は物理を表してない物理と乖離してる、とかいうけど、そんだったら実体全部無効になるけど、それが真実?
0254poem
垢版 |
2022/07/20(水) 16:52:06.73ID:???
>>253訂正
実体←実験
0255poem
垢版 |
2022/07/20(水) 16:59:17.71ID:???
正しい真実な物理量(※物理量って複合単位の事なのかな?)、単位は物理から乖離してるはずが無いんだけど

ここと、どこでも学者がみんな、物理量や単位は物理的真実を表さない、物理から乖離してる、って言ってるの、そう思って信じてるの、

自分の習った式が、自分の信じてる物理的常識の式が、先人の偉人の出した式が、他の人が全員理解して先行研究を使ってる式が、自分の出した式が、自分の出そうとしてるこれから先の研究と式が

真実から乖離してて間違ってるだけな気がするpoemさん

真実から乖離しない物理量、単位は物理的真実を表し
真実から乖離してるそれらや、式が物理的架空な妄想なだけ

物理的解釈や物理的機構も
0256poem
垢版 |
2022/07/20(水) 17:02:06.75ID:???
観測値が違うなら、物理的真実が分裂してる歪んでる(相対論を信じるなら)か
観測値が違うのを、統一できる真実がある(相対論は結局間違ってる)か

のどちらか
0257poem
垢版 |
2022/07/20(水) 17:04:03.17ID:???
相対論を信じるならタイムトラベルや事実のパラドックスが起こる、歪みや分裂を信じなきゃいけない。

物理的真実が歪みや分裂しない、統一されるなら、相対論は間違ってる
0258poem
垢版 |
2022/07/20(水) 17:07:26.66ID:???
ちなみに相対的な速度は、観測値が違うとかじゃなく、Am/s−Bm/s=(A−B)m/sで、別に観測値が違うじゃなく、AとBの速度差だから違うだけで分裂してない

分裂してる、観測値が違うのは、時空の歪み、長さが縮んだり、時間が遅くなったり。
0259poem
垢版 |
2022/07/20(水) 17:10:04.41ID:???
時空が歪むから、タイムトラベルや事実のパラドックスが起こるわけで、

これが観測者による世界の分裂。
光が分裂すれば世界は分裂しない
光が分裂しないなら世界が分裂する
無数の分裂点を合成すると連続した歪みになる。世界の連続体、連続した多世界
0261poem
垢版 |
2022/07/20(水) 17:14:03.94ID:???
>>259
連続変化する位相違いの多世界の居る場所が変わるのが世界の分岐の多世界。

もし光が分裂するなら、相対論は間違い。

poemさんは物体は慣性座標に従い、光は非慣性座標に従い、慣性座標と非慣性座標の座標同士は相互作用しないから、慣性座標から非慣性座標の光がどこからでも見えるから分裂したように見えるだけ
と思ってる。poemさんの光遅度説
0262poem
垢版 |
2022/07/20(水) 17:15:49.26ID:???
>>260
多様体ってわかる?

>光が分裂しないなら世界が分裂する
>無数の分裂点を合成すると連続した歪みになる。世界の連続体、連続した多世界
>連続変化する位相違いの多世界の居る場所が変わるのが世界の分岐の多世界

あってるでしょ?
0264poem
垢版 |
2022/07/20(水) 17:19:09.89ID:???
>>260
宇宙空間が多様体で、1mが測る観測者や時代により変わる。1mの単位は物理的事実から乖離してて、物理的真実じゃない
とか言いたい?

観測・実験での過去の先人の実証が全部無効になるよ
0265ご冗談でしょう?名無しさん
垢版 |
2022/07/20(水) 17:20:40.84ID:???
>>264
いや「多様体」を知ってるかと聞いてるんだけど
知ってるなら念のため自分の口で説明してくれ
次の話はそれをした後かな
0266poem
垢版 |
2022/07/20(水) 17:22:23.27ID:???
>>263
知らなかったけど予想でレスして、知らなかったから今調べたけど、
グーグルのトップの説明見ると相対空間のことでしょ?

レスしたの合ってるじゃん

全ての物理が多様体=相対空間、宇宙は相対空間の多様体。…→観測値が分裂しまくる、って。
0267poem
垢版 |
2022/07/20(水) 17:23:27.02ID:???
>>265
多様体とか難しい学問出さずに説明して欲しい
0268poem
垢版 |
2022/07/20(水) 17:26:38.53ID:???
座標の歪みの話?
0269poem
垢版 |
2022/07/20(水) 17:30:07.32ID:???
関数の歪みは積分
積分は元の関数を係数にした関数

座標の歪み、空間の歪みは、元の座標・空間が変数になった座標・空間
0270poem
垢版 |
2022/07/20(水) 17:31:01.95ID:???
>>269 また打ち間違った
変数←係数
0271poem
垢版 |
2022/07/20(水) 17:32:30.80ID:???
多様体ってこういうこと?
座標・空間の歪みを話したいの?
積分とか最近わかって、難しいのはわからないから初歩でお願い
0272ご冗談でしょう?名無しさん
垢版 |
2022/07/20(水) 17:35:10.45ID:???
>>266
全然違う

多様体とは局所的にn次元ユークリッド空間と同相、つまり連続的に各点が1対1対応する空間。座標が定義されていて、基本的にはこの座標をもとに、もとの空間の性質を解析する
あくまで多様体と呼ばれる確固とした空間が先にあって、そこに座標を設定しているだけ。つまり座標の作り方には任意性がある

そしてこの多様体上に計量と呼ばれる一定のルールを満たす概念を導入することで、多様体に距離の概念を導入できる。重要なのは、あくまで距離は多様体に設定されるもので、座標系としてどんな座標系を導入するかに寄らない
例えば「ものさし」は多様体上に存在するもので、これはどんな座標から見ても長さが変わらない。「ものさし」は単位そのものであるので、これも座標変換で変わることはない

相対性理論は根本的にこの「距離」が導入された多様体に構築された理論であり、ローレンツ変換等は座標変換なので原理的にローレンツ変換による多様体上の理論の矛盾は生じ得ない
もし矛盾が生じたら必ず議論の方に穴がある
0273ご冗談でしょう?名無しさん
垢版 |
2022/07/20(水) 17:35:59.24ID:???
多様体は普通に面白いので地道でもいいから挑戦してみてほしい
今が積分を理解したばかりだと結構大変だろうけど
0275poem
垢版 |
2022/07/20(水) 17:56:57.40ID:???
>>272
このぐらいの噛み砕いた説明も自分には理解できないほど頭がわるいんだけど
相対論ない空間→ローレンツ変換→相対論ある空間
と変換しても物理が変わらないと言いたいの?
それならローレンツ変換で時空が伸び縮みするわけがない。座標変換しても物理が変わらないなら、時空の伸び縮みという物理の違いは起こらない。ローレンツ変換で時空の伸び縮みが起こる時点で座標変換はものさしの変更じゃなく物理変換して変な物理出してる。
だからその多様体というのも物理の観測値を変換するだけじゃなくて、観測値が変わる=時空が伸び縮みしてるという変な物理が出てるんだから、観測値が違う→観測値が分裂・世界が分裂してるが真。多様体が何かわからないけど、多様体が物理を出さなくても多様体の変換は物理を出すから、観測値が分裂するのは座標のものさしが変わるだけで世界は分裂してない、多様体で証明、補償されてる、は座標変換してるから偽。相対論で観測値が分裂してるのは真実は同じ値を言ってる、→偽。多様体が相対論での物理の保存を何も証明できない
0277ご冗談でしょう?名無しさん
垢版 |
2022/07/20(水) 19:07:12.48ID:6XJdBHCu
とりあえず一つずつ指摘していこうか

>相対論ない空間→ローレンツ変換→相対論ある空間と変換しても物理が変わらないと言いたいの?
 まず『相対論ない空間→ローレンツ変換→相対論ある空間』という流れが間違ってる。どちらかと言うと『相対論のない空間→相対論のある空間(導入した計量の下でローレンツ変換が特別な意味を持つ)』、そしてかな。
 『相対論のない空間』つまり多様体とは、距離が定義されず、あくまで空間の各点同士の繋がり方が、ユークリッド空間の繋がり方と同じだよという空間。この定義の下だと、例えばドーナツとティーカップは、我々から見たら明らかに違う形状の物だけど、どちらも各点が二次元(ユークリッド)空間であり、そして全体的な『二次元空間』の繋がり方が同じなので、多様体という『見方』の下では同一の物になる。逆に言えば、『多様体』の基本概念そのものには、我々が感じるような『湾曲』という概念は存在せず、あくまで繋がり方の解像度しかない。しかし、重要なのは、それでもあくまで『多様体』は実体として扱われており、その多様体自身が持つあらゆる『量』は、「どんな座標系で測ろうとも」同じでなくてはならない。多様体は決して曖昧な存在ではないのだ。
 更に、ここで多様体が『計量』と呼ばれる概念を内在しているとする。これは座標におけるパラメータ当たりの『距離』に該当する概念で、直前の文はつまり、各点間に「距離」が存在するような多様体を考えるという事である。これはリーマン多様体と呼ばれる。(相対論では『計量』の条件が少し変わっており、擬リーマン多様体と呼ばれる概念になっている。これは相対論において「距離」の概念が「長さ」ではなく「√(時間^2-長さ^2)」に変わっている事に起因する。相対論登場以前の物理はもちろん普通の距離が導入されたリーマン多様体で、しかも平坦、つまり普通距離が定義された3次元ユークリッド空間そのものである。) 相対性理論を考える事は、『距離』が定義された(擬)リーマン多様体を考える事と同義である。
0278ご冗談でしょう?名無しさん
垢版 |
2022/07/20(水) 19:07:42.20ID:6XJdBHCu
 『距離』が存在すれば『空間の歪み』を捉えることができる。例えば、一つの円が存在して、その円周の長さと直径を比べてみるとしよう。普通ならば、「円周=2π直径」が成立している筈である。ここで、もし円周を計ると2π[cm]だったのに、直径を計ると1.3[cm]だったら? そうなっていれば、そのような空間は『曲がっている』という事になる。(これは正に球面上に住む二次元人の視点で上のような測定ができる。逆に直径が0.5[cm]になる場合は、これは鞍状の面の上の二次元人の話になる)
 ところが、特殊相対性理論の話になると、これは一切曲がっていない空間、つまり『平面』上の相対論の話になる。平面上の理論では、ローレンツ変換と呼ばれる座標変換の特別性が顕著に現れる。それはローレンツ座標変換前とローレンツ座標変換後で、『計量=各座標パラメータ当たりの各方向に対する距離』の形が、全く変わらないという事である。これは、ローレンツ変換前後で見える物理法則の記述が全く変わらないということである。(あくまで記述法=物理的な方程式の形が変わらないというだけだ。方程式の「解」は座標変換の影響を受けて見かけ上変わる。例えば運動方程式はローレンツ変換前後で変わらないが、運動方程式の解、例えばボールの軌道は、座標変換の影響を受けて見かけ上変わる。しかし、見かけが変わって見えるのは、あくまで「座標側」が変わった為であり、多様体(実体として有る空間)側から見れば軌道は全く変わっていない。) 
 以上を纏めると、相対性理論はまず我々の住まう空間が『計量(距離)』を持つ4次元空間であると考え、その様な空間の下では、ローレンツ座標変換が顕著に特別な意味を持つようになるという事である。特に特殊相対性理論においては、ローレンツ変換は物理を全く変更しない。
0279ご冗談でしょう?名無しさん
垢版 |
2022/07/20(水) 19:08:25.81ID:6XJdBHCu
>ローレンツ変換で時空が伸び縮みするわけがない
 以上の流れを踏めばむしろこの疑問は尤もだ。これはこう答えることができる。
「その通り、ローレンツ変換で時空は『伸び縮み』しない。」
 そもそも特殊相対性理論は「平坦な空間」の話なので、ことさら「時空の歪み」という概念は有り得ない。ではなぜ「ローレンツ収縮」という、まるで「距離の歪み」のような現象が起きるのか。それは「距離」に対して誤解があるために生じると言える。
 ローレンツ変換の下で不変となる「距離」は、一般に我々が考えるような「距離」ではないのだ。前述の括弧の中で注釈したように、相対論の中で「距離」に該当する概念は、「√(時間^2-長さ^2)」になっている。つまり、相対論における「物体の『距離』」は、時間方向にも物体が『回転』していることも考慮に入れて考えられている。率直に言えば、ローレンツ変換で物体が「縮んでいる」ように見えるのは、物体が実際に縮んだのではなく、時間方向に物体が「回転」したために縮んだように見えるのである。これは、棒を横から見て、棒を前後方向に回転させれば、棒の長さが変わっているように見える事と(殆ど)完全に対応している。ローレンツ収縮とは、運動している棒の両端の位置を比べた時に、実はその比較が、棒自身にとっては異なる時間での点を比べて図られている事になる。この「時間差」を考慮に入れて初めて「長さ」は不変になるのであり、逆に考慮に入れなければ、「長さ」が変わって見えるのは必然である。
 

とりあえずこんなものかな。ちょっと疲れた。やる事もあるから今日はこの辺にして置く。
0280ご冗談でしょう?名無しさん
垢版 |
2022/07/20(水) 19:12:23.17ID:6XJdBHCu
>>278
ごめん『円周=2π直径』じゃなくて『円周=2π半径』だ。
直径は全部半径に読み替えてくれ。
0281ご冗談でしょう?名無しさん
垢版 |
2022/07/20(水) 19:53:13.75ID:6XJdBHCu
 コラムとして「双子のパラドックス」についてちょっと言及してみようかな。
 双子のパラドックスと言えば、兄が地球に、弟が宇宙旅行に行って、弟が宇宙旅行から地球に帰ってくると、特殊相対論の計算に依れば、兄は弟よりも老化しているという事が起こる。しかし、特殊相対性理論は「相対論」なので、弟側から見て兄は「宇宙旅行に行った」と捉えることも出来る。すると弟の方が兄よりも老いていなければならないという事になるパラドックスである。
 このパラドックスの誤りは二人が完全に「相対的」であるように見えて、実際にはそうでない事から来ている。
 兄弟が「出発」から「帰還」までに過ごす「時間」を考えよう。ここで、相対論における「距離」が「√(時間^2-長さ^2)」である事を思い出そう。本人の主観では、本人が「その場所」から動く事は無いので、「本人」が「長さ」方向に変化する事は有り得ない。すると本人の経過時間を計ることは、本人が時空上で描く軌跡の「距離」を測る事に他ならなくなる。実際、本人にとっては長さ方向の変位が0なので、「距離」=√(時間^2-長さ^2)=√(時間^2=時間、と「距離」そのものが時間になる。つまり、兄弟各々の経過時間は各々が時空上で描いた「線」の「距離」を測ればいい事になる。この事実はとても理に適っている。なぜならば、多様体上に存在する「物理的な時計」の表示は、どのような座標系で見ても変わらない、多様体そのものに存在する『量』である。「出発」から「帰還」までのこの『物理的な目盛り』の差は、座標変換で変わらない量で示される筈であり、「距離」も座標変換で変わらないから、『兄弟各々経過した時間』=『各々が持っている物理的な時計の時間』と「距離」が等号で結ばれる事は全く奇妙なことではない。
 
0282ご冗談でしょう?名無しさん
垢版 |
2022/07/20(水) 19:53:44.82ID:6XJdBHCu
 さて、兄はその場から動かないので、「出発」から「帰還」まで時空上では直線を描く。一方、弟は一度兄から離れてまた戻ってくる軌跡を描く。これは必ず曲線になる。もし直線になるとすれば、それは二つの直線が一度離れて戻ってくるという状況を考える事になり、これは「曲がった空間」でしか有り得ない。特殊相対論下では平坦な空間を考えているので、直線から外れてまた同じ直線と交わるならば、それは曲線にならざるを得ない。これが、兄弟の間の「非相対性」であり、パラドックスの原因となった物である。
 さて、我々の知っている普通の平面上の距離は、二点を結んだ直線と曲線を結ぶと、曲線の方が距離が長い。しかし、相対論で定義される「距離」は二点間を結ぶ直線と曲線を比べた場合、直線の方が「距離」が長くなるという性質を備えている。つまり、普通の距離では直線が最短距離を表していたが、特殊相対論上の「距離」は、直線が最長「距離」を表すという事である。よって兄は「出発」から「帰還」まで直線距離で進むのでその「距離」=経過時間が他の軌跡に比べて最も長い。一方、弟は「出発」から「帰還」まで曲線上を進むので、それに比べて短い「時間」になるという事である。
 以上で双子のパラドックスが実はパラドックスではない事が分かる。

 ちなみに途中で兄が「直線上を走る」と断定した事に疑問を憶えたかもしれないが、実はそのあたりはパラドックスにおいて本質的ではない。
 このパラドックスの本質は「2人が同じ場所から出発してまた出会う時、『相対性』を考えれば経過時間に差が出る事は有り得ない」という主張であり、以上で示したように、特殊相対論下という平坦な時空の中では、二つの軌跡の間に距離差=時間差が「座標の取り方に寄らず」現れるのは明白なので、その点でもパラドックスを解消している事になる。

 今度こそやるべきことやるわ
0283poem
垢版 |
2022/07/20(水) 20:03:56.17ID:???
>>278はわからなかった
>>279はわかった。

で279の根本は
普通の空間(相対性理論のない空間)は距離の単位はm。距離の物理構成もm1個。
物理学的常識の空間(相対性理論のある空間)は距離の単位はm。距離の物理構成は√(時間^2-長さ^2)

前者なら物体は時空上で見掛け変形してはいけない。見掛け変形することが文字通り変形だから。速度によって変形してはいけない。相対性理論のような現象になってはいけない。
後者なら物体は時空上で見掛け変形できる。見掛け変形することは文字通り変形ではなく3次元を超越した次元数での回転だから。速度によって変形していい。相対性理論のような現象が起こる。これがローレンツ変換(多様体は変換できる、多様体だから変換できる…ってこと?)

279の言いたいことだけはわかったし
相対性理論の根本、ローレンツ変換の根本、時空の伸び縮みの根本が、

距離m=距離mでなく、距離m=√(時間^2-長さ^2)
という等式に立脚してるのはわかった。


でも根本的にさ
「√(時間^2-長さ^2)」って単位(物理次元)はmじゃないよね。(m^2-s^2)^(1/2)って単位だよね。

つまり人間の身長の単位が170c(m^2-s^2)^(1/2)の身長ってことだよね?

根本的にこれ物理の仕組みとして空間じゃないじゃん

これが真なら空間(距離)という世界のシステムが空間(距離)単体でなく、空間(距離)が世界のもっと根本のシステムの合成物ってことになる
この時mとsの空間に合成されるそれらの根本のシステムって、空間(距離)や時間じゃなくて何?亜空間?亜時間?

これまで人間が使ってた、身長170cmのmは亜空間(亜距離)だったってことになる。
0284poem
垢版 |
2022/07/20(水) 21:38:50.22ID:???
>>281
>>282
読んで全部は理解できてないけど
・弟から見て兄が宇宙旅行するから、弟が先に歳取る

・兄にとって弟が宇宙旅行してるようにも見えるから、兄が先に歳取る

なんか理論で誤魔化してどちらか一方に帰着させるか両方歳取らないに帰着させて、パラドックス解消したと思い込んで解消できてないのはわかった
0285poem
垢版 |
2022/07/20(水) 21:46:31.28ID:???
話戻してあと一言

赤の定義は赤。赤以外でも青以外でも青でもない。
善の定義は善。善じゃないでも、悪じゃないでも、悪でもない。

空間の定義は空間。亜空間でもない。空間mの定義は√(時間^2−空間^2)?でもない。4次元空間の定義は時間という物理は空間という物理ではないから、時間を入れて4次元空間にはできない。4次元時空とできても空間と時間を同列にして時間を自由に移動できることはない。

単位は物理の機能を表す記号。その定義は物理の機能を表す。√(s^2-m^2)?の時点でこれは空間の機能じゃない。空間とは別物。空間の機能の定義が空間の機能と時間の機能の合成物なんてことはない
0287poem
垢版 |
2022/07/20(水) 22:12:21.45ID:???
ちなみに記号は指示語。これとかあれ。空間機能を指してmと言ってる。
1とか2も、数値1のこと数値2のことという指示語かな
この指示語で指した物が世界のシステムの真実に近いほど、定義は正しい。
0288poem
垢版 |
2022/07/20(水) 22:15:32.05ID:???
だから赤、や、善も、赤や善って言う物を指してこれあれって言ってる。
だから赤や善の定義は赤や善。
指示語は指示して言ってるんだから指示語が定義で真。
その指示語の世界のシステムを研究するのが物理や数学を始めとした色々なもの
0289poem
垢版 |
2022/07/20(水) 22:19:37.37ID:???
だから
これ(空間機能)=(は)これ(空間機能m)
は真
これ(空間機能)=(は)√(あれ^2−これ^2)?(√(時間機能^2−空間機能^2))
は式からして間違い。式からして矛盾してる。

だから相対性理論は間違い。
0290ご冗談でしょう?名無しさん
垢版 |
2022/07/20(水) 23:32:08.09ID:6XJdBHCu
ちらっと見たら自分が一部の表記を省略したせいでかなり変な方向に批判が進んでるみたいなので訂正しておきます

相対論における距離の定義を√[時間^2-長さ^2]と書きましたが、厳密には√[(光速×時間)^2-長さ^2]です。なので単位はしっかり揃ってます。
表記が冗長になる、かつ意味が伝わればいいと思っていたので書きませんでした。紛らわしくてごめんなさい。(あと、相対論ではもう時間の単位[s]が冗長な単位になるので、全部[m]で扱ってしまう=時間と距離を等価なものとみる。ということも表記の優先度を下げる原因でした。詳しくは後述します)
 ちなみにもっと正確に言えば、ルートは要りません。基本的に計量は単位の次元が距離の二乗の物で定義されるので、距離を扱う際は[時間^2-長さ^2]で考える事が殆どです。これを基に考える理由は、時間方向よりも長さ方向の方が長い線分の「距離」を考えると明らかです。この線分は負の「距離」を持つ事になります。時間方向の方が長い場合は正の「距離」になります。相対論においては、多様体に導入されている「距離」の二乗が正負の量を持つ(これが相対論の多様体が「リーマン多様体」でなく「擬リーマン多様体」と呼ばれる理由)ので、基本ルートは付けて考えません。そして、本来これを『「距離」の二乗』と捉えること自体が筋違いです。これだと「距離」が先で「距離の二乗」=「計量」が後に見えます。実際には、「計量」が先に存在し、そこから「距離」がどう定義するかは慎重に考えなくてはいけません。というか、実際には「計量」だけで十分で、そこから「距離」に該当する概念を考えるのは本来不必要です。
(人によっては「計量」を[長さ^2-時間^2]とする人もいます。本質的な違いはありません。個人の好みです。ちなみに自分は本当はこっち派です。)
0291ご冗談でしょう?名無しさん
垢版 |
2022/07/20(水) 23:32:33.54ID:6XJdBHCu
 ちょっと単位の話をします。特に距離の単位について話しましょう。
((擬)リーマン)多様体上における距離の単位とは、もちろん多様体上における一定の「長さ」を表します。『単位を基にして測る』ということは、その一定の「長さ」を基準に、「対象の長さ」との比率を測るということになります。
 相対論においては「距離」の二乗、つまり「計量」は[長さ^2-(光速×時間)^2]とできます。(先刻の定義から改めました)この下で測られる何らかの「ものさし」が単位となります。ここでは、「長さ」方向に延びる「ものさし」を考えましょう。時間方向が0になりますので、純粋に長さ^2という値になります。ここにルートを課したものを「単位」として採用しましょう。これが我々が今まで使ってきた丁度(普通の距離で考えた)1[m]の物差しであれば、単位は[m]そのものになります。
 ここで今一度距離の二乗の定義を見つめ直しましょう。すると気付く事があります。「この「ものさし」、時間方向にも測れるぞ?」
 つまり、時間方向に延びる線分を考えると「距離」の二乗が負の値が出てきます。これを先程定義した単位を用いて[m^2]で割れば、[m^2]を基に測った「距離の二乗」が測れます。これは負の値を持ちます。
 我々が普段用いる時間の単位[s]を基に測ろうと思えば、この負の「距離の二乗」を絶対値化してルートを掛け、更に光速で割れば導けます。
 ところが、ちょっと考えてみましょう。光速は、どんな座標系から見ても一定です。これを利用し、光速を基準に、光が進んだ「長さ」を基に経過した時間を計る、という使い方が出来ます。これは「長さ」が【常に】「時間」と一定の比例関係にあるという言い方が出来ます。ということは、「時間を計る」ということが「距離を計る」ことと等価であり、[s]で測っていた物が、実は[m]だけで「本質的に」測れる物であったという事になります。これは相対論以前で考えられた物理空間では考え得ないことでした。相対論が認められたことで、時間の単位[s]が余剰な物であるということが判明したのです(計算上はc=1として扱われます、つまり、3×10^8[m/s]=1⇒[s]=3.0×10^8[m])。
0292ご冗談でしょう?名無しさん
垢版 |
2022/07/20(水) 23:32:56.90ID:6XJdBHCu
 単位の余剰可能性は別におかしなことではありません。例えば電磁気でよく用いられる単位「クーロン」も、実は余剰な単位です。実際には単位系から消去することができ、[m],[s],[kg]だけで表現できます。相対論においては、此処から更に[s]が消去できる事が明らかになっただけです。
 加えて量子力学の知識から光速と同様にプランク定数も「とびとび」を表す根源的な単位として考えられていますので、さらにもう一つ余剰単位が存在する事となり、光速と同様ℏ=1として、単位が一つ消去され、全ての物理量は質量単位(例えば[kg]とか。素粒子論だと[MeV]とかが殆ど)で表すことができると考えることができます。c=h=1と置く単位系を自然単位系と呼びます。
 少なくとも、相対論の中においては[s]は余剰単位になります。これは「距離」が多様体の保持する座標不変な量であり、更にその「距離」の定義の中に時間と長さが(殆ど)等価な形で同居している点で、奇妙な事ではありません。多様体側から見れば、その二つは量的な違い(正負の違い)であり、質的な違い(単位の違い)ではないのです。二つは同じ単位で測れる、正負の異なる量に過ぎない訳です。
 我々にとって時間と長さが全く別物に見えるのは、時間が負の量、長さが正の量に対応しているからと言えます。また、我々人間が感じられる時間スケール[s]と長さのスケール[m]に大きな開き([s]=3.0×10^8[m])があることも、理由の一つでしょう。考えている物理現象のスケールに[s]>>[m]という差がある事は、相対論が何らかの近似理論に帰着されるという事に他なりません。それが我々が相対論以前に扱ってきた物理であり、時間と空間が完全に分断された理論なのです。
0293ご冗談でしょう?名無しさん
垢版 |
2022/07/20(水) 23:44:33.09ID:6XJdBHCu
>>284
兄弟の間にある本質的な【非相対性】は、考えている多様体に距離が導入されている所から起因します。本来多様体は「湾曲」という概念が存在しない、曲面と平面の区別も付けられない物でした。こういった多様体上で二点を結ぶ二つの軌跡を考えても、両者に本質的な違いは生まれません。
(正確には『交点が存在する』、『空間の穴を周回している』とうの特徴で「相対性」が失われる事が有り得ますが、双子のパラドックスを考える上では本質的ではありません)
 ところ、「計量」則ち「距離」が導入されたことで、平面と曲面に区別が付けられるようになります。もっと言えば、これによって【空間が確固とした形状を持つようになった】という事です。この確固たる形状を持つ多様体上を走る軌跡は、この「多様体の形状」と常に比較されますので、「Aが直線で、Bが曲線」という主張は、正に「多様体の形状」を基に主張される物であり、「Aから見てBは曲線だけど、Bから見てAは曲線だから相対的」という事は出来ません。常に、多様体側から見た形状で判断されなければなりません。故に、「距離」が導入された特殊相対論上の多様体では、双子の軌道が非相対的になるのです。


やるべきこと、やります
0294poem
垢版 |
2022/07/21(木) 10:00:21.83ID:???
赤=((赤/青)^2−青^2)^(1/2)
単位同じになるってほんとに思うの?
0295poem
垢版 |
2022/07/21(木) 10:04:06.17ID:???
赤の定義が赤以外だとでも?
赤の定義は赤という同じ指示語か
赤の定義を分解することはできるが
赤の定義に青が入る事はない

距離の定義に時間が入る事はない
0296poem
垢版 |
2022/07/21(木) 10:05:51.81ID:???
現代物理学者が誤魔化しで生きてるのはわかった。
0297ご冗談でしょう?名無しさん
垢版 |
2022/07/21(木) 10:10:36.54ID:???
>>294
>>290の最初の数行で明言したように、より正しい定義は √[(光速×時間)^2-長さ^2]なので、√(([m/s]×[s])^2-[m]^2)=[m]ときちんと単位は揃ってますよ?
0298poem
垢版 |
2022/07/21(木) 10:14:08.94ID:???
多分ここの板の現代物理学者はこの人の説明を正解だ物理はそうなってると思うだろう
0299poem
垢版 |
2022/07/21(木) 10:19:05.10ID:???
√(([m/s]×[s])^2-[m]^2)=[m]
とすればね
上の読んで中の[s]なかったじゃん。今入れただけじゃん
それとその前は
√([s]^2-([m/s]×[s])^2)=[m]
[s]入ってなかったし、単位違うし
0301poem
垢版 |
2022/07/21(木) 10:21:25.20ID:???
そもそも
√([s]^2-[m]^2)=[m]
じゃないの?
0302poem
垢版 |
2022/07/21(木) 10:24:31.57ID:???
しかも
√([s]2-[m]^2)=[m]
がどうやったら
√(([m/s]×[s])^2-[m]^2)=[m]
になるんだ
右の-[m]^2はそのままだが
左の[s]^2から([m/s]×[s]^2)になるの[m/s]を挿入してるけど、右側に入れないのか
0303ご冗談でしょう?名無しさん
垢版 |
2022/07/21(木) 10:26:52.49ID:???
>>301
いえ、それは違いますよ
自分が以前に「√時間^2-長さ^2」と書いていたのはあくまで「意味合い」として話で、定数である光速は省略して書いただけです。単位でみると間違った式であるというのは正しくて、実際には「√(光速×時間)^2-長さ^2」が正しいし、実際の相対論はこっちの形で議論します
0304poem
垢版 |
2022/07/21(木) 10:30:24.52ID:???
>>299早とちり
なかったのはm/s
0306poem
垢版 |
2022/07/21(木) 10:37:34.52ID:???
文はね

でも√(([m/s]×[s])^2-[m]^2)=[m]が意味合いが√([s]^2-[m]^2)=[m]になるなら、これは間違ってる。もしこういう意味になるなら定義が自己矛盾してるから
0307ご冗談でしょう?名無しさん
垢版 |
2022/07/21(木) 10:41:46.33ID:???
>>306
定数を省略するのが嫌なら別に上述全ての議論を「√(光速×時間)^2-長さ^2」として捉えて貰えれば良いですよ、それでもちゃんと説明が通りますので
光速を省略して「意味自体はある程度通る」と思ったのは自分の落ち度であって、元々厳密な定義でしか議論してない相対論側には落ち度の無い話です
0309poem
垢版 |
2022/07/21(木) 16:20:49.28ID:???
というか難しい学術わからないから
√(([m/s]×[s])^2-[m]^2)=[m]で単位揃っちゃうと、距離の定義は距離だ=単位が揃わなきゃ間違いだ、って反論できないんだよね。
この面からは反論できないから、正確にこの式で単位揃うならこちらの負け。難しい学術わかるくらい頭良ければ単位揃ってても、[m]=[m]のものを√(([m/s]×[s])^2-[m]^2)=[m]と無駄な冗長して複雑化してはいけない、またはこの複雑化が式によっては間違いになるケースがわかって場合によっては勝てるだろうけど、簡単な観点では負けしかない
0310poem
垢版 |
2022/07/21(木) 16:22:14.29ID:???
↑訂正
正確にこの式(が正解で)で単位揃うならこちらの負け。
0311ご冗談でしょう?名無しさん
垢版 |
2022/07/21(木) 20:55:18.15ID:???
>>309
まあ地道に今の勉強を積み重ねて将来の貴方に納得して貰うしか道は無いでしょうね
特殊相対論を学ぶだけでも線形代数、微分積分学、解析力学は必須ですし、余力があるならここに電磁気学も加わってほしい
さらに多様体的に考えを深めるなら、集合論、位相空間論が要ります。特に位相空間論は最難関でしょうね...定義だけ見ても意味不明でしょうから、連続とは何か、解析学(解析力学とは全く別の学問、微分積分学に近い)を元にじっくり理解を深めていくしかないでしょう

ちなみに『[m]=[m]のものを√(([m/s]×[s])^2-[m]^2)=[m]と無駄な冗長して複雑化』と書いてありますが、むしろ逆で、我々が相対論以前では[s]と[m]は全く独立な単位として扱われていましたが、相対論の登場で時間と空間の取り扱いが統一化されて、時間も空間も[m]の単位のみで考えれば良くなりました。時間と空間の統一性が『√(([m/s]×[s])^2-[m]^2)』という距離の定義式に現れていて、更に『√(([m/s]×[s])^2-[m]^2)=[m]』となる事が、[s]という単位の不要性を示唆していると言えます。つまり、『√(([m/s]×[s])^2-[m]^2)=[m]』は[m]が【複雑化】したのではなく、[s]という概念が[m]に吸収される過程を表す式と言えて、[s],[m]という単位系を【単純化】したと言えるのです。
0312ご冗談でしょう?名無しさん
垢版 |
2022/07/21(木) 20:56:56.80ID:???
>>311
訂正します
[s],[m]という単位系を【単純化】したと言えるのです。
⇒[s],[m]という単位系を[m]に【単純化】したと言えるのです。
0313poem
垢版 |
2022/07/21(木) 21:40:37.14ID:???
[m]は関数システム。乗除比の数学システム
[s]は数列システム。加減差の数学システム

従う数学システムの違いが[m]や[s]や[kg]という物理量。物理量とは従う計算システム。物理のシステム。

[s]を[m]に吸収はできない。[m]は相対システム。[s]は絶対システム。
0315poem
垢版 |
2022/07/21(木) 21:48:02.58ID:???
相対論は

>ちなみに『[m]=[m]のものを√(([m/s]×[s])^2-[m]^2)=[m]と無駄な冗長して複雑化』と書いてありますが、むしろ逆で、我々が相対論以前では[s]と[m]は全く独立な単位として扱われていましたが、相対論の登場で時間と空間の取り扱いが統一化されて、時間も空間も[m]の単位のみで考えれば良くなりました。時間と空間の統一性が『√(([m/s]×[s])^2-[m]^2)』という距離の定義式に現れていて、更に『√(([m/s]×[s])^2-[m]^2)=[m]』となる事が、[s]という単位の不要性を示唆していると言えます。つまり、『√(([m/s]×[s])^2-[m]^2)=[m]』は[m]が【複雑化】したのではなく、[s]という概念が[m]に吸収される過程を表す式と言えて、[s],[m]という単位系を[m]に【単純化】したと言えるのです。

という原理だから
この原理であることから相対論は誤り
0316ご冗談でしょう?名無しさん
垢版 |
2022/07/21(木) 22:08:24.60ID:???
>>313
スレ見ても要点が全く分からんから、今このスレ議論してる事に沿ってもう一度主張したい内容をフォーマットしてくれ

ちなみに因数分解は厳密に行いたければ単に『環 』を勉強しさえすればいい

>>315
どういう原理? 理由を明記しないと何も分からん。コミュニケーションになってないよ
0317poem
垢版 |
2022/07/21(木) 22:49:51.51ID:???
>>316
因数分解理解したいから助言はベリーテンクスなんだけど、「環」ってWikipediaみて全く理解できなかった
0318ご冗談でしょう?名無しさん
垢版 |
2022/07/21(木) 23:02:17.94ID:eXmlxNFw
あと君は「定義」という言葉を大いに勘違いしているように見える。

「定義」とは「概念の明確化・線引き」である。ある「概念」を「定義」に従って考える時、「概念」をこの「定義」から外れて考えてはいけない。「概念」はその「定義」その物であり、それ以外の何物でもないのだ。

【微分は関数の接線の傾き
積分は関数の描く面積

…これ間違い。
定義はもっと簡単。単純。

微分は関数の係数
積分は関数を係数にする】

この文章は完全に「定義」の意味を捉え間違えている。これは「定義」ではなく「解釈」だ。貴方が「定義」を読み、そこから「意味」を読み取ろうとし、「解釈」した結果に過ぎない。『定義はもっと簡単』という言葉は、『定義』とは何かを知っていれば、絶対に生まれ得ない言葉である。


 微分の定義は簡単に書いて『lim[h→0](f(x+h)-f(x))/h』だ。本来はもっと厳密な表現があるが、要は数式の操作として「定義」されていることが貴方に伝わればいい。このように定義された数式的操作には一切の曇りが無く、「解釈の余地」を挟まない。ここで言う「解釈の余地」とは、「解釈」によって結果が変わるような可能性という意味である。(面積の定義は省略しよう。微分に比べて簡潔な物では無い)
 この「定義」に対して、「意味的解釈」をどのように捉えるかは、人によって変わり得る。だが、注意せねばならないのは、この「意味的解釈」は概ね元の「定義」に還元できることが望ましいのである。
0319ご冗談でしょう?名無しさん
垢版 |
2022/07/21(木) 23:02:37.23ID:eXmlxNFw
『微分は関数の接線の傾き、積分は関数の描く面積』というのは、微分・積分各々の「定義」ではなく、「定義」を基に「意味的解釈」を施した結果だ。だが両者ともかなり元の「定義」を尊重できている。というのは、この「意味的解釈」から元の「定義」がかなり復元可能だからだ。
 一方、『微分は関数の係数、積分は関数を係数にする』という言葉は、かなり定義への復元可能性が欠けているように見える。『関数の係数』の「定義」が不明瞭だからだ。

 長々と話したが、此処で言いたいのは「定義」と「解釈」は違うものだという事。相対論や微分積分学は、「数式」を定義に含めており、決して曖昧模糊な言葉ではない。各々の理論は、そのように明確化された「定義」に従ってしか構築されていないので、矛盾を持たない理論に成長したのだと言える。(ちなみに、現在の基本的な数学的な議論は、ZFC公理系と呼ばれる論理記号を基にした公理的集合論で記述できるので、微分や積分をわざわざ生身で記号論として構築し直そうとしなくても、既にその仕事は過去の数学者によって終わっていると言える。)
 貴方にはその当たりを注意して「定義」がしっかりした、あるいはそれを踏まえた文章を書いてほしいですね。
0320poem
垢版 |
2022/07/21(木) 23:05:05.90ID:???
>>316
あのスレの要点は「わかったことがある」で、この相対論の問答にURL貼って引っ張り込でツッコミの武器にしたかった要点は「時間という物理機能(物理)(物理量…複合単位でなく単体だから物理単位か)の正体」
世界の真システムが根源にあって、数学が枝葉について、物理がさらに枝葉につく。物理のより根源が数学で物理は数学の枝葉だから物理は数学を言語とし表現できる。物理学はやがて数学に到り、数学は世界の真システムに到る。
物理の物理機能、時間や空間の物理、それの働きを記述する指示語の時間機能空間機能を表す時間空間の単位。

空間とは関数システム
時間とは数列システム

単位とは、従う数学の機能が異なる物が異なる数学機能から異なる物理機能を持ち、時間と空間は物理機能、数学が異なり、時間や空間の世界での発生がその従う数学に解明のヒントがあるはず。

そのあのスレでの「わかったことがある」の原理の元、相対論のあなたが言う原理、315はこれに反するから誤りとpoemが思うこと。
0321poem
垢版 |
2022/07/21(木) 23:15:33.64ID:???
微分は関数の係数
積分は関数を係数にする】

このぐらいは理解して欲しかったな

y=ax(指数や多項略)の基本形のaが係数。
微分はaのこと。
積分はy=axをaとしたy=ax。

だから
微分は関数の係数
積分は関数を係数にする】

微分は傾き
積分は面積
と言えるけど
もっと根源的な定義が抽出と挿入。a…つまり変数に掛かる係数を抽出が微分、aに…つまり変数に掛かる係数を挿入が積分

このくらいは理解できて
0322ご冗談でしょう?名無しさん
垢版 |
2022/07/21(木) 23:17:37.80ID:???
>>317

あー数学をwikipediaで理解するのは止めた方がいい。どんなに才能があっても、初学者なら99.9%無理だと思う。

検索結果にwikipediaに続いて出てくる奴読んだ方がいいよ
https://manabitimes.jp/math/1744
https://mathlandscape.com/ring/
https://math-note.xyz/algebra/basics-of-algebra/

因数分解の一意性を知りたいなら多項式環まで行く必要がある。
その過程で環論を拡張した体論とか、ユークリッド互除法等の整数論の知識とかも要るかも
ちょっと昔過ぎて忘れちゃったけど、この辺が因数分解という物を厳密に捉えようとするために必要な道具かな


あと次の返信、もしかしたら近日中にできんかも
0323poem
垢版 |
2022/07/21(木) 23:18:24.35ID:???
あのスレの微分積分の話は本題ではない。相対論に関係ない。
0324poem
垢版 |
2022/07/21(木) 23:20:54.70ID:???
>>322
>あと次の返信、もしかしたら近日中にできんかも

りょです


↑の定義の話、自分の「定義」のコンセプトは
「真理」でない間違った「定義」は間違った「定義」
0325poem
垢版 |
2022/07/21(木) 23:23:45.30ID:???
貴方のような人は
定義:1+1=3
としても従うんだろう。
先人がそう出したら従うんだろう

先人の定義を究明しようとはしない「定義が間違ってるかを究明しようとはしない」「定義が真理から乖離してるかどうかは考えない」のだろう
0327poem
垢版 |
2022/07/21(木) 23:26:32.30ID:???
「真理」を使って学術を行え
「定義」が「真理」から乖離してるかどうかを考え、乖離してるならそれを扱って真理を学術することは難しい
0328poem
垢版 |
2022/07/21(木) 23:28:58.29ID:???
>>326
間違った定義を真理だと思うからいけない
0329poem
垢版 |
2022/07/21(木) 23:35:13.47ID:???
まあ、あのスレ読みにくくて理解しにくくて読めないよね。読めない可能性もわかってる。

相対論の原理、それが真理か。
相対論のコンセプトの定義が、真理か。
相対論が正しいかはそこから立って考えなければわからない。
自分はそこから、そこに立って、そこからの理屈で色々言ってる。
相対論を弄くり回しても相対論が正しいかは出ない。
他の理論を弄くり回しても出ない。
根源を弄くり回さなければ。
これが定義を弄くり回すこと。
定義が真理から乖離してるかどうかを考えること
0330ご冗談でしょう?名無しさん
垢版 |
2022/07/21(木) 23:45:10.04ID:???
>>325
とりあえずこれだけに返信する

『1+1=3』は定義にはならない。なぜなら1も3も+も=も既に別の定義が与えられてるから、(詳しくはペアノの公理を検索)、1+1=3は定義ではなく関係式である。そして矛盾しているので不成立になる。
もし本当に1+1=3を根本的な定義にしようとするなら、(もし+と=と1が既に定義されていれば)3は『3』という文字の形を取ってるだけの『2』or『0(排他的論理和)』だろうね

ちなみに定義は何でもかんでも受け入れられてる訳じゃなくて、自己矛盾した場合は『無意味な定義でしたね、残念でした』となって終わり。

「真理」という存在が不明確なものどうやって使うの? どうやって正否を判断するの? その正否の判断の正否はどうやって判断するの? その『真理』の『真理』性を証明できなければ『真理』は使えない。証明できなければ、それは『真理』とは呼べない。『真理』の実体性を証明できなければ、『乖離性』にも言及できない。
 いきなり『的の中心』を射抜けるなんて魔法使いか何かだろうか? それよりは実験を元にして、定義を立脚し、少しずつ現実に起こっている現象の記述の解像度を上げていった方が、その『的の中心』を射抜く勝算が有ると自分は思う。(そのお陰で現在の技術世界があるとも言える)
0331poem
垢版 |
2022/07/22(金) 01:13:24.63ID:???
>>330
「真理」は指示語
「世界の真システム」本当に真な物を指す指示語
これ、あれ。
「定義」が「真理」じゃなければ
>自己矛盾した場合は『無意味な定義でしたね、残念でした』となって終わり。
0332poem
垢版 |
2022/07/22(金) 01:14:26.93ID:???
>>331
「世界の真システム」も本当に真な物を指す、ラベル
0333poem
垢版 |
2022/07/22(金) 01:29:36.87ID:???
真理を証明しろとか、真理が正しいかの正否とか言われても
真理は真を指す指示語で、正しいかの正否で正しくないと証明されれば真理じゃない。真理の定義は真理。真理以外を真理と言わないから、真理が真理でないことの証明・真理が真理と言うのが間違っていたって反証とか、真理という指示語が真理という指示語でなかったことの証明とか、成り立つと思う?じゃあ真理の指示語を真理以外の何にすればいいの?

真理が真理かどうか定量的定性的定意的定論的に示せとか
1が1であることを定量的定性的定意的定論的に示せとか
例え示せたとして真理が真理、1が1なことはなにを指示語で指示するかの時点で定まってる。指示語で指示するそれら以外をそれらと言わない。定量的定性的定意的定論的に示せたからそれらなんだ、じゃない。それらを指してるからそれらで定量的定性的定意的定論的に示せるだけ。それら以外をそれらと言わない。

真理の定義を言えとか証明しろとかなんちゅうセンスでしかない
0338ご冗談でしょう?名無しさん
垢版 |
2022/07/24(日) 19:15:11.81ID:???
>>337
あれ、自ら始めた Right-angle lever paradox の話、結局どう解決するかまったく説明しないんですか?
明らかにわかってなさそうだから期待せん方がいいね。
聞かれても困る Right-angle lever paradox の話、なんで自分から始めちゃったんだろうね?
0340ご冗談でしょう?名無しさん
垢版 |
2022/07/24(日) 19:32:48.44ID:???
>>339
過去スレで散々してた話で、何も話す気すらないのに、なぜ今日わざわざこの話題を自分で挙げたの?
自分で話題に挙げるのは何か話す気があってのことでしょう?
0342ご冗談でしょう?名無しさん
垢版 |
2022/07/24(日) 19:44:26.82ID:???
>>341
書いてたね。登記を馬鹿にするためだっけか?
自分でも答えられないような話題でようやるわ。
きみは後先考えられないでやらかしちゃう奴ってことで了解でーす。
0343ご冗談でしょう?名無しさん
垢版 |
2022/07/24(日) 20:15:13.86ID:???
>>342
はあ?

> 自分でも答えられないような話題でようやるわ。
これは何よ? 登記並みの馬鹿のお前じゃあるまいし。
0344ご冗談でしょう?名無しさん
垢版 |
2022/07/24(日) 20:49:04.79ID:???
>>343
いくら強がってみたところで、自分で始めた Right-angle lever paradox の解決法について、いまだまったく話始める気配さえない事実。
なんで後先考えずにそんな話しようと思ったんや?
困ったときに馬鹿馬鹿いって誤魔化すだけなら登記でもできるで。
0345ご冗談でしょう?名無しさん
垢版 |
2022/07/24(日) 20:50:43.97ID:???
>>343
こいつはアスペ症だから物理記事の検索コピペで粘着貶しはできるが
自分の考えで他人に説明することができない。

たぶん実生活もコミュ障害で使いものにならないから無職
0348ご冗談でしょう?名無しさん
垢版 |
2022/07/24(日) 21:16:30.58ID:???
>>346
おまえが言い出した話じゃないならわざわざ聞かんわな。
自分で始めた話題で何一つ話す気がないっておかしいでしょう。
なんで自分でわかりもせんことで調子に乗ってマウント取ろうと思ったんや?
0349ご冗談でしょう?名無しさん
垢版 |
2022/07/24(日) 21:28:41.24ID:???
普通の人間は〜パラドックスを話題にしたいならまづ自分で簡単な説明をする

>>346 はアスペ症が悪化してそれすら出来ない粘着荒らしだけのスライム
0350ご冗談でしょう?名無しさん
垢版 |
2022/07/24(日) 21:29:42.29ID:???
>>347
登記は無職だよな

>>348
登記の過去の馬鹿晒しを3つ挙げたら、登記は lever paradox にだけムキになって喰らいついた。

> なんで自分でわかりもせんことで調子に乗ってマウント取ろうと思ったんや?
リンク先の解説も理解できない登記並みの馬鹿は絡むなっつうの、ウザいからよ
0352ご冗談でしょう?名無しさん
垢版 |
2022/07/24(日) 21:39:21.57ID:???
>>350
粘着スライムが反応したか

俺は現役引退して暇だからな、直角レバーは電磁気学で解いたからレスしただけ
トルートン・ノーブル実験も同時に解けて一石二鳥だ。
0353ご冗談でしょう?名無しさん
垢版 |
2022/07/24(日) 21:46:15.11ID:???
>>352
登記は無職の爺!

> 直角レバーは電磁気学で解いたからレスしただけ
> トルートン・ノーブル実験も同時に解けて一石二鳥だ。
ローレンツ変換だけで解ける!
なんてほざいてて?
0354ご冗談でしょう?名無しさん
垢版 |
2022/07/24(日) 21:47:11.55ID:???
粘着アスペには理解不能だろが、
電場・磁場のローレンツ変換は4元ベクトルの変換と同じ結果になるのだよ
0355ご冗談でしょう?名無しさん
垢版 |
2022/07/24(日) 21:56:07.58ID:???
>>350
4年も前の思い出話 right-angle lever paradox をいきなり出して登記を挑発してんのはおまえでしょう。

リンク先にwikiには簡単にパラドックは解決できる旨は書いてあるがざっくりしたヒントだけだ。
おまえとしてはそれ以上の補足がなにもない状態。
そんな状態でよく4年も前の思い出話を何回もしてるわな。
毎度ウザイ思い出話でいちいち登記に絡んでゴミを掘り起こすなよ。
0356ご冗談でしょう?名無しさん
垢版 |
2022/07/24(日) 22:05:05.41ID:???
>>354
E、Bのローレンツ変換なんか出来て当たり前
力のローレンツ変換出来なくて、パラドックスの入口にすら辿り着けない馬鹿がお前だ、登記

>>355
> リンク先にwikiには簡単にパラドックは解決できる旨は書いてあるがざっくりしたヒントだけだ。
あのヒントで解けんのか、この馬鹿は。
Laue current 計算するだけだがな。電磁気学のパラドックス解くのにもよく出てくるから覚えておけや低能
0357ご冗談でしょう?名無しさん
垢版 |
2022/07/24(日) 22:15:06.02ID:???
>>356
そんなもやっとしたことなら誰でも言えるわな。
イキって馬鹿馬鹿だけ言いまくるだけで、実のあることが何一つ言えてないのは滑稽やで。
0358ご冗談でしょう?名無しさん
垢版 |
2022/07/24(日) 22:24:39.79ID:???
>>357
ホントうぜえな。
お前は、解説読んで理解できなかった登記並みの馬鹿確定。
計算式は過去ログにURL貼ってるから探せ。
0359ご冗談でしょう?名無しさん
垢版 |
2022/07/24(日) 22:27:08.12ID:???
>>356
アスペが発狂したか

電磁場のローレンツ変換で計算するのは電荷に働くローレンツ力だ、アホ

所詮、粘着アスペはネット検索コピペしてるだけで自分では何も考えられない
アホ確定したから おしまい。
0361ご冗談でしょう?名無しさん
垢版 |
2022/07/24(日) 22:36:54.86ID:???
>>358
>URL貼ってる

オマエが計算式のコピペも書かない理由は、式の意味を聞かれても自分で答えれない
馬鹿だからだ、ミエミエ
0362ご冗談でしょう?名無しさん
垢版 |
2022/07/24(日) 22:37:32.62ID:???
>>358
おまえが理解できとらんのだろう
だから具体的な話は一切できない
話ができないのに無意味な煽りレスだけを続けているおまえに意味はない
0377ご冗談でしょう?名無しさん
垢版 |
2022/07/25(月) 21:33:02.79ID:???
解説記事読んでも分からん馬鹿が登記の他に2名居た。
そういう奴等はしつこく解説求める。数年前から変わらん。
0379ご冗談でしょう?名無しさん
垢版 |
2022/07/26(火) 00:33:14.33ID:???
The views of space and time which I wish to lay before you have sprung from the soil of experimental physics, and therein lies their strength. They are radical. Henceforth space by itself, and time by itself, are doomed to fade away into mere shadows, and only a kind of union of the two will preserve an independent reality. (Hermann Minkowski (1908))
0381ご冗談でしょう?名無しさん
垢版 |
2022/07/26(火) 00:39:57.67ID:UU7p2tyV
空間も時間もつまらない。いっそ、一緒にしちまった方がおもろくなるよ
0383ご冗談でしょう?名無しさん
垢版 |
2022/07/26(火) 01:16:06.95ID:???
>>382
総ツッコミってお前ら馬鹿2人じゃん。
登記の馬鹿解答を一緒に嘲り笑ってればいいのに、同レベルの馬鹿だから登記側に付いちゃった。
0388ご冗談でしょう?名無しさん
垢版 |
2022/07/26(火) 14:34:20.07ID:xprjiGBH
e=mc^2 を導き出し方を教えてください。
0389ご冗談でしょう?名無しさん
垢版 |
2022/07/26(火) 14:38:07.79ID:UU7p2tyV
次元解析
0390ご冗談でしょう?名無しさん
垢版 |
2022/07/26(火) 14:44:03.28ID:???
んな雑な議論求めてないだろ
一粒子の作用は∫dsで
ds=[√(1-v^2)]dt
あとはハミルトニアン出してもらってv=0の条件課せばmc^2が出てくる
0392ご冗談でしょう?名無しさん
垢版 |
2022/07/26(火) 14:56:51.60ID:???
ごめんもっと正確にはmc∫dsです...
c→∞の極限でラグラジアンが1/2 mv^2に漸近しなきゃならないので(vの0次オーダーmcはc→∞で無限に飛ぶように見えるけど定数項なので実質運動方程式に影響を与えない)
0393ご冗談でしょう?名無しさん
垢版 |
2022/07/26(火) 16:03:12.05ID:UU7p2tyV
百年早そうね
0394ご冗談でしょう?名無しさん
垢版 |
2022/07/26(火) 16:16:43.71ID:6Wy9ggvL
とりあえずひと段落、また近い内に返信できなくなる。

すっと読んで思った事

相対論否定論者には主に二種類いると思ってて
@相対論は理論の内部に矛盾を含んでおり、理論そのものが誤りである
A理論の内部に矛盾は無いが、それが現実世界の理論として適用されるのは誤りである

poem君はどっちだろうか?

更にAだった場合、「誤り」とする基準にも違いがあって、以下の二つに分けられると思う

Ⓐ近似的には正しく現実を再現するだろうが、より深い理論が存在して相対論はその近似理論に過ぎない(つまり相対論は正確な理論ではない)
Ⓑ近似的にも誤りである。現実は相対論的では絶対にありえない。

poem君はどっちだろうか?

ちなみに自分はAⒶ派で、というか殆どの科学者はそんな感じだと思う。
Aとする理由は、そもそも相対論は古典論(非量子論)なのでそっくりそのまま現実世界には適用できないから。実験事実として相対論が生身で正しいことは絶対にありえない。他の理論(量子論)との結合に成功して初めて、完全に正しいと言えるレベルになると思う。だが今現在、重力理論(一般相対性理論)において現在、かなり難航中である。(紫外発散が解消できない) ひも理論なり修正重力理論なりみんな頑張ってる現状である。
Ⓐとした理由は、そもそもとして近似的には正しいことが実験的に証明されているから。少なくとも、それよりハイレベルな理論が存在したとしても、我々のエネルギースケールに極限を取った時に相対論に帰着されなければおかしいレベルである。

自分の立場はこんな感じ
0395ご冗談でしょう?名無しさん
垢版 |
2022/07/26(火) 16:18:00.73ID:6Wy9ggvL
あと真理について

 我々の現実世界において何らかの「真の理論(真理)」が存在したとする。宜しい、「真の理論(真理)」の存在は認めるとしよう。(究極的に科学者はそれを期待して研究していると言えるし)
 だが存在したとして、その「『存在』という仮定」からどれ程の情報を抜き取れるだろうか? 直感的には、抜き取れたとしてもそれはそう多くはないはずだし、たいして具体性も持てない筈だ。もし、非自明に具体的な定理が得られたら、それは論文にすべきだ。(有限単純群の分類定理然り、限られた「有限群」という条件からモンスター群なる不可解な位数を持つ群が、群の中でも特別な物として現れるのは驚くべきことだ)
 poem君の標榜する「システム」やらの言葉が飛び交う理論は、純粋に「『存在』という仮定」から導出されたとは到底思えない。多くの飛躍が積み重なっていると第三者から思われるのは至極当然のことで、もし確固たる正しい理論だと思うならば、『存在』という仮定」から丁寧に導けることを示す必要があるし、確固たるとまでは言わなくても、凡そ妥当な線を行っていると思う理論ならば、どこの部分が、「自分が妥当だと思う主張」で、何処が「『存在』という仮定」から導かれる主張なのかをしっかり整理して公表しなければならない。そうしなければ人はまともに相手してくれないだろうし、衝突は絶えないだろう
 自分の考えとしては、「『存在』という仮定」から具体的な、『強い』定理が導出できるとは考えていない(証明したのではなく、現実的でないと考えている)。よしんば可能だとしても、頭の片隅で考えて思い付けるような楽な物では無いと思っている。そして、辛うじて何かその断片的な定理が得られたとしても、その断片だけでは我々の宇宙に適用するにはまだ甘いと思っている。というのは、我々の宇宙が、「その定理がそのまま適応できるような条件を満たす場所」に存在するかどうかすら怪しいからである。
0396ご冗談でしょう?名無しさん
垢版 |
2022/07/26(火) 16:18:32.09ID:6Wy9ggvL
 つまり、「真の理論」が直接住まう「地盤」があったとして、その上に立つ家は、「真の理論」に忠実であろう。しかし、その家の上に更に家を建てた場合、その家は「真の理論」ではなく、「「真の理論」に建てられた家」を基礎に置いている。基礎が異なるから異なる構造の家ができる。そしてその上に建つ家もまた、異なる構造を持つようになるであろう。我々の宇宙が、「地盤」から遥か遠い場所に存在する「存在」である可能性は大いにあり得る。単純で限られた素粒子の集まりに過ぎない筈の物体が、この地球に住まう多種多様、多彩な振舞いを持つ「生命」としての「多様性」すら持てるように、単純な法則の積み重ねが巡り巡って思わぬ彩りを生むことは有り得る。我々の住まう宇宙がその「具体的な彩り」の一つでしかない保証はどこにもない。故に、何らかの「地盤」に対する『強い』定理が存在したとして、それが我々の「見える世界」に存在するかどうかは分からない。我々は結局限られたスケールの「物理」しか調べられない卑小な存在である。
 長々と書いたが、つまるところ、多くの科学者が「真理」を基に物事を考察しないのは、そういった理由も関係していると思う。余りにも抽象的過ぎる「存在の仮定」から我々の宇宙を導出するよりも、まずは実験を積み重ねて今目の前にある現象を説明できるようにした方が、まだ「現実」に対する情報は集まるという物だ。その情報は、「真理」を考察するにあたって必要な条件(ヒント)を与えてくれるだろう。その積み重ねの中で「真理」の断片らしきものが発見されれば万々歳である。
0397ご冗談でしょう?名無しさん
垢版 |
2022/07/26(火) 16:18:34.31ID:???
>>391
全然違うよw
自由粒子の作用Sは、ローレンツ不変なスカラーだから
ローレンツ不変な定数αと世界間隔sを使って
S = α ∫ds
と書けて、c→∞の極限での接続条件から
α = mc
と求まる。
0404ご冗談でしょう?名無しさん
垢版 |
2022/08/03(水) 11:47:08.64ID:???
未だに納得出来ずにもやもや中のお馬鹿ちゃんなのですがエロい人が居たら教えてください

真空状態で、鉛の球と発泡スチロール製の球を同時に落とせば同時に落ちる。これは実験結果からも証明されているわけですが、引力と重力の関係は地球と月の重力差のように質量差によって大きく違いますよね?

であれば、質量の高いオスミウム製の球体とマイクロ格子状の非常に軽い球体を真空状態で落下させた場合、地球の重力に加えて多少なりとも差が加味されてもいいんじゃないかと思うのですが実際の所はどうなんでしょう?
全く変わらない場合はその理由が知りたいです。

また、質量と引力の関係を誰の目にも見える形で観測するにはどれぐらいの質量の物体があればわかるのでしょうか?
0408ご冗談でしょう?名無しさん
垢版 |
2022/08/05(金) 01:45:01.97ID:+nHoXDyy
ある意味、デカルトは時代の先をいってたんだな
ニュートン力学を否定した相対性理論に近いやん

渦動説 - Wikipedia
渦動説とは、ルネ・デカルト(1596 - 1650)が提唱した、天体などの運動の原理を説明するための学説。
後にニュートン力学が現れ、17世紀から18世紀にかけてデカルト派とニュートン派に分かれて大論争に発展した。
渦動説は、デカルトの形而上学と自然哲学に関する教科書的な書『哲学原理』(1644年)に記述され、人々に知られるところとなった。

1633年ころの『世界論』の草稿においては、物体とは独立した空間を認めて「運動というのは、空間の中の、ある位置から別の位置への移動」と見なしていたが、
その後デカルトは考え方を変えて真空という概念は認めなくなり、世界は延長(=おおむね現在で言うところの物質)で満たされているとした。

デカルトの渦動説は、天体を運動させているのは天体を囲んでいる物質(流体、エーテル)が天体を押しているからだとし、その物質は渦のように動いているとする。
また、物体の落下については、水の渦の中に木片を置くとそれが渦の中心に引き込まれるが、言わばそれと同じ原理で、起きているエーテルの渦によって引き込まれていると説明した。

渦動説の評価
渦動説は中世ヨーロッパに大きな影響力をもっていたアリストテレスの自然学をそのまま受け継いだものであり、真空の否定もアリストテレスの考えからきている。
その意味で当時の正統理論であった。
デカルト派は、遠隔作用説をとるニュートンの万有引力をスコラ学での「隠された性質(occult quality)」への逆行とみなし、「オカルトフォース」と呼んで批判した。

ニュートン力学が受容されてゆく過程で、『プリンキピア』の説と対立すると見なされた渦動説は、荒唐無稽の説のように書きたてられ、現在でもしばしば初学者などはそのようなイメージを持ってしまっている。
20世紀になりアインシュタインによって一般相対性理論が提唱されたが、これはある意味で近接作用論が復興したような面も備えている。
0410ご冗談でしょう?名無しさん
垢版 |
2022/08/05(金) 10:26:57.70ID:G7fttJCt
そうとも言える
0412ご冗談でしょう?名無しさん
垢版 |
2022/08/05(金) 15:56:34.80ID:???
>>408
デカルトの様な哲学者・物理学者は宇宙に遠隔作用など存在しないと信じており
無神論に近い強い信念と共通する

現代でも、世界の人口の90%以上は神仏宗教(遠隔作用の一種)を信じている
殆どの人は神にすがるしかないほど弱いのだ(そして騙される、騙されたい)
0414ご冗談でしょう?名無しさん
垢版 |
2022/08/05(金) 19:59:12.88ID:G7fttJCt
そうとも言えるね
0418ご冗談でしょう?名無しさん
垢版 |
2022/08/06(土) 17:14:32.84ID:???
ニュートンの運動方程式習ったときはバネとか万有引力とか具体例を使って方程式解くし
量子力学習ったときも調和振動子とか井戸型ポテンシャルとか具体例使って解いて
理論の理解を深めていくのに
なんで特殊相対論習っても具体的なポテンシャルを当てはめて演習しないの?

電磁気学で勉強済みだから繰り返さなくてもいいよね?ってスタンス?

量子力学は具体例によってニュートン力学の違いを学んだりできるのに特殊相対論ではそれをやらないから
習ってもうーん理論は分かるけど…て分かったような分からんような感じになるんだとおも。

例えば特殊相対論的バネとか特殊相対論的調和振動子ってどう解くの?
0419ご冗談でしょう?名無しさん
垢版 |
2022/08/06(土) 17:41:00.93ID:???
特殊相対論は速度の計算とか固有時の計算とかの演習するだろう
バネとかそういうもんは手計算でできるもんじゃないからやらんけども
0422ご冗談でしょう?名無しさん
垢版 |
2022/08/06(土) 23:38:18.95ID:???
重力圏外から見た地球上のバネの運動を知りたいとかなら一般相対論の話だけども
そういう話なん?
0423ご冗談でしょう?名無しさん
垢版 |
2022/08/07(日) 11:21:51.20ID:???
そもそも特殊相対論は平坦なミンコフスキー時空なだけで別に加速運動も記述できる訳だけど、それでもバネ運動は一般相対論が要るの?
0424ご冗談でしょう?名無しさん
垢版 |
2022/08/07(日) 11:22:32.00ID:iNtMZs2x
>>422
>重力圏外
意味不明の定義をよろしく
0428ご冗談でしょう?名無しさん
垢版 |
2022/08/07(日) 11:39:44.77ID:iNtMZs2x
つまり、特殊相対性理論では、運動量もエネルギーも、したがって質量も扱うことはできない。
0430ご冗談でしょう?名無しさん
垢版 |
2022/08/07(日) 11:47:05.11ID:iNtMZs2x
よって、特殊から一般への修正は必然的であったのだが、一般では特異点が不可避であり破綻した
0432ご冗談でしょう?名無しさん
垢版 |
2022/08/07(日) 17:48:21.12ID:???
ちなみにばね定数k=1、錘の質量1kg、ばねが1m伸びた状態で手を離した場合、
錘の最大速度は 1 m/s。
特殊相対論を加味するとこれよりわずかに遅くなるが、誤差は
 5.56×10^-18

一方ばねを伸ばした状態で蓄えられている位置エネルギー 0.5J
E = mc^2 よりこれを質量換算すると 5.56×10^-18 kg
一般相対論を加味するとわずかに時間が遅くなるが、これを質点と仮定して1m離れたところでその誤差は
 4.13×10^-45

特殊相対論の効果に比べてほば完全に無視できる値だ。
0433ご冗談でしょう?名無しさん
垢版 |
2022/08/07(日) 19:36:59.18ID:???
マイケルソンモーリーは定在波の速度の調べる実験だった。
定常波なので、一定の速度しか観測されない。
0434ご冗談でしょう?名無しさん
垢版 |
2022/08/07(日) 19:41:09.93ID:iNtMZs2x
定在波に速度があるんだって、ヒソヒソ
0437ご冗談でしょう?名無しさん
垢版 |
2022/08/10(水) 14:32:52.02ID:???
頭の悪い奴ばっかだな、理論計算できなければ物理理論と言わない

特殊相対論でも(水平方向)バネの運動方程式はニュートン力学と基本は変わらん

dP/dt = -kx , dK/dt = -kxv (運動量Pと運動エネルギーKの連立方程式といえる)
(copyright)
0439ご冗談でしょう?名無しさん
垢版 |
2022/08/10(水) 15:57:03.36ID:???
馬鹿は出来もしない事を妄想するだけで何もできない、何もしない。

頭の良い人は主要素だけで簡略化し出来ることから解決する。
0440ご冗談でしょう?名無しさん
垢版 |
2022/08/10(水) 16:22:09.96ID:???
バネ定数の定義自体が問題だな
バネ定数は距離と力の比例定数だが
この距離ってのは固有距離か座標距離かで意味が違う
バネの固有定数ならバネ自体の固有距離にしたい所だが
運動中はバネの各部分は速度が違うから
すごく面倒になるぞ
座標距離なら簡単だが
バネの静止座標で観測したバネ定数が速度で変化する
という意味不明なものになる
0441ご冗談でしょう?名無しさん
垢版 |
2022/08/10(水) 17:46:07.61ID:???
バネの各部分が異なる加速度系で
その中の応力テンソル場を扱うって
重力場がないだけで計算手段は一般相対論じゃねーか
0443ご冗談でしょう?名無しさん
垢版 |
2022/08/10(水) 21:07:52.32ID:???
バネをゆっくり伸ばして手を離したら、素早く戻る過程でバネ自体がローレンツ収縮して、
伸ばした時よりも大きな力で戻るよね。
バネを伸ばした位置エネルギーより大きな運動エネルギーが得られることにならない?
0446ご冗談でしょう?名無しさん
垢版 |
2022/08/11(木) 09:36:35.97ID:???
>>418

電磁ポテンシャルを例外として、一般に力学的ポテンシャルは特殊相対論と反りが合わない。
理由はポテンシャルを導入したら光速より速く伝わる作用が可能となるから。
だから特殊相対論でポテンシャルは出てこない。

アインシュタインが一般相対論を考えた動機も、ニュートン的な重力ポテンシャルが特殊相対論で使えないからだよ。
0450ご冗談でしょう?名無しさん
垢版 |
2022/08/11(木) 17:49:28.45ID:???
ここの住人はニュートン力学でも計算が複雑な弾性物質のバネ運動から議論なしで
いきなり相対論うんぬんの基地外か?
正常人なら
慣性系で質点と座標距離に比例する力で、いわゆる単振動が特殊相対論でどう変わるか
だけ知りたいだけ
光速より非常に遅い速度では、正弦波近似の周期にならなければ相対論が間違ってる
速くなれば非正弦波になり、周期が0以外の値に収束するのか? 知りたいだろう。
0453ご冗談でしょう?名無しさん
垢版 |
2022/08/13(土) 17:13:48.69ID:???
昨日ハップル宇宙望遠鏡で640光年離れた「ベテルギウス」の大規模噴火がニュースになりました。
640年前の星の爆発を見ることができるって何だか不思議ですね。
これを量子論で考えた場合、現時点では640年前に噴火が起きた事実が確認できます。
観測できたのが昨日だとすると、おとといの時点ではベテルギウスは噴火しているのでしょうか?
1.噴火していない
2.噴火している
3.噴火している状態と噴火していない状態が重なり合っている
0458ご冗談でしょう?名無しさん
垢版 |
2022/08/15(月) 22:02:40.22ID:???
>>446
ポテンシャルを導入すると超光速が出てくるのも解んないし電磁ポテンシャルならそれが回避できるというのも分からない。 

だとしても、それなら特殊相対論的に許されるポテンシャルは何か、という議論になるのが自然では?
0460ご冗談でしょう?名無しさん
垢版 |
2022/08/16(火) 19:04:41.19ID:???
飛行機の中にハエが飛んでると、ハエの飛ぶ速度はどれ位なんだろう。
飛行機の中の人から見れば時速1km/h位だとして、飛行機の外から見ると時速300km/h位か。
地球は時速1700km/hで自転している。
火星から地球の人を見ると時速1700km/hで移動してるように見えるんかな。
時速の概念が立ち位置により変わる考え方なので、光の速度を時速で表現するのが適切ではない気がする。
0461ご冗談でしょう?名無しさん
垢版 |
2022/08/17(水) 01:22:55.09ID:???
>>458

超光速のくだりは江沢相対論の受け売りだ。悪かったね。
読み直してみたが掘り下げて書いてなかったわ。

ローレンツ力の式 F = eE+ev×B に第二項が無かったら、電磁気学が相対論と矛盾するだろ。
力学的ポテンシャルでは駄目で、電磁ポテンシャルなら回避できるという事実がここにある。

>特殊相対論的に許されるポテンシャルは何か?

遠心力ポテンシャルのような見かけのものや誤差を無視してよい近似のものを除けば
おそらくケージ場だけだろうね。
0463ご冗談でしょう?名無しさん
垢版 |
2022/08/17(水) 07:17:02.28ID:???
>>461
「力学的ポテンシャルは特殊相対論と反りが合わない」ってのは導入や計算が難しいって意味かと思ったら、矛盾するって意味か?
力学的ポテンシャルだろうがなんだろうが矛盾したらおかしいだろ
0464ご冗談でしょう?名無しさん
垢版 |
2022/08/17(水) 11:59:25.07ID:???
位置だけの関数になっているなど、同時刻の相対性を考慮してない定義だったりしたら
特殊相対論とは矛盾するわな
0465ご冗談でしょう?名無しさん
垢版 |
2022/08/17(水) 13:10:41.77ID:???
ポテンシャル中の質点の運動を考える。
質点の運動エネルギーは質点が静止する位置でゼロ(最小)になるから、
質点が静止する位置でのポテンシャルの値は最大になる。
ところが質点の静止する位置というのは、慣性系の選び方によって無数に変化する。
ある位置でのポテンシャルの値は、慣性系の選び方次第で最大になったりならなかったり、
さらには勝手な値を取り得る。
つまりポテンシャルはローレンツ変換またはガリレイ変換に対するスカラー関数としての
基本的な性質を満たさない。

こんな変なもの相対論の中でどうやって使うんだよ?
0467ご冗談でしょう?名無しさん
垢版 |
2022/08/17(水) 15:35:19.77ID:???
重力ポテンシャルを相対論で扱おうとすると一般相対論になってしまう
バネとかだったら元は電磁力
他のゲージ場もあるがな
0468ご冗談でしょう?名無しさん
垢版 |
2022/08/18(木) 08:10:28.24ID:???
>>466

「ガリレイの相対性理論」では使ってないから困ることはない。
ニュートン力学でポテンシャルを使っているのは、座標変換が時間を含まないものか、
含んでいるとしてもそれでポテンシャルが変化しないものだけ。
0469ご冗談でしょう?名無しさん
垢版 |
2022/08/18(木) 09:30:57.34ID:???
解析力学ではポテンシャルが時間を陽に含む場合も議論されとるがな。

ポテンシャルを使って相対論「的」な計算をしたければ、
(1/2)mv^2をmc^2に変えれば良いだけじゃないのか?
0470ご冗談でしょう?名無しさん
垢版 |
2022/08/18(木) 15:01:16.39ID:???
>座標変換が時間を含まないもの
そんな状況なら相対論でも困らん

>含んでいるとしてもそれでポテンシャルが変化しないもの
そんな状況なら相対論でも困らん
0473ご冗談でしょう?名無しさん
垢版 |
2022/08/22(月) 23:34:58.24ID:4BD0JdTo
ブラックホールの蒸発のメカニズムが今一理解できないんだが説明してくれる人いる?
事象の地平面に粒子・反粒子のペアが生成された際、反粒子のみがブラックホールに
捕らわれた際に起こるホーキング放射により、ブラックホールは徐々に蒸発していくという
説明が正しいとして、よく解らないのが、反粒子と同じ確率で粒子もまた事象の地平面
に捕らわれ、その結果ブラックホールの質量は徐々に増大するから、全体としてみた場合、
ブラックホールの質量は変わらないんじゃないか?
0474ご冗談でしょう?名無しさん
垢版 |
2022/08/23(火) 00:14:13.51ID:???
対生成された一方を粒子もう一方を反粒子というだけで、
どっちが落ちても落ちなかった方が質量を持ち去るから一緒やで。
0475ご冗談でしょう?名無しさん
垢版 |
2022/08/23(火) 00:19:23.82ID:???
>>473じゃないが>>474の説明はよく分からない
質量を電荷に置き換えても同じ議論ができそうで、つまり正電荷を帯びたBHの蒸発を考えるとして、すると粒子と反粒子の区別はつけられるように思えるが
0479ご冗談でしょう?名無しさん
垢版 |
2022/08/23(火) 06:50:38.62ID:???
+電荷に-電荷を放り込んで電気的に中和されるのを電荷の蒸発とはいわない
粒子も反粒子も正の質量を持つ
質量が出ていくのと電気が中和されるのはまったく別の話
0480ご冗談でしょう?名無しさん
垢版 |
2022/08/23(火) 10:55:06.61ID:???
だから、電荷を持つBHを蒸発させるには電荷を消滅させることが必要条件だから、電荷の片寄ったBHは粒子で構成されてるとして、そのBHは事象の地平面で反粒子がより多くBHに落ち込むような作用が無いとおかしいんじゃないかって話
0481ご冗談でしょう?名無しさん
垢版 |
2022/08/23(火) 20:26:41.31ID:???
>>480
それだったら+に帯電したBHの周りには外向きの電場があるんだから、
電子と陽電子が対生成したとき、陽電子が遠方に飛び去って電子がBHに落ち込む確率の方が高くなるでしょう
0482ご冗談でしょう?名無しさん
垢版 |
2022/08/23(火) 20:35:40.95ID:???
>>481
まあそうなるわね
それでさ>>474の説明はおかしいじゃん
粒子と反粒子はどっちがどっちだか決まってるわけじゃん
BHは粒子で構成されてる筈だから、落ち込むのは反粒子じゃないといけない。落ち込んだ方が反粒子とはならない。落ち込んだe-は反粒子じゃないでしょ?
0483ご冗談でしょう?名無しさん
垢版 |
2022/08/23(火) 22:13:06.16ID:???
>>482
> 粒子と反粒子はどっちがどっちだか決まってるわけじゃん

決まってない
反粒子とはある素粒子(または複合粒子)と比較して、質量とスピンが等しく電荷など正負の属性が逆の粒子を言う
例えば電子と陽電子、どっちを粒子と呼んでどっちを反粒子と呼んでもいい

BHの蒸発に関しては、もちろん粒子と反粒子どっちが飛び去っても質量は減る
どっちも正の質量を持ってるんだから
0484ご冗談でしょう?名無しさん
垢版 |
2022/08/24(水) 02:02:22.03ID:???
>>483
どちらを粒子と呼ぶかには任意性があるけど一度決めたら好きに変えちゃダメでしょ?
BHは(殆ど)「粒子」の集合が重力崩壊を起こしてできた「粒子の星」なので、蒸発のために事象の地平面で内側に落ち込むのは、既に定められた反粒子の方でなければいけない。「落ちた方を反粒子と呼ぶ」なんて芸当は既にできない筈だが
0485ご冗談でしょう?名無しさん
垢版 |
2022/08/24(水) 07:34:32.90ID:???
>>484
それなら勘違いしてるのはここだ

> BHは粒子で構成されてる筈だから、落ち込むのは反粒子じゃないといけない。
> 蒸発のために事象の地平面で内側に落ち込むのは、既に定められた反粒子の方でなければいけない。

BHの蒸発ってのは、BHの中の構成物が反物質と対消滅して起こる現象じゃない。
粒子だろうが反粒子だろうが直接BHに放り込んだら質量が増えてBHがでかくなるだけ。
そうじゃなくて対生成された粒子なり反粒子なりの一方が遠方にエネルギー(質量)を持ち去るから蒸発するって話だ。
このときエネルギーを持ち去るのは粒子であるか反粒子であるかはどちらでもよい。
0486ご冗談でしょう?名無しさん
垢版 |
2022/08/24(水) 10:00:58.40ID:???
>>485
それだったら電子-陽電子対が生成されたとき、どちらも質量は正なのでBHの質量が増えながら質量を放出するってならないか?
これってそもそも対生成で説明するの無理がないか?
0487ご冗談でしょう?名無しさん
垢版 |
2022/08/24(水) 13:08:37.01ID:???
>>473
2つの話が混線してる
まず反粒子が出ていくんじゃなく正負のエネルギーを持つ粒子のうち
正エネルギー粒子が出ていき、負エネルギー粒子がブラックホールに
落ちることでブラックホールの質量が減る
この理由は
運動エネルギー + ポテンシャルエネルギー > 0
の場合しか脱出軌道にならないから。

次の話はブラックホールに正物質が落ちても
ホーキング放射は正反両方が均等に出ていくから
「正物質」という情報が失われる。という
「ブラックホールの情報喪失問題」
これはホーキングが賭けに負けたことで有名だけど
ホーキングが主張した情報喪失は結局否定されて
正物質が落ちたら正物質が出ていき
情報は失われないという結論になった
0488ご冗談でしょう?名無しさん
垢版 |
2022/08/24(水) 18:26:38.28ID:???
>>487
なるほどね、かなり釈然と行く説明だわ
「正物質が落ちて正物質が出ていく」ってうのは不思議だなあ、何か理由あるの?
0490ご冗談でしょう?名無しさん
垢版 |
2022/08/24(水) 21:34:26.40ID:dOJgHEgP
ブラックホールには誰も落ちることはできない。要するに、そんなものは存在しない
0494ご冗談でしょう?名無しさん
垢版 |
2022/08/27(土) 16:18:33.86ID:FOJAv3Le
>>461
特殊相対論のポテンシャルってそんなに制約がきついんだ。
スカラー場φをもってきて∂μφとすれば必ずベクトルになるから古典力学のようにこれを相対論的ポテンシャルとして議論はできないの?

例えば調和振動子に似せてφ=x^2としたら自然に特殊相対論の要請を満たす力がでてこない?
としたらスカラーやベクトルとは別の制約を持ち込んで特殊相対論的に許容される
ポテンシャルや力というものを定義するといった議論になっても良い気がするのだがどうだろう。
0496ご冗談でしょう?名無しさん
垢版 |
2022/08/27(土) 18:49:39.62ID:???
場所だけで力が決まるなら矛盾せんだろう
ばねの運動などは場所だけでは力が決まらんからややこしいだけで
0499ご冗談でしょう?名無しさん
垢版 |
2022/08/28(日) 12:26:30.56ID:???
>>498
座標変換すればポテンシャルの形が変わるのはあたりまえだろう。
そんなのはガリレイ変換でも同じだ。
変換前の座標に対してポテンシャルが定義できる状況なら、変換後の座標に対してもポテンシャルは定義できるので問題ない。
0500ご冗談でしょう?名無しさん
垢版 |
2022/08/28(日) 12:35:57.66ID:zi66JC7U
ポテンシャルはスカラーだからローレンツ変換とやらを何度やっても変わらないあるね。よって問題解決
0501ご冗談でしょう?名無しさん
垢版 |
2022/08/28(日) 13:34:26.44ID:???
ポテンシャルはスカラーだけでない。

電磁場はベクトルポテンシャル、重力場はテンソルになるから座標変換で変わる。
0502ご冗談でしょう?名無しさん
垢版 |
2022/08/28(日) 13:37:46.73ID:zi66JC7U
電位はスカラーポテンシャルでしょ。だから別扱いされる
0503ご冗談でしょう?名無しさん
垢版 |
2022/08/28(日) 15:01:59.21ID:???
重力場がテンソルってどういうことなんだろうね?
質点にかかる重力なんて普通にベクトルで表せるような気がするが
0505ご冗談でしょう?名無しさん
垢版 |
2022/08/28(日) 19:24:36.92ID:???
そりゃ重力場方程式は書けんでしょう
質点にかかる重力がベクトルで表せるんだったら、別に重力場方程式を書く必要もないでしょう
0508ご冗談でしょう?名無しさん
垢版 |
2022/08/29(月) 13:06:39.88ID:???
>>505
重力場方程式を何だと思ってるんだ?
重力場の変化を記述する微分方程式だぞ
重力場自体は放置で重力場の効果しか興味ないのか
0509ご冗談でしょう?名無しさん
垢版 |
2022/08/30(火) 07:20:52.01ID:???
>>507

ポテンシャルの意味や定義、あるいはポテンシャルを考える理由を教えてくれよ。
他の物理概念との関係性を何も与えず、ただ座標変換と整合するだけでいいなら、
それは単なるスカラーだ。
0512ご冗談でしょう?名無しさん
垢版 |
2022/08/30(火) 12:03:51.54ID:UX119HDV
じゃ、言ってみ
0513ご冗談でしょう?名無しさん
垢版 |
2022/08/30(火) 17:54:56.92ID:???
四次元時空内に分布しているある物理量が、座標系S、S'でそれぞれA(x)、A'(x')と表わされているとする。
xとx'が同じ時空点を指しているときA(x)=A'(x')となるならば、A(x)をスカラー場という。
スカラー場はしばしばスカラーと省略されて書かれる。
0516ご冗談でしょう?名無しさん
垢版 |
2022/08/30(火) 23:45:29.43ID:UX119HDV
あなんか書いてる以上の意味はなさそう
0523ご冗談でしょう?名無しさん
垢版 |
2022/09/28(水) 14:40:51.25ID:???
ここで聞いていいのかわからないのですが、疑問に思ったことがあるので質問です。
光の速度のロケットで進むと時間が遅くなり止まるという話がありますが、
相対的に地球はロケットから光の速度で遠ざかっていることになるので、
ロケットから見れば地球が光の速度で遠ざかっていて、
地球では時間が止まるということになると思うのですが、
それぞれの観測者がパラレルワールドに分岐して世界線がずれていくような想像をついついしてしまいます。
このあたりをわかりやすく教えてほしいです。
0524ご冗談でしょう?名無しさん
垢版 |
2022/09/28(水) 15:06:30.57ID:zMI7ILLn
>光の速度のロケットで進む
そんなものは実現不可能だから忘れて良い
0526ご冗談でしょう?名無しさん
垢版 |
2022/09/28(水) 18:10:16.13ID:???
>>523
時間が遅くなるのは観察対象である相手側であって
ロケットも地球も自分の時間は普通に流れているので矛盾は起きないのです
0527ご冗談でしょう?名無しさん
垢版 |
2022/09/28(水) 19:00:47.30ID:???
地球側の人から観測すると光速ロケットの中の人は歳をとらない。
ロケット側の人から観測すると、相対的にロケットから光の速度で地球が遠ざかっていったので地球こそが光速に達したことになり、地球の人が歳をとらない。
どっちが正しいのか、という質問。
0528ご冗談でしょう?名無しさん
垢版 |
2022/09/28(水) 19:23:01.10ID:???
「光の速度」で遠ざかるのは不可能なので「光に近い速度」と読み替えたうえで・・・
どっちも正しい。以上
0530ご冗談でしょう?名無しさん
垢版 |
2022/09/29(木) 08:32:24.26ID:???
ならば実際に2者が出会ってみたら、どちらが歳をとっているのか
矛盾ではないのか
というのが昔から言われている「双子のパラドックス」という話だ
0532ご冗談でしょう?名無しさん
垢版 |
2022/09/29(木) 12:03:15.85ID:sayvQ6L9
お腹が空いた <=> お金がない みたいな話だよ
0533ご冗談でしょう?名無しさん
垢版 |
2022/09/29(木) 12:39:22.45ID:???
パラドックスではないよ
それについてはID:6XJdBHCuが上でいろいろ書いてる
ちなみに兄弟が再び出会わずずっと離ればなれになるような状況を考える場合はそもそも過ぎた時間を比べることができないので何の意味もないよ
どちらも相手が止まっていくように見えるのは単に向こうからの映像が遅れて届いているようなものだからね
0535ご冗談でしょう?名無しさん
垢版 |
2022/09/29(木) 13:02:00.46ID:???
詳しいことはわからないですが、
地球は慣性系(直線)で、ロケットは加速系(曲線)なので、
実際には地球の方が時間が経過しているという話なんですかね???

加速しないで一瞬で光速に達する場合や、光速で地球を通過するロケットに飛び乗り、
大きな重力源にそってぐるりと曲線を描いて加速も減速もなく地球に光速のまま戻ってきたら結果は変わるのでしょうか・・・
よくわからなくなってきました・・・
0536ご冗談でしょう?名無しさん
垢版 |
2022/09/29(木) 13:48:54.14ID:???
>>535
そんな感じですね
少なくとも重力場がない平坦な空間においては、ユークリッド的、つまり一つの点から二つの直線が出て再び交わることはない。なので片方が直線(慣性系)で、両者が再び出会うなら、もう片方は必ず曲線(加速系)にならざるを得ない。そして曲線の方が直線よりも一般的に過ぎる時間が遅いという話です
あと、単にどういう経路を通るかの問題でしかないので「一瞬で光速になって~」とかは関係ないです。

重力場がある場合は...結局経路の「長さ」によって比較した結果が出てくるという感じでしょうね
0537ご冗談でしょう?名無しさん
垢版 |
2022/09/29(木) 13:57:04.31ID:sayvQ6L9
頭の周りをハエがぐるぐる回るとどうなるの?
0543ご冗談でしょう?名無しさん
垢版 |
2022/10/01(土) 08:21:33.99ID:???
>>539 に同意
この板全体的に、言葉で書かれていないどころか背景や文脈としてもありえないことを勝手に読み取って、よくわからんコメントするやつ多すぎ
0546ご冗談でしょう?名無しさん
垢版 |
2022/10/01(土) 16:40:37.60ID:???
下のに書いてあったのですが、なんでアインシュタインは特殊相対性理論で電磁気学しか扱わなかったのですか?相対性理論に基づいた力学には何か困難があったんでしょうか?
電磁気学だけでは質量のある電子の運動とかは記述できないと思うのですが、そういうのはガン無視で電磁波だけ考えてたんですか?

https://jp.quora.com/%E3%81%AA%E3%81%9C%E7%89%B9%E6%AE%8A%E7%9B%B8%E5%AF%BE%E6%80%A7%E7%90%86%E8%AB%96%E3%81%AF%E7%AD%89%E9%80%9F%E7%9B%B4%E7%B7%9A%E9%81%8B%E5%8B%95%E3%81%A7%E3%81%AA%E3%81%84%E3%81%A8%E3%81%AA%E3%82%89%E3%81%AA/answers/388148185?ch=15&oid=388148185&share=02f3af65&srid=ueWWYJ&target_type=answer
0548ご冗談でしょう?名無しさん
垢版 |
2022/10/01(土) 16:59:03.16ID:???
アインシュタインの論文のタイトルは「運動物体の電気力学について」だったけど
電磁気学と関係あるのは光速だけじゃん
0549ご冗談でしょう?名無しさん
垢版 |
2022/10/01(土) 17:05:25.13ID:???
特殊相対論は加速系を扱えないとか意味不明なことを言う連中もいるけど
そいつらは特殊相対論の基本すら理解していない。
0550ご冗談でしょう?名無しさん
垢版 |
2022/10/01(土) 17:07:02.45ID:???
>>547
mc^2のmとかやっぱり力学ですよね

>>548
論文の内容知らないから上のurlから想像したんだけど、もしかしてマックスウェル方程式とローレンツ変換についての論文じゃなかったりする?
0551ご冗談でしょう?名無しさん
垢版 |
2022/10/01(土) 17:14:32.45ID:???
Einsteinの相対論の原論文の§10は、
加速度が小さい場合の電子の力学
という題名で、電磁場で加速される電子の運動を論じている。
質量とエネルギーの取り扱いに注意が必要であることを明確に述べている。
0553ご冗談でしょう?名無しさん
垢版 |
2022/10/01(土) 17:29:50.83ID:???
>>552
ドイツ語よくわからんけど、§10の最初の式って、F=maであってるよね?
やっぱり質量のある物体の運動扱ってるやんけ
quoraに書いてるの完全にデタラメだったのかな
0554ご冗談でしょう?名無しさん
垢版 |
2022/10/01(土) 17:30:05.41ID:???
特殊相対論が「特殊」なのは、「重力を扱っていない」という意味であって
「加速度を扱えないから特殊」というのは完全に間違った解釈。
原論文の§10を見れば一目瞭然で、
明確に加速度と力と質量の新しい関係を定義している。
0555ご冗談でしょう?名無しさん
垢版 |
2022/10/01(土) 17:32:37.08ID:???
quoraはバカも多いから、肩書や他人の評価を盲信せずに
書いてある内容を自分で吟味しなければならない。
0556ご冗談でしょう?名無しさん
垢版 |
2022/10/01(土) 17:43:44.03ID:???
Einsteinの相対論の原論文はドイツ語だから分かりづらいという人は
アインシュタイン 相対性理論 (岩波文庫)
を見れば§10で縦質量と横質量という
新しい相対論的概念が導入されていることは理解できるはず。
ただし、解説を書いてる内山だか山内だかは
説明が極めて杜撰で分かりづらいので見ない方がいい。
0557ご冗談でしょう?名無しさん
垢版 |
2022/10/01(土) 17:51:43.24ID:???
特殊相対論の良い解説書は日本語の本はろくなのがないけど
英語で良ければ
Special relativity for beginners: A Textbook for Undergraduates
Jurgen Freund
が良いと思う。
これも奇しくも原著はドイツ語のようだけど、
新しい概念の導入を細かくセクションに区切って解説していて
演習問題も効果的なものだけが厳選されてるから解く意味がある。
0558ご冗談でしょう?名無しさん
垢版 |
2022/10/01(土) 18:06:47.65ID:???
手元には積読のシュッツって人の相対性理論の本上下巻があるんだけど、これじゃだめかな?
quoraのトンデモ見分けられるようになれると嬉しい
0559ご冗談でしょう?名無しさん
垢版 |
2022/10/01(土) 18:13:49.79ID:BwEgKKx1
いろいろ10冊くらい買えば理解できるかな?
0560ご冗談でしょう?名無しさん
垢版 |
2022/10/01(土) 18:26:56.05ID:???
本を買うだけでは理解できないし
杜撰な説明の本を何百冊読んでも時間の無駄にしかならない。
ガキが書いた相対論の本を、中身も読まずにニュースキャスターくずれの老人が褒め称える
といったいびつな出版社会である日本では初学者向けのマトモな相対論の解説書など出るわけがない。
0564ご冗談でしょう?名無しさん
垢版 |
2022/10/01(土) 23:17:09.04ID:BwEgKKx1
なんでいけないの? 単に、理解できなかっただけじゃないの?
0568ご冗談でしょう?名無しさん
垢版 |
2022/10/01(土) 23:38:05.20ID:BwEgKKx1
よくわかんなんだけど、物を縦に置いたときと横に置いたのでは相対的に質量が違うのかな?
0570ご冗談でしょう?名無しさん
垢版 |
2022/10/02(日) 01:02:29.08ID:YmhGhvVi
長い方が縦、短い方は横だっけ。あれ、逆だっけ?
0571ご冗談でしょう?名無しさん
垢版 |
2022/10/02(日) 13:03:36.19ID:???
相対論で速度によって質量が違うなんて考えると
速度方向の慣性で測る質量と垂直方向で測る質量が違ってしまう
それを縦質量, 横質量と言ったのさ
馬鹿げてるから今じゃ抹殺されたがな
0572ご冗談でしょう?名無しさん
垢版 |
2022/10/02(日) 15:13:59.24ID:???
>>569
アインシュタインの特殊相対論論文では力(ベクトル)の定義をニュートン力学と同じ
F = ma にしたために質量がスカラー量でなく運動方向に依存した値(縦質量、横質量)になる
力と加速度の定義を変えれば式も変わるとアインシュタインが論文に記述している。

現代の相対論では静止質量m0がスカラー量であり、力と加速度の方向が一致しない。
0574ご冗談でしょう?名無しさん
垢版 |
2022/10/02(日) 20:33:29.51ID:YmhGhvVi
もうめちゃくちゃ、意味不明
0577ご冗談でしょう?名無しさん
垢版 |
2022/10/03(月) 05:15:43.44ID:qAtf/KGC
やはり、相対性理論は間違っとる。確信はくなった
0580ご冗談でしょう?名無しさん
垢版 |
2022/10/03(月) 07:57:39.19ID:qAtf/KGC
悔しそう。勝った、勝った
0584ご冗談でしょう?名無しさん
垢版 |
2022/10/03(月) 13:32:50.92ID:???
単なる数式の解釈に対する誤解の話なのになあ
「『落下する隕石が赤熱するのは摩擦熱のせい』は誤りだよ」って話をしたら「物理学は間違ってる!」って返ってきたような滑稽さだ
0585ご冗談でしょう?名無しさん
垢版 |
2022/10/03(月) 13:54:20.80ID:qAtf/KGC
それ赤方偏移、それくらいしてるさ
0586ご冗談でしょう?名無しさん
垢版 |
2022/10/03(月) 14:05:01.83ID:???
赤方変移なら何で隕石接近してんのに赤くなるんだよ、波長が短くなるんだから青くなるだろが
赤方変移は関係ないんだよ
0587ご冗談でしょう?名無しさん
垢版 |
2022/10/03(月) 14:07:00.69ID:qAtf/KGC
あ、そうなの。じゃなんで?
0590ご冗談でしょう?名無しさん
垢版 |
2022/10/03(月) 14:21:18.58ID:???
一般人向け特殊相対論の解釈では質量物体を加速しても光速に到達できない理由を
速度が光速に近づくと慣性質量が増大して光速では無限大になるからと説明する。

さらに、物体を加速するための外力(ベクトル)の方向が運動方向に対して斜め
の場合、外力の方向と加速度の方向が異なる。
運動方向の加速度の係数を縦質量、運動方向に垂直な加速度の係数を横質量という。

単純な直角レバーのパラドックスは外力と加速度の関係を考慮しない計算から起こる。
0591ご冗談でしょう?名無しさん
垢版 |
2022/10/03(月) 14:21:45.71ID:qAtf/KGC
それって、相対性理論とやらと関係するの?
0597ご冗談でしょう?名無しさん
垢版 |
2022/10/04(火) 23:23:05.40ID:???
>>590
>単純な直角レバーのパラドックスは外力と加速度の関係を考慮しない計算から起こる。
直角フレームレバー端の質点に静止状態で同じ大きさの外力Fが作用している、支点・は固定とする
下図は静止状態では等長の直角レバー全体がx方向の速度v運動でローレンツ収縮した状態、
 
 ・――・m
 |  ↑Fy
 |
 |
m・←Fx
従来一般向けの特殊相対論の解説ではローレンツ収縮でLx=Lγに縮小し、質点の質量が 
m = m0/γ に増大するなどと書いてある。
静止質量m0 , γ= √(1-(v/m)^2) <= 1 , Ly=L

単純な直角レバーのパラドックスの説明では、Fyのみにγ因子が掛かって Fy=Fγ
力のモーメントの大きさ比較で LyFx = LF > LxFy = LFγ^2  右回転してしまう
という説だ。
つづく
0598ご冗談でしょう?名無しさん
垢版 |
2022/10/05(水) 00:02:12.61ID:???
>>597
このパラドックス説でおかしいのはローレンツ変換でFyのみが小さくなるのかということ
正しいローレンツ変換をすればFxも当然小さくなるはずである。

ここでは簡単にパラドックスを解決する為に、アインシュタイン論文の「縦質量」「横質量」
の定義を現代的に解釈し直してみる
つまり、静止質量がスカラーで不変であり、力ベクトルがローレンツ変換で小さくなる。
縦方向の力 Fx = Fγ^3 , 横方向の力  Fy = Fγ^2 となる
力のモーメントの大きさを比較すれば
 LyFx = LFγ^3 == LxFy = LFγ^3
しっかり釣り合っているから回転しない!

時代遅れの「縦質量」「横質量」の式をしっかり使って直角レバーのパラドックスが簡単に解決した。
めでたし、めでたし
0605ご冗談でしょう?名無しさん
垢版 |
2022/10/05(水) 00:49:51.18ID:???
>>598
相対運動で光速に近づけば「力」の作用は0に収束するということだ
例えば2個の電荷同士が並進運動してれば光速の極限で電磁気力が0になる。
0613ご冗談でしょう?名無しさん
垢版 |
2022/10/05(水) 10:49:03.27ID:???
相対論のγの定義を勝手に変えるというのは確かにアスペそのものだな
コミュニケーションに必要な共通概念を持たないから著しい障害を生じる
0614ご冗談でしょう?名無しさん
垢版 |
2022/10/05(水) 12:07:21.06ID:???
相間のwebサイトでは直角レバーのパラドックス説を鵜呑みにして
特殊相対論が間違ってる証拠だと解説している。
0615ご冗談でしょう?名無しさん
垢版 |
2022/10/05(水) 12:23:55.41ID:???
>>598も、やってることは相間と同じだね
相対論の正しい変換式を否定することによってパラドックスを解決したつもりになっているから
0618ご冗談でしょう?名無しさん
垢版 |
2022/10/05(水) 12:56:22.40ID:???
>>616

自覚のない相間ですか?
力に対して相対論の教えるものとは異なる定義をして、それが(パラドックスを解決するので)正しい力の定義だと言ってるんでしょ?
つまり相対論の教える定義は間違っていると言ってるのと同じじゃん
0621ご冗談でしょう?名無しさん
垢版 |
2022/10/05(水) 13:23:16.93ID:???
>>618
>力に対して相対論の教えるものとは異なる定義をして、それが(パラドックスを解決するので)正しい力の定義だと言ってる
頭悪いな

アインシュタイン特殊相対論論文の力(加速度)の定義では(運動方向に依存した)質量が増大する
と解釈して「縦質量」「横質量」の式になってる。
それを現代的に解釈し直すと(静止)質量が不変で力が(運動方向に依存して)小さくなる。
その再定義使えば、直角レバーのパラドックスが簡単に解決できるのだよ。

以前のローレンツ力を使う計算方法より簡単に解決できる、4元ベクトルも必要ない。
0623ご冗談でしょう?名無しさん
垢版 |
2022/10/05(水) 13:50:45.57ID:???
>>622
今の特殊相対論の教科書では F=ma 型の定義を使ってないだけだ。

古い定義を再解釈しても特殊相対論であることに変わりない。
0624ご冗談でしょう?名無しさん
垢版 |
2022/10/05(水) 14:17:00.00ID:???
>>623

いや、相対論から逸脱してるから。
Fyなんて力の(正規の)ローレンツ変換をした上に俺様解釈を重ね掛けして帳尻を合わてるけど、いいのこれ?
0625ご冗談でしょう?名無しさん
垢版 |
2022/10/05(水) 14:30:34.89ID:???
>>624
>Fy
アインシュタイン論文の定義による「横質量」の式をy方向の力に再解釈しただけだ

>力の(正規の)ローレンツ変換
そもそも力の定義が違うのだからそれ(正規)を混ぜて考える意味がない
0626ご冗談でしょう?名無しさん
垢版 |
2022/10/05(水) 16:02:20.32ID:???
>>625

横質量の式 F=γma の両辺をγで割って F/γ=ma という式を作って、F/γを新たに力と再解釈したら、
元々のF=d(γp)/dt (力は運動量の時間微分に等しい)が意味を失うじゃないか。
0627ご冗談でしょう?名無しさん
垢版 |
2022/10/05(水) 16:17:04.87ID:???
>626
>横質量の式
アインシュタイン論文の横質量の定義式をちゃんと見てみ オマエの式と違うだろ
0629ご冗談でしょう?名無しさん
垢版 |
2022/10/05(水) 16:41:51.21ID:???
>>627

確認した。確かに違うね。
しかしそうするとF=eEが問題になるね。どうするの? F/γ^2=eE/γ^2 として電場が電荷を解釈し直すの?
0632ご冗談でしょう?名無しさん
垢版 |
2022/10/05(水) 17:01:10.93ID:???
>>631

アインシュタインの論文の話をしてるんだよ。
μβ^2 d^2z/dt^2 = εE'_ζを「現代的に解釈し直す」と F/β^2 = μ d^2z/dt^2 = εE'_ζ/β^2 でしょ。
電場か電荷も解釈し直さないと、辻褄が合わない。
0633ご冗談でしょう?名無しさん
垢版 |
2022/10/05(水) 17:48:43.14ID:???
>>632
アインシュタイン論文の力の定義と縦質量、横質量の式の導出は
電磁場のローレンツ変換・ローレンツ力を使ってるから当然、辻褄は合ってる。

自分で納得するまで計算してみるんだな
おしまい
0635ご冗談でしょう?名無しさん
垢版 |
2022/10/05(水) 17:57:11.69ID:???
アインシュタインの論文の中身は辻褄が合っていて当然。
そこに俺様流現代的解釈を入れてしまえば皺寄せが多方面に及ぶけどどうするんだよ、って話なんだけど。

結局>>598は、その場しのぎの説明でしかないんだよ。
0637ご冗談でしょう?名無しさん
垢版 |
2022/10/05(水) 20:30:59.72ID:???
>>635
頭悪い奴と粘着アスペ荒らししか湧かんな

初期の特殊相対論であるアインシュタイン論文ではニュートン力学の第二法則の形式を
使用して力(加速度)を定義している。
a = F/m
相対速度で観測する加速度ベクトルaが変化する効果の解釈で力Fを変えずに質量mを変化させる
のがアインシュタイン論文の相対論解釈だから、一般解説書も質量が増大するなどと説明している。

当然、特殊相対論の理論を変えないで、質量mを変えずに力ベクトルFを変化させる解釈も成り立ち、論理矛盾も起こらない。
それを「直角レバーのパラドックス」の簡単な解決に応用しただけだよ。

5ちゃん馬鹿どもにはそれが理解できないらしい。
0642ご冗談でしょう?名無しさん
垢版 |
2022/10/05(水) 21:20:16.85ID:???
>>637
ニュートン以来、慣性系の(真の)力は絶対(不変)と思われてきた、日本語の「力学」
という用語はそれを象徴している。
それに倣ってアインシュタインの最初の特殊相対性理論でも力を変えないで質量を変化させ
「縦質量」「横質量」を定義した。
結果として
特殊相対性理論の登場によって、力の絶対性は否定され定義で変化するものであり、
静止質量(と電荷)がローレンツ変換の不変量(スカラー)と認識されたのだよ。

めでたし、めでたし
0644ご冗談でしょう?名無しさん
垢版 |
2022/10/06(木) 04:30:20.84ID:???
>>638
>レバーパラドックスはローレンツ変換だけじゃ解けない
相間のアスペ馬鹿か

特殊相対性理論の原理はローレンツ変換と相対性原理しかない
それを基に”特殊相対論のパラドックスが解決不能”ならば理論が間違ってることになる
現在までそんなパラドックスは無いから、相間馬鹿が力の定義か計算が間違ってるだけだ。

相間のwebサイトを見ればローレンツ変換関連の数式だけかき集め、おかしな計算して
”相対論は間違ってる”と主張してるだけなのが判る。
0647ご冗談でしょう?名無しさん
垢版 |
2022/10/06(木) 05:13:56.80ID:???
レバーパラドックスは江沢の相対性理論に載っているが、
力を解釈し直せば釣り合うなどという出鱈目な説明はしていない。
0648ご冗談でしょう?名無しさん
垢版 |
2022/10/06(木) 06:44:04.92ID:???
>>644
いつもいつも数式だけかき集めておかしな計算して答えだけ辻褄合わせてんのはおまえだわな
相間のwebサイトとおまえは結論が反対なだけで、やってること自体なにも変わらん
0649ご冗談でしょう?名無しさん
垢版 |
2022/10/06(木) 07:49:54.54ID:???
>>644
>それを基に”特殊相対論のパラドックスが解決不能”ならば理論が間違ってることになる
馬鹿、実際お前は解けてないじゃん。
つうか、力のローレンツ変換すら未だに出来ない馬鹿だし。
0650ご冗談でしょう?名無しさん
垢版 |
2022/10/06(木) 11:16:39.05ID:???
>>642 続き
>特殊相対性理論の登場によって、力の絶対性は否定され定義で変化するものであり、
>静止質量(と電荷)がローレンツ変換の不変量(スカラー)と認識されたのだよ。

それに基づいた力の定義を書くと (注意:定義だから認めない人は論外)
dP/dt = F , P = m0V/γ , γ=√(1-(v/c)^2) P:運動量ベクトル F:力のベクトル
V:速度ベクトル (大文字はベクトルを示す)
つまり、始めに力の定義をしてから議論しないからおかしな計算間違いが蔓延るのだよ。

それを前に書いた単純な直角レバーのパラドックスに適用すると力の計算結果は
>>597
 ・――・m
 |  ↑Fy
 |
 |
m・←Fx

Fx = fγ^2 , Fy = fγ となる、力のモーメントは Tx = Lfγ^2 , Ty = Lfγ^2
大きさが等しく釣り合っている。(アインシュタイン論文は力の定義が違うだけ)
めでたし めでたし
以上が特殊相対性理論の「力学」の基本、初心者が長さが縮む、時間が伸びるだけでは
ダメなのだと判るだろ

粘着アスペの類の自分で何もせず貶し荒らししかできない馬鹿にならないように気を付けようね
0651ご冗談でしょう?名無しさん
垢版 |
2022/10/06(木) 12:15:30.61ID:???
実際の物理的な力を反映していない「自分勝手に定義した力」で何か意味のあることが言えると本気で思っているのか?
0654ご冗談でしょう?名無しさん
垢版 |
2022/10/06(木) 12:51:31.70ID:sgqAUW03
【ワクチン被弾】 中日ドラゴンズ木下雄介投手(27)
://rio2016.5ch.net/test/read.cgi/meikyu/1665016356/l50
0655ご冗談でしょう?名無しさん
垢版 |
2022/10/06(木) 12:58:30.01ID:???
>>651
>実際の物理的な力を反映していない
頭悪いな

アインシュタイン論文を基に現代的な力の再定義したにすぎない
現代の教科書等の力の定義と同じだ。
0661ご冗談でしょう?名無しさん
垢版 |
2022/10/06(木) 13:45:37.55ID:???
>>659
刷り込まれた任意記号に死ぬまで拘るつづけるのがオマエの劣化した脳だ

しがらみがなく自分が理解しやすいように任意記号を定義できるのが脳が柔軟な証拠。
0666ご冗談でしょう?名無しさん
垢版 |
2022/10/06(木) 20:22:57.79ID:???
しかし毎度飽きもせず俺様物理を発表したがるのはなんなんだろうね
ジジイになっても中二病続いてたらもう一生治らんのかね
0667ご冗談でしょう?名無しさん
垢版 |
2022/10/06(木) 21:10:50.11ID:???
>>666
どこがオレ様物理か? お前のようなボケ老人は教科書燃やして火葬にでも使え

丸暗記用に堕落した物理教科書を捨て、アインシュタインの精神で物理を復活させてるのだよ
0671ご冗談でしょう?名無しさん
垢版 |
2022/10/07(金) 02:49:49.77ID:???
>>668
粘着アスペ症でオマエの脳が可塑化した末期症状だな

それに比べればマトモに式の意味を質問してきた人は将来有望。
0672ご冗談でしょう?名無しさん
垢版 |
2022/10/07(金) 07:24:47.06ID:???
登記の馬鹿の変節

力も進行方向に収縮するのでトルクは発生しない!
  ↓
レバーの静止座標系で回転しないのだから回転しない!
  ↓
縦質量、横質量で力を再定義!(now)
(オリジナルの問題に質点は設定されていない)
0675ご冗談でしょう?名無しさん
垢版 |
2022/10/07(金) 13:11:01.98ID:???
>>673
登記の馬鹿も、4年かけて問題の趣旨を理解できた!
って褒めてんだよ。
解法を理解すんのは当分先、生きてるうちは無理かな
0677ご冗談でしょう?名無しさん
垢版 |
2022/10/07(金) 14:56:26.34ID:???
粘着アスペ症は他人のスレを理解すらできないから解らんだろが
>>650
では新しく定義した力で直角レバーのx,y方向の力をローレンツ変換で計算した結果だから
アインシュタイン論文の縦質量も横質量も計算で使ってないのだよ。
端の質点は元の図を借用しただけ
本筋にもどると
次は、新しい力の定義をつかって運動エネルギーを導出することで
素人でも知ってる式 E = mc2 が誰でも分かるように説明しよう
カミングスーン
0679ご冗談でしょう?名無しさん
垢版 |
2022/10/07(金) 18:04:17.85ID:???
>>650 >>677 のつづき
>特殊相対性理論の登場によって、力の絶対性は否定され定義で変化するものであり、
>静止質量(と電荷)がローレンツ変換の不変量(スカラー)と認識されたのだよ。
>それに基づいた力の定義を書くと (注意:定義だから認めない人は論外)
> dP/dt = F , P = m0V/γ , γ=√(1-(v/c)^2) P:運動量ベクトル F:力のベクトル
> V:速度ベクトル (大文字はベクトルを示す)

m0:(粒子の)静止質量 v:(粒子の)速度の大きさ ・:内積 
この力の定義を基にして 運動エネルギーKの式は dK/dt = F・V = dP/dt ・V
内積してから積分して運動エネルギーKを計算する、速度V=0,K=0の条件により
dK/dt = vdp/dt = vm0/γ^(3/2)dv/dt
K = ∫vm0/γ^(3/2)dv = m0c^2/γ + C
積分定数Cは速度0(静止)で運動エネルギー0 の条件から C = -m0c^2
K = m0c^2/γ-m0c^2 となる

この式を次のように熱力学的に物理解釈する
粒子の 全エネルギー = 運動エネルギーK + 内部エネルギーE
m0c^2/γ = m0c^2/γ-m0c^2 + m0c^2
となる、粒子の内部エネルギーEは粒子の静止質量m0に比例する。
これで有名な E = mc^2 の式の意味が明らかになった。

めでたし めでたし (copyright)
0682ご冗談でしょう?名無しさん
垢版 |
2022/10/07(金) 19:51:20.67ID:???
頭悪いやつばっかだな
粒子同士の非弾性衝突を計算すれば直ぐ解るだろ
熱力学の変換を考えないと全エネルギーが保存されない。
0685ご冗談でしょう?名無しさん
垢版 |
2022/10/07(金) 20:08:09.22ID:???
>>683
>静止質量は熱力学的な内部エネルギーである
物理的に正しい。

内部エネルギーが分子運動以外の力であっても問題無し。
0686ご冗談でしょう?名無しさん
垢版 |
2022/10/07(金) 20:09:35.26ID:???
>>682
なにいってんの?
エネルギーの定義を E=1 ってことにしちまえば絶対保存されるはずだろ。
いっそのこと F も m も a も全部定数 1 にしちまおうぜ。
するとどんなことがあっても無矛盾な物理体系になるじゃないか。
それこそおまえが求めていた理想の物理学だ。
0690ご冗談でしょう?名無しさん
垢版 |
2022/10/07(金) 21:03:04.43ID:???
>>688
お前らの類がいくらE=mc^2の式を丸暗記したところで何一つ理解できないということだ

自分で特殊相対論力学から計算で導出して初めてその物理的意味が理解できるのだよ。
0691ご冗談でしょう?名無しさん
垢版 |
2022/10/07(金) 21:15:35.09ID:???
>>690
E=mc^2 はおまえが丸暗記してる式だろが
力のローレンツ変換しときながらなんでその式だけ頑なに拘ってんだよ
相対論もへったくれもなく好き勝手に式決めるなら E=1 でいいといっとろうが
0692ご冗談でしょう?名無しさん
垢版 |
2022/10/07(金) 21:38:43.38ID:???
コンプトン効果の計算を見れば分かるけど、熱力学なんかまったく関係なく相対論的な計算ができる。静止質量が熱力学的な内部エネルギーなどという珍解釈は全く無用。
0693ご冗談でしょう?名無しさん
垢版 |
2022/10/07(金) 21:49:58.96ID:???
俺が学んだ熱力学はマクロな物理量の間に成り立つ関係を現象論的に構成した理論なんだけど、登記熱力学は全くそれとは違う学問のようだ。
0694ご冗談でしょう?名無しさん
垢版 |
2022/10/07(金) 21:56:53.51ID:???
そらそうだ
式も理論も好き勝手に変えるんだから言葉なんてなおさら好き勝手に変える
そもそも登記が語ってるのが物理の話だと思ってるならその前提がおかしい
0696ご冗談でしょう?名無しさん
垢版 |
2022/10/07(金) 22:46:48.67ID:???
>>692
>コンプトン効果
頭悪いな
電子の静止質量が反応の前後で変わらないから、電子の内部エネルギーは変化しない。
対消滅の反応ならば電子の静止質量が無くなりその内部エネルギーは全て放出される。

内部エネルギーの定義を調べもしないで喚いてるバカが多いから簡単に書くと
(その粒子状態の)全エネルギーから力学的エネルギー(運動エネルギー、外部のポテンシャルエネルギー等)を引いたエネルギーを内部エネルギーという。

その定義を熱力学が最初に使ってるだけで熱力学固有の概念ではない。
0697ご冗談でしょう?名無しさん
垢版 |
2022/10/07(金) 22:48:40.86ID:vIlYCjwe
>>696
>電子の内部エネルギー
0700ご冗談でしょう?名無しさん
垢版 |
2022/10/07(金) 23:06:16.20ID:???
登記はとんでもないデタラメ山盛りにしときながら煙に巻いて
そんなに間違ってるとはいえないところばかりに噛みつくかせる手法がqqqそっくりなんよな
0701ご冗談でしょう?名無しさん
垢版 |
2022/10/07(金) 23:13:16.84ID:???
>>696
横長になったのでもう一度簡単に書くと
(その粒子状態の)全エネルギーから力学的エネルギー(運動エネルギー、
外部のポテンシャルエネルギー等)を引いたエネルギーを内部エネルギーという。

熱力学でなければならない理由はどこにもないが、応用として
E = mc^2 の内部エネルギー定義から、熱した金属の粒子全体の静止質量が僅かに増える
ことが簡単に理解できるだろ。
0703ご冗談でしょう?名無しさん
垢版 |
2022/10/07(金) 23:24:42.52ID:???
>>701
熱力学の内部エネルギーの定義はエネルギー状態の任意性があるが

E=mc^2 の内部エネルギーは静止質量mから座標系と無関係に一意的に決まる。

素晴らしい、アインシュタインは大天才だ!
0706ご冗談でしょう?名無しさん
垢版 |
2022/10/08(土) 00:52:14.27ID:???
>>705
行列を2回掛け算してやればよい。
 t''' = γ^2 ((1 + v^2 /c^2)t' - 2vx'/c^2)
 x''' = γ^2 ((1 + v^2 /c^2)x' - 2vt')
かな、たぶん。
0707ご冗談でしょう?名無しさん
垢版 |
2022/10/08(土) 00:55:31.13ID:???
ちなみに直角レバーのパラドックスについては登記爺は完全なデタラメなので補足しとく。
少し調べた限りではここの説明がいちばん丁寧だと思う。
 https://ikumi.que.jp/blog/archives/364

簡単にいえば相対論では内力によるトルクが0にならない。
それが外力のトルクと打ち消し合って0になるようだ。
0708ご冗談でしょう?名無しさん
垢版 |
2022/10/08(土) 06:35:20.00ID:???
>>707
そのページや参照先の Kevin の解説記事も4年前にリンク貼られてたんだが登記の馬鹿は理解できんかったのよ、英語読めない知障だし。
Laue current は電磁気学のパラドックス解くのにも使われる。
0710ご冗談でしょう?名無しさん
垢版 |
2022/10/08(土) 07:01:11.63ID:???
>>707-709
粘着アスペ症のキチガイだな、こいつの症状は自分で考える能力が無く
記事のURLや式のコピペと貶ししかできない、それを何年も5ちゃんで続けてる。

そのURLの中身も他人の記事うんぬんてるだけで問題を自分で解決できない
粘着アスペかその同類のサイトとしか思えん内容だ。
0713ご冗談でしょう?名無しさん
垢版 |
2022/10/08(土) 07:22:39.28ID:???
>>710
まず考える前に物事を正しく理解することから始めよう
意味もないことを荒唐無稽に考えるだけならサルでもできる
下手の考え休むに似たり
0714ご冗談でしょう?名無しさん
垢版 |
2022/10/08(土) 07:35:31.79ID:???
そもそも「直角レバーのパラドックス」は静止状態が前提だから
慣性系同士で座標をローレンツ変換しても回転運動など起こらない。
くり返しだが
それなら何がパラドックスなのかと言えば特殊相対論の力の定義と
使い方が間違ってるからだ。

その解決法も静止状態なのだから>>650の図のような最も簡単な、
質量0に近い剛体フレームのモデルで力を適用すればいいだけだ
つまり、直角フレームが外力で歪まず仕事もしない。

ふつう力学の演習問題では当たり前で振り子の糸は力で伸びず直線を保つ
力で問題のブロックを押しても変形しないのが当たり前だ。
馬鹿がゴム紐の糸や、コンニャクでは運動が違いだろなどと喚いても
ニュートン力学の3法則が間違ってるからだなどと言うのは馬鹿だけ。

だから複雑な弾性体の直角レバーを云々したところで特殊相対論が間違いには
ならんのだなよ。

>ラウエ 等が何故わざと複雑な弾性体に拘った理由は俺には簡単に推測できるが
馬鹿のアスペにはわからんだろ
0715ご冗談でしょう?名無しさん
垢版 |
2022/10/08(土) 07:36:57.08ID:kp3ajx/V
>>713
>荒唐無稽に考えるだけならサルでもできる
そんなサルはいないと断言するよ。
0717ご冗談でしょう?名無しさん
垢版 |
2022/10/08(土) 07:47:06.97ID:???
>>714
>ラウエ 等が何故わざと複雑な弾性体に拘った理由
答えは簡単
アインシュタイン論文がニュートン力学と同じに力を不変に取り、「縦質量」「横質量」
で定義してしまったからだ!

その定義では、変形しない剛体フレームの直角モデルが使えない!
0718ご冗談でしょう?名無しさん
垢版 |
2022/10/08(土) 08:27:24.40ID:???
>>717
馬鹿、相対論的には剛体は存在できないからローレンツの解法は合ってんだよ。
それと、ラウエは Laue current を用いたのであって弾性体としては論じてない。
登記は英語読めない馬鹿だもんな腹イテェ
0719ご冗談でしょう?名無しさん
垢版 |
2022/10/08(土) 08:33:33.44ID:CHN90hCw
>>718
>相対論的には剛体は存在できない
そのうそほんと?
0721ご冗談でしょう?名無しさん
垢版 |
2022/10/08(土) 09:04:12.99ID:???
>>717
> アインシュタイン論文がニュートン力学と同じに力を不変に取り、「縦質量」「横質量」

登記爺が勘違いしてる根本のところはそこだな。
だから力についてわけのわからん勘違いをしてわけのわからん計算をしてる。

「縦質量」「横質量」というのは力を加え続ければ加速度が遅くなるのは、
質量が増えるからと考えるか、空間の性質と考えるかの違いだけだ。
どちらも力の考え方はまったく変わらない。

観察する座標系が異なれば力の大きさは当然変わる。
それは力のローレンツ変換の式で示される。
座標変換しても力が不変だなどと馬鹿なことはアインシュタインも言ってない。
0722ご冗談でしょう?名無しさん
垢版 |
2022/10/08(土) 09:28:31.69ID:???
>>714
> それなら何がパラドックスなのかと言えば特殊相対論の力の定義と
> 使い方が間違ってるからだ。

> 特殊相対論が間違いにはならんのだなよ。

登記本人がまだ自分のスタンスの矛盾に気付いていなさそうだから言うが、
客観的には、おまえはまぎれもなく「相対論は間違ってる」側の人間やで
0724ご冗談でしょう?名無しさん
垢版 |
2022/10/08(土) 13:26:36.08ID:???
>>720
>ローレンツの解法はアインシュタインの論文の前だぞ
1905年以前ということになる

粘着アスペによればラウエの論文は1911年
ラウエはアインシュタインの熱烈な支持者であるからアインシュタイン論文の
力の定義を変えるはずもない。

以上から物理学史で考察すればニュートン以来300年の間、力は慣性系で不変的な
物理量として定義され、現実の力もその定義が正しいと信じていたのだよ。
1905年当時の物理学会と殆どの物理学者がニュートンの定義を信じているのだから
アマチュアのアインシュタインの投稿論文を読んでもらうためには、ニュートンの定義を
使うしかなかったのだ。

アインシュタインがニュートン力学の力の定義を絶対だと考えていないことは、
アインシュタイン論文のF=ma定義と縦質量、横質量の式の直ぐ後にわざわざ
”もし力と加速度の定義を変えれば、これと違った質量の値を得るのは当然である
・・・いろいろな理論を比べる場合非常に注意すべきだ”
と記述している事実からも判る

さらに、注釈:で”このような力の定義は(相対論では)便利ではない
運動量とエネルギーの法則が最も簡単な形をとるように力を定義するのが適切である”
と付け加えている。
”便利ではない”の具体例が物体が静止状態なのに「剛体」モデルが使えないことなのだよ。

それではいつごろから現在の「(不変な)静止質量を基にした力の定義」の教科書が主流
になったのか?
知ってる人いるかな
0725ご冗談でしょう?名無しさん
垢版 |
2022/10/08(土) 13:41:34.28ID:???
>>724 補足
>便利ではない”の具体例が物体が静止状態なのに「剛体」モデルが使えない

力は物体が静止していても作用させることができる。力の釣り合いなどで誰でも使ってる
剛体モデルならば作用反作用が自明であるから応力を考える必要もない。

ところが、縦質量、横質量なるものはその方向の運動による加速度が前提なのだよ!
つまり、剛体モデルのつり合いでは変形しないから縦質量、横質量の作用が記述できない。
0729ご冗談でしょう?名無しさん
垢版 |
2022/10/09(日) 00:19:48.72ID:???
縦質量、横質量の定義が廃れたと同時に100年前のレバーパラドックスもお蔵入り
アスペ馬鹿と相間がほじくり出して喚いてるだけ。
0731ご冗談でしょう?名無しさん
垢版 |
2022/10/09(日) 00:26:35.53ID:oR5W8mxZ
エネルギーが質量なら増えるはず。
しかし、増えないのは相対性理論が間違ってるから。
それだけのこと。
0733ご冗談でしょう?名無しさん
垢版 |
2022/10/09(日) 10:23:29.28ID:???
>731
>エネルギーが質量なら増えるはず。しかし、増えないのは相対性理論が間違ってる

この類の特殊相対論が間違い説は、物理理論の原理と定義を知らない理解できない人
”相対論では運動方向の長さが短くなる”だけ聞くと、高校物理レベルの一般人ならば
単純化した思考実験で
 ・――・
 |  ↑F
 | v→
 |
 ・←F
の図から自明なように右方向に回転してしまうと考える。
なぜなら、高校物理はニュートン力学で正しく学習出来た人なら、
力ベクトルは並進速度vで運動しても力F変わらないからだ。(試験問題では正しい)

相間サイトによれば特殊相対論の結果と称して←の力の大きさは変わらないが、
↑の力は小さくなると計算している。
右回転が増強されるから”特殊相対論が間違ってる”説の根拠にしている。
(アスペ馬鹿はさらに複雑な問題にほじくり返してアスペ発症荒らし)
0735ご冗談でしょう?名無しさん
垢版 |
2022/10/09(日) 10:48:59.10ID:oR5W8mxZ
かくて無能の空い時間が無駄に過ぎゆくのであった
0736ご冗談でしょう?名無しさん
垢版 |
2022/10/09(日) 11:00:57.32ID:???
>>734
>↑の力は小さくなると計算している。

お前もそこだけしか見ない奴と同じだ
>←の力の大きさは変わらない
(相対論計算?)と主張する相間サイトの説を鵜呑みにしてる

マトモな脳の人ならニュートン力学で変わらない力
を相対論では”力が変る”と変更したのだから
”←の力”も変わってしかるべし、釣り合ってるはずだと推論する。
相間サイトの相対論計算?が間違ってると判断できるのだよ。
0737ご冗談でしょう?名無しさん
垢版 |
2022/10/09(日) 11:25:15.36ID:???
>>736

↑の力は小さくなる
←の力は変わらない

これらは相間サイトの説などではなく、疑いようもない相対論の主張だ。
0738ご冗談でしょう?名無しさん
垢版 |
2022/10/09(日) 11:28:45.73ID:???
>>737
>↑の力は小さくなる
>←の力は変わらない
>これらは相間サイトの説などではなく、疑いようもない相対論の主張だ。

相間サイトのインチキ計算を鵜呑みにする馬鹿(相間本人か)のスナップショット
0742ご冗談でしょう?名無しさん
垢版 |
2022/10/09(日) 11:50:41.66ID:???
>>471
特殊相対論の力の定義は現代の教科書を調べれば同じだと解る。

それを基に正しい力のローレンツ変換を自分で計算出来た人のみが解決できる
同時に運動エネルギーを導出してE=mc^2のゴールに到達できるのだよ。

凡人が微積分の計算を間違わずに一発で正解できると思うなよ、諦めない事だ。
0743ご冗談でしょう?名無しさん
垢版 |
2022/10/09(日) 11:58:23.50ID:???
>>742

ソースを示せないわけだな。
結局お前は、自分の勘違い相対論を広めようとしているだけだ。
害悪だから止めてくれ。
0745ご冗談でしょう?名無しさん
垢版 |
2022/10/09(日) 12:12:15.39ID:???
>>743
お前のようなコピペしかできない馬鹿で怠け者には自分の頭で解決不可能なだけだ。
消えろ

微積分演習とA4ノート何ページも計算デバックを繰り返えした者が解決できる。
0746ご冗談でしょう?名無しさん
垢版 |
2022/10/09(日) 12:19:17.16ID:???
>>744
γ記号が気に入らないなら、自分でγ'=1/γ に置き換えればいいだけだけ
それすら出来ないオマエは馬鹿怠け者=粘着アスペ
0747ご冗談でしょう?名無しさん
垢版 |
2022/10/09(日) 12:22:21.43ID:oR5W8mxZ
我慢の限界というものが
0748ご冗談でしょう?名無しさん
垢版 |
2022/10/09(日) 12:29:33.96ID:???
自分の頭で数式の微積分を何回も計算する人なら(分数)より簡単な式に置き換えたほうが
計算し易くデバッグし易い。
コピペしか出来ないアスペ馬鹿にはそれすらワカラナイ
0749ご冗談でしょう?名無しさん
垢版 |
2022/10/09(日) 12:39:36.86ID:???
泥臭く計算するの嫌で手を汚したくない数学馬鹿の人は抽象的な数学理論に逃げる。

それは物理学というより応用数学だな。
0755ご冗談でしょう?名無しさん
垢版 |
2022/10/09(日) 15:41:49.82ID:oR5W8mxZ
相対性理論
0759ご冗談でしょう?名無しさん
垢版 |
2022/10/09(日) 18:54:56.68ID:???
教科書に書いてあることを相間サイトに書いてあることだから間違いだと断じ、
教科書に書いてない自分の間違い説を教科書に書いてあることと同じだから正しいと訴える。

本当に馬鹿
0760ご冗談でしょう?名無しさん
垢版 |
2022/10/09(日) 19:07:45.51ID:???
5ちゃん馬鹿ども には >>650 が理解不能らしいが
(現代の)特殊相対論の定義では静止した座標系から観測した物体(粒子)の
運動量はmvではない。
始めから(相対論の)係数が掛かっているのだよ!

そのように定義した運動量を静止座標系の時間で微分したのが(現在の)力の定義である。
後から、他の慣性系(の速度)にローレンツ変換してその座標系での力を計算する
ということだ。
それを区別し、かつローレンツ変換の計算が出来る人がパラドックスを解決できる。

つまり、相間サイトは”力の定義”も理解できずに式だけコピペしてインチキ計算し
お決まりの”相対論は間違っている”と喚いてるだけ。
5ちゃんの馬鹿共も同じだ
0761ご冗談でしょう?名無しさん
垢版 |
2022/10/09(日) 19:42:17.51ID:???
相間サイトの極め付きは
”x 方向の力 Fx は ローレンツ変換で不変である”

相間の計算?を大前提にして以下次々と辻褄合わせをやり続けるのが笑えるが
そもそも前提が間違ってるのだから一致するわけがない。

挙句の果てに
”直角レバーのパラドックスに 解決策はなく、QED や 標準模型などの相対論的な理論は 間違いということになる。”
(笑うしかない)
0762ご冗談でしょう?名無しさん
垢版 |
2022/10/09(日) 19:58:42.54ID:???
>>761
>相間サイトの極め付きは
>”x 方向の力 Fx は ローレンツ変換で不変である”

(笑う)だけではつまらんので馬鹿でも分かるように論破すると
現代の特殊相対論の(粒子の)質量の定義は静止質量m0でローレンツ変換で不変であるから

相間の”x 方向の力 Fx は ローレンツ変換で不変である” なら簡単に光速を超えてしまう。
0763ご冗談でしょう?名無しさん
垢版 |
2022/10/09(日) 20:04:04.72ID:oR5W8mxZ
>>762
>質量の定義は
E=Mc^2 ですよ
0765ご冗談でしょう?名無しさん
垢版 |
2022/10/09(日) 20:35:42.67ID:???
>>762
具体的にいえば 電子の静止質量は 9.1x10^-31 kg である。

粒子加速器で静止系からみた電子の速度が光速の99.999% でも質量は9.1x10^-31 kg
のままということだ。
0766ご冗談でしょう?名無しさん
垢版 |
2022/10/09(日) 20:54:44.15ID:oR5W8mxZ
またまた出たら目を、だから良い子は相対論は信じちゃいけないのよ、
0767ご冗談でしょう?名無しさん
垢版 |
2022/10/09(日) 21:41:25.75ID:???
教科書に書いてあることを間違ってるって、登記爺の中で正しいことって一体なんやねん
まさに >>669 のコメが刺さる

逆走老人はジョークネタかと思ってたが、
世間の逆走老人て、心の底からほんとに自分以外全員逆走してると思ってるんかもしれんな
0768ご冗談でしょう?名無しさん
垢版 |
2022/10/09(日) 21:53:17.73ID:???
>>767
>教科書に書いてある

お前のいう教科書とは100年前の虫食い本の類だろ火葬の薪にでも使え
質量が増えるとか混乱の元にしかならんから、今では誰も使わない。
0769ご冗談でしょう?名無しさん
垢版 |
2022/10/09(日) 21:59:22.39ID:???
>>768
もちろん縦質量横質量なんて俺らは使わんよ
このスレで縦質量だの横質量だの言い始めたのは登記爺だ
どうも言ってることが支離滅裂だが認知症か?
0770ご冗談でしょう?名無しさん
垢版 |
2022/10/09(日) 22:12:42.20ID:???
あっ、読み返してみたら登記爺の呆けポイントに合点がいった。
登記爺は「おまえらが使ってるのは、縦質量、横質量を前提にした間違った力の変換式だ。
俺が正しい力の変換式に書き直してやったぞ」って話だな。

呆けてるにしても方向性が異次元すぎて推察に苦労するわ
0772ご冗談でしょう?名無しさん
垢版 |
2022/10/09(日) 22:20:30.78ID:???
>>771
おまえ自分で dP/dt = F って書いてただろ。
なんでそれで光速超えると思うんだよ。

Pは運動量のつもりじゃないんか?またおまえの独自定義か?
0776ご冗談でしょう?名無しさん
垢版 |
2022/10/10(月) 08:41:13.77ID:3wq9kzNf
>>1
動くと距離が短くなってしまうから、
光速を守りために時間が遅くなる、
ということだろうが、
時空が縮んで光速一定というのが、
わからない、、、
0777ご冗談でしょう?名無しさん
垢版 |
2022/10/10(月) 08:42:43.76ID:ISfpUQHb
時空はゴムでできてると考えるといいらしいぞ。おれは信じないけど
0778ご冗談でしょう?名無しさん
垢版 |
2022/10/10(月) 11:06:47.15ID:???
>>776
>時空が縮んで光速一定というのが
よくある間違い

特殊相対論による光速不変の原理では慣性系の観測者からみた時空は変化しない。
変化しない時空座標に対して、並進運動する物体の長さ縮む時間が遅れて観測される。

という意味だから、運動する物体自体の物理状態は何も変化しない。
0779ご冗談でしょう?名無しさん
垢版 |
2022/10/10(月) 22:31:25.46ID:???
>>776
遠ざかる光も、近づいてくる光も一定なんだよ
単純に時間の伸び縮みでは説明できないでしょ
でもそれを完全に成立させるのが相対論
0780poem
垢版 |
2022/10/11(火) 00:16:06.04ID:???
>>779がいいこと言ってる
779を要約すると
・相対性理論は時間空間の伸び縮みの理論(ローレンツ変換とかいうのに従い伸び縮みする)

779・その説の根幹、その説の全て、"全ては光速度不変""光が観測者の速度によらず定速で届く"、これだけを扱った理論。これだけを物凄く怪奇な理論にした。

・"光速不変"と"観測者の速度"のズレを同時に扱った結果生じるズレを"空間と時間の伸び縮み"と言う結論にしたのが相対性理論。これがローレンツ変換に従う。つまりローレンツ変換とは定速と変速の物理を変換してズレを計量する方法

・相対性理論は定速と変速の物理を変換してズレを計量するという理論。これにより計量されたズレを空間と時間の伸び縮みと主張している。

もし・"光速不変"が、あるいは"定速と変速"が実は空間と時間の伸び縮みによらない物理なら、空間と時間が伸び縮みしてるから光が"光速不変になる"または空間と時間が伸び縮みしてるから光と物体が"定速と変速の変換になる"のでない、相対性理論の結論がミスリードなら

・相対性理論は間違い。ただ光速不変は光を含む量子が慣性系に従わないだけの、独自の物理で、ローレンツ変換の必要はなく、ローレンツ変換が誤り、である
0781poem
垢版 |
2022/10/11(火) 00:31:21.28ID:???
Q問
光速度不変は
1.光の速度不変と観測者の速度可変の変換が時間空間の伸び縮みを発生するから、光は速度不変。相対性理論で計算される空間時間の伸び縮みが光速度不変を起こす原理
2.光を含む量子は慣性系に従わないから光速度不変である。相対性理論で計算される空間時間の伸び縮みで光が速度不変になるのでなく、量子の効果、量子の物理であり、空間時間には関係ない。光速不変は量子の物理が発生する原理

相間/相信、あなたはどちら?
0783poem
垢版 |
2022/10/11(火) 00:35:04.87ID:???
光は…マクロに現れる量子物理
量子物理は…マクロの慣性系に従わない
相信なら…ミクロの量子物理は慣性系に従うと主張する
0784poem
垢版 |
2022/10/11(火) 00:43:54.00ID:???
>>782
まあ、再三詳しく言ってるけど、相対性理論信じてるの揺らがないから、再四言っても無駄だろうし、また書いても何度も書いてるのと同じこと書くだけだから、別に今も779がいいこと言ったから、つい書いた数レスで言いたいこと充分表現したレス、発作で書いてしまっただけだし、言いたいこと充分表現したから、再四同じこと書くの面倒くさいし、再三まで書いて無駄だったからまた書いても無駄だから、これ以上今のところ書くつもりない。反応してツッコミはするかもだけど、というか面白いこと書かれたらする、いつもどおり
0785ご冗談でしょう?名無しさん
垢版 |
2022/10/11(火) 00:47:50.57ID:hNCfCFEg
要するに相対論と量子論は両立しないのである
0786ご冗談でしょう?名無しさん
垢版 |
2022/10/11(火) 00:54:32.36ID:???
特殊相対性理論と量子力学は無矛盾に両立しており、場の量子論という極めて予測精度の高い理論が出来上がっています。
0787ご冗談でしょう?名無しさん
垢版 |
2022/10/11(火) 01:39:23.05ID:???
>>778
特殊相対性理論で時空間が変化するという説は、質量が変化する説と同じ古い解釈だ
慣性系同士で観測量が変るだけ。
その簡単な理由は物体の長さ時間が静止状態と違って観測されるだけだから、物体自体が
何の物理作用も受けないことから解る。(レバーのパラドックスもその一つ)

それならば真の時空間の変化とは何か、それが巨大質量による時空間の歪で重力作用を伴う。
光速も不変ではなくなり、光線は曲がる。
地上の全ての物体は主に地球質量による時空間の変化が原因の重力の物理作用を受け
人間生活の全てに影響を及ぼしている。
これが真の時空間が変化したという物理結果でありその中に我々人間が居るのだよ。
0789ご冗談でしょう?名無しさん
垢版 |
2022/10/11(火) 07:55:05.49ID:???
>>762
>相間の”x 方向の力 Fx は ローレンツ変換で不変である” なら簡単に光速を超えてしまう。
お前は、ちょっと前まで Fx をレバーと同じローレンツ収縮させてたろ。
変節した理由は?
0790ご冗談でしょう?名無しさん
垢版 |
2022/10/11(火) 09:24:22.26ID:???
>>788-789
粘着アスペ症の鳥頭か

レバーのパラドックスはとっくに俺が解決した、オマエはアスペ症でそのスレも読めず
理解できないアホ
他のマトモな頭の人はとっくに理解している。

>>相間の”x 方向の力 Fx は ローレンツ変換で不変である” の部分がアスペ症が推薦するデタラメ説

>なら簡単に光速を超えてしまう。
現代の定義では物体の質量が変化しないのだから、馬鹿でも分かる簡単な結論だ
アスペ症のアホには理解不能らしい

>Fx をレバーと同じローレンツ収縮させてた
アスペ症アホのデタラメ

アホは現代の力の定義を俺が書いてやってるのに理解できない、
定義からx方向の力←の正しいローレンツ変換をした式も理解できないアホ
'長さのローレンツ収縮’と式が違う ことが見えないアスペアホ

アスペ症末期のおまえは精神病院に行くしかない、さっさと消えろ。
0791ご冗談でしょう?名無しさん
垢版 |
2022/10/11(火) 10:16:25.57ID:???
>>787
>物体自体が何の物理作用も受けないことから解る。(レバーのパラドックスもその一つ)

慣性系同士のローレンツ変換はガリレイ変換と同様に物体自体には何も物理作用が起こらない。
それを前提として認めれば、直角レバーが回転しないのは自明なのだよ。
つまり、相間サイトのFxのようにデタラメな力の計算をしてパラドックスモドキになるだけだ。

科学史的に直角レバーパラドックスの原型であるトルートン・ノーブル実験が大真面目で
実際に実施されたのか?  アホには解らんだろが

当時はエーテル説が物理学会の主流だったので地球がエーテル中を相対運動したときに
エーテルによる物理作用が起こるはずだと信じていたからなのだよ。
0793ご冗談でしょう?名無しさん
垢版 |
2022/10/11(火) 11:20:26.25ID:???
>>791
>当時はエーテル説が物理学会の主流だったので地球がエーテル中を相対運動したときに
>エーテルによる物理作用が起こるはずだと信じていたからなのだよ。

マイケルソン・モーレーの実験は、マイケルソンがエーテル説によればエーテルとの物理作用
で干渉計の干渉縞が変化するはずだと信じていたから実験を実施した。

干渉縞が変化しない事実を説明する為にローレンツとフィッツジェラルドが
フィッツジェラルド‐ローレンツの仮説(式は相対論のローレンツ収縮と同じ)
を発表したがその長さの収縮も、エーテルとの物理作用によると信じていたのだよ。

直角レバーパラドックスの原型であるトルートン・ノーブル実験結果がフィッツジェラルド
‐ローレンツの仮説だけでは説明できないことはこのスレで散々説明した通りだ。
0796ご冗談でしょう?名無しさん
垢版 |
2022/10/11(火) 20:31:34.69ID:???
レバーバラドックスは Laue current を使って解決するのが正しい理解(または標準的な理解)だということでいいのかな?
Laue current は中々取っつきにくい概念だけど。
0798ご冗談でしょう?名無しさん
垢版 |
2022/10/11(火) 20:41:56.74ID:???
そんなググっても出てこんようなものなど知らん
登記のようにデタラメでなければなんでもいいから好きにせい
0799ご冗談でしょう?名無しさん
垢版 |
2022/10/11(火) 20:42:53.88ID:w63WenOc
迷ったときには原点に帰れ、これは人類普遍の原理ですよ
0800ご冗談でしょう?名無しさん
垢版 |
2022/10/11(火) 20:46:04.33ID:???
>>796
エネルギーの流れが運動量を持つ、っていう至極当たり前の解法。これにケチをつけたのは登記の馬鹿とqqqだけ。
0801ご冗談でしょう?名無しさん
垢版 |
2022/10/11(火) 20:47:46.98ID:???
現代特殊相対論の力の定義と静止剛体モデルで簡単に解決できるのに
痴呆爺は消化できないらしい
0803ご冗談でしょう?名無しさん
垢版 |
2022/10/11(火) 20:58:28.46ID:w63WenOc
>>800
>エネルギーの流れが運動量を持つ
質量の流れの間違いでしょ
0805ご冗談でしょう?名無しさん
垢版 |
2022/10/11(火) 21:01:02.47ID:???
>>801
痴呆爺には消化不良だろが、今の学生なら簡単すぎる問題に還元できる

はじめに静止座標から見て力Fが質点に作用している
 F→・
左に速度-v で運動する座標系から見てFがどう変わるか?
 F→・→v
という簡単な問題に還元できるのだよ

相間サイトの相間はFは不変だと相対論で計算?したと主張している。
0809ご冗談でしょう?名無しさん
垢版 |
2022/10/11(火) 21:13:21.08ID:???
>>807
相間説とはどこまでを指す?
レバーパラドックスの概要までか、そこから導き出された(だから相対論は間違っているという)結論までか
0810ご冗談でしょう?名無しさん
垢版 |
2022/10/11(火) 21:13:46.71ID:???
>>806
頭悪いな
左方向に相対運動する座標系から見れば質点・は右にvで動いてるだろが
作用してる力が変わるのか? 変わらないのか? という簡単な問
0813ご冗談でしょう?名無しさん
垢版 |
2022/10/11(火) 21:18:03.02ID:w63WenOc
>>808
そのままおかえしいたしますわ。おほほほほ
0814ご冗談でしょう?名無しさん
垢版 |
2022/10/11(火) 21:18:43.33ID:w63WenOc
>>812
>エネルギーは質量を含むんだよ
ノーベル賞?
0817ご冗談でしょう?名無しさん
垢版 |
2022/10/11(火) 21:29:29.84ID:???
>>810
ニュートン力学の理論なら当然、質量は不変で力も不変となる。

現代の特殊相対論の定義ならば、ローレンツ変換では質量(=静止質量)が不変である。
そこまでの前提だけでも、作用してる力が変るしかないとマトモな脳なら考える。
0818ご冗談でしょう?名無しさん
垢版 |
2022/10/11(火) 21:34:08.47ID:???
>>818
相間サイトを見てみ、相間がコピペしたいろいろの辻褄合わせ法を紹介している
その後でお決まりの”相対性理論は間違ってる”と結論
0821ご冗談でしょう?名無しさん
垢版 |
2022/10/11(火) 21:53:56.96ID:???
つうか、ひょっとして君は、その辻褄合わせ法が間違っているという相間の主張に共感してるのかよ?
0822ご冗談でしょう?名無しさん
垢版 |
2022/10/11(火) 22:27:49.20ID:???
>>821
古い力の定義で、後から辻褄合わせを考え出すより

現在の相対論の定義からx,y,z方向の力を同時に計算した方が簡単
そんだけ
0823ご冗談でしょう?名無しさん
垢版 |
2022/10/11(火) 22:36:46.09ID:???
>>822

定義に古いも新しいもない。定義を変えることで5Nの力を3Nに変えたとして、
それで実際の現象としての力の大きさも変わると思っているのか?

定義を変えることで現実から乖離した辻褄合わせをしているのは君の方だと知るべき
0824ご冗談でしょう?名無しさん
垢版 |
2022/10/11(火) 23:02:40.19ID:???
>>823
>定義を変えることで現実から乖離した辻褄合わせ
オマエは馬鹿だとしか言えない

現代の特殊相対論の質量と力の定義が妄想らしい
0827ご冗談でしょう?名無しさん
垢版 |
2022/10/11(火) 23:12:35.55ID:???
運動方程式F=γmaがあれば、力の大きさはFであってF/γではない。
こんなことも理解できない馬鹿はいらない。
0828ご冗談でしょう?名無しさん
垢版 |
2022/10/11(火) 23:43:52.55ID:w63WenOc
それが力の定義?
0829ご冗談でしょう?名無しさん
垢版 |
2022/10/12(水) 00:02:29.78ID:???
>>828
>それが力の定義?
他の誰でも疑問に思う罠

>>827 がオレ様定義したのが >運動方程式F=γma  だということだ

現代の特殊相対論による質量と力の定義とは全く違ってる。
0830ご冗談でしょう?名無しさん
垢版 |
2022/10/12(水) 00:41:35.23ID:???
>>827
>運動方程式F=γma
オマエの力のローレンツ変換の式らしいが、

単純に因子γを付けただけで力のローレンツ変換だとでも、勘違いしてるのか?
0832ご冗談でしょう?名無しさん
垢版 |
2022/10/12(水) 01:19:59.32ID:???
>>805

馬鹿なお前に問題を出すぞ
これを解いてお前の期待する結果を出してみろ

①質量mの質点に一定の力Fをかけ続ける
 この質点が描く世界線の式をどれでもいいから1つ示せ

②Fと逆方向に速さ-vで動く慣性系から見たときの質点の世界線の式を求めよ
 つまり①の質点の世界線をローレンツ変換せよ

③②で求めた世界線から、質点に働いている力の大きさを求めよ
 これで力Fを別の慣性系から見たときに、Fがどう変化するか分かる
0834ご冗談でしょう?名無しさん
垢版 |
2022/10/12(水) 05:42:04.57ID:???
登記の馬鹿は自己レスしまくり。何考えてんだか。

>>824
> 現代の特殊相対論の質量と力の定義が妄想らしい
お前のデタラメ定義が妄想な。

>>826
>現代の相対論の質量と力の定義は4次元空間の座標変換を基にして定義されているのだよ
で、お前は未だに4元力をローレンツ変換出来ない馬鹿なわけだ。
0835ご冗談でしょう?名無しさん
垢版 |
2022/10/12(水) 06:23:32.40ID:???
>>805
>  F→・→v
> という簡単な問題に還元できるのだよ
>
> 相間サイトの相間はFは不変だと相対論で計算?したと主張している。

正しいけど?
登記は計算出来ない馬鹿だもんな
0836ご冗談でしょう?名無しさん
垢版 |
2022/10/12(水) 06:33:47.69ID:???
電磁気力と特殊相対性理論 戸田昭彦
https://home.hiroshima-u.ac.jp/atoda/Electromagnetism/23EM-Relative.pdf
p.10
1) x 軸方向に直線運動する物体では, F・V = F ₓV ₓ なので,
 Fₓ’ = Fₓ
となり,進行方向成分の外力は座標系に依らない。

進行方向成分の外力は座標系に依らない。
進行方向成分の外力は座標系に依らない。
進行方向成分の外力は座標系に依らない。
0837ご冗談でしょう?名無しさん
垢版 |
2022/10/12(水) 07:58:00.83ID:???
>>836
馬鹿は同じことを3回繰り返す

馬鹿は間抜だから、抜けているので付け加えてやると
速度 V で運動している物体の質量は γだけ重い。

今でも、110年前の運動で質量が増える! 定義を使ってる奴がいるらしい。
(参考書)太田浩一
0840ご冗談でしょう?名無しさん
垢版 |
2022/10/12(水) 08:12:36.27ID:???
>>838
そこの1ページ目に(参考書)太田浩一
今でも、110年前の運動で質量が増える!!!定義を使ってる奴がいる
0841ご冗談でしょう?名無しさん
垢版 |
2022/10/12(水) 08:31:22.59ID:???
(参考書)太田浩一「電磁気学の基礎II」東京大学出版会, 2012

東京大学もこんな古い時代遅れの本を出版してるようじゃ地に落ちたな
0842ご冗談でしょう?名無しさん
垢版 |
2022/10/12(水) 08:42:19.31ID:QAsT7dAP
灯台に逆らうとはええ度胸やな。どっちにしろ相対論は間違っとるがのう。
0844ご冗談でしょう?名無しさん
垢版 |
2022/10/12(水) 09:13:15.86ID:???
>>839
俺も5年前くらいまではアインシュタイン論文の定義が正しいと信じてたが
理由は電場ベクトルEをローレンツ変換するとEx'=Ex となるからだ。

その定義だと、直角レバーのパラドックスが簡単に解けない、辻褄合わせの方法も怪しい。

良く考え直してみると、空間に有る電場ベクトルEは力ではない!
電荷の有る質量に作用した状態で始めて力となるのだ。(つまり力の定義で変わる)

つまり、質量(=静止質量)を不変量にした力の定義の方が便利であり、直角レバーのパラドックスが簡単に解ける。
頭のいい学生ならば、始めから質量(=静止質量)不変の特殊相対論の教科書を使えば
試行錯誤せずに済むのだよ。
0849ご冗談でしょう?名無しさん
垢版 |
2022/10/12(水) 11:48:03.89ID:QAsT7dAP
はっきり言おう。全部間違い。どうだ、勝った勝った
0852ご冗談でしょう?名無しさん
垢版 |
2022/10/12(水) 13:22:32.82ID:???
登記の馬鹿頭で理解出来ないから間違い!
で、トンデモ理論でデタラメ計算する。
やってることはqqqやポエムと同じ。
0853ご冗談でしょう?名無しさん
垢版 |
2022/10/12(水) 13:33:35.09ID:???
>>847
日本人の先生が書いた教科書はニュートンの形式から構成してるのが多い
後は曖昧な定義か両論併記的なまま終わってしまう。
(悪く言えば無味乾燥の本)
後は自分で調べるんだな
0854ご冗談でしょう?名無しさん
垢版 |
2022/10/12(水) 13:41:27.46ID:???
>>853

この手の質問をしたらいつも逃げる
こういう手合いは教科書を読めないから出せるわけないというのが真相だろうな
0855ご冗談でしょう?名無しさん
垢版 |
2022/10/12(水) 13:57:20.65ID:???
>>853
電磁気学の教科書を例にすれば

最初にクーロンの法則を覚え込ませて、最後にマックスウェル方程式(基本法則)が出て来る
ところが最後まで消化できる学生は少ない

普通の学生はクーロンの法則が基本原理だと信じてるから、あとから静電気でしか成り立たない
と言われてもそれで計算しようとするだろ。
0856ご冗談でしょう?名無しさん
垢版 |
2022/10/12(水) 14:12:13.23ID:QAsT7dAP
悲惨亜授業を受けてたんだね
0857ご冗談でしょう?名無しさん
垢版 |
2022/10/12(水) 14:49:57.61ID:???
>>800
>エネルギーの流れが運動量を持つ、っていう至極当たり前の解法。

この解法を使う例をレバーパラドックス以外でいくつか教えて欲しい
0858ご冗談でしょう?名無しさん
垢版 |
2022/10/12(水) 18:04:20.88ID:???
>>800 >>857
>エネルギーの流れが運動量を持つ
それ自体は物理的におかしくない

釣り合ってる物体(直角レバー)の辻褄合わせに使う方法が汚いだけ

簡単に言えば慣性運動で物体内部にエネルギーの流れがある? 
そんな物理解釈が認められないだけだ。
0860ご冗談でしょう?名無しさん
垢版 |
2022/10/12(水) 18:19:14.98ID:???
>>859
オマエのような馬鹿は

ローレンツ変換では、物体に何も物理変化が起こらない。

と何度も書いてるのにまったく理解できなようだ。
0866ご冗談でしょう?名無しさん
垢版 |
2022/10/12(水) 21:34:50.14ID:???
数式書きたいなら書いてもいいが、2、3行で済む範囲にしとけよ
数式だけで100レスも埋められたらそれこそ荒らし行為だ
ここに書かんでいいから数式が書いてあるサイトを参照させろよ
0868ご冗談でしょう?名無しさん
垢版 |
2022/10/12(水) 22:49:30.14ID:???
>>867
>静電場と静磁場からなる系のPoyinting vector

辻褄合わせのためにエネルギーの循環があるなどと解釈するのが間違いだ
0869ご冗談でしょう?名無しさん
垢版 |
2022/10/13(木) 06:15:28.07ID:???
>>868
馬鹿は引っ込んでろって
こんな馬鹿なレス↓してる知障なんだからよ

https://rio2016.5ch.net/test/read.cgi/sci/1605951072/956
言葉だけでは解らない人の為に
レバーのパラドックスを簡単に書くと直角レバーの端にの力の釣り合い
       ↓F
   ――――
  |
  |
  |
 F→
速度→vのローレンツ変換(収縮)で以下の図となりレバーが回転する?パラドックス
     ↓F
   ――
  |
  |
  |
 F→

このパラドックスを以下の図のように書き直すとレバー端の電荷qのローレンツ力の釣り合い
       ↓E
   ――――q
  |
  |
  |
 E→q
になる。 後はこのスレのように電磁場のローレンツ変換でローレンツ力を計算すれば
簡単に力のモーメントが釣り合うことが分かる。 (登記)

ついでに、一般の力のローレンツ変換まで解るから相対論初心者には一石二鳥。
やって見れ。
0873ご冗談でしょう?名無しさん
垢版 |
2022/10/13(木) 11:54:10.97ID:???
>>868
>静電場と静磁場からなる系のPoyinting vector
>辻褄合わせのためにエネルギーの循環があるなどと解釈するのが間違いだ

5ちゃん馬鹿どもにはこれが読めんのか
ポインチングベクトルの正しい物理解釈は閉局面全体で計算したエネルギーの流れだ
静電場と静磁場のケースでは0でエネルギーの出入りは無い。

類推から、直角レバーの場合も同様に全体でのみ正しい物理解釈ができる
俺は天才かも
0875ご冗談でしょう?名無しさん
垢版 |
2022/10/13(木) 13:10:35.56ID:???
>>872
しかも5年前に気付いたとか嘘ついちゃう。
登記はマジキチよ

>>844
「俺も5年前くらいまではアインシュタイン論文の定義が正しいと信じてたが
理由は電場ベクトルEをローレンツ変換するとEx'=Ex となるからだ。

その定義だと、直角レバーのパラドックスが簡単に解けない、辻褄合わせの方法も怪しい。」
0876ご冗談でしょう?名無しさん
垢版 |
2022/10/13(木) 13:50:19.29ID:???
>電場ベクトルEをローレンツ変換するとEx'=Ex となるからだ。

特殊相対論の教科書ならどれでも計算が書いてある。

直角レバーうんぬんは書いてないから、粘着アスペがそれを使って荒らしだしただけだ

ローレンツ変換では物理状態を変えないのを知ってれば、普通の人は気にしてない。
0877ご冗談でしょう?名無しさん
垢版 |
2022/10/13(木) 17:09:12.48ID:???
>>873
>類推から、直角レバーの場合も同様に全体でのみ正しい物理解釈ができる
エレガントにアホパラドックスを一掃する!
現代の特殊相対論は運動量の定義が基になっている(ニュートンのF=maではない)
(重心)物体の運動量の定義 P=m0V/√1-(v/c)^2 から力は F=dP/dt で表すだけ

この定義の運動量の物理意味は、速度Vが変化しなければ時間経過で位置が変わっても
運動量が変化しない。
同様に(重心)物体の運動量の角運動量も時間経過で変化しない。

ローレンツ変換で運動量は定義式から違う値になるが、時間経過で変化しない
物体系全体の運動量と角運動量が保存された状態は変わらない。
アホパラドックスは全て一掃された!
つまり、ローレンツ変換では物理状態を変えないという意味が理解出来ただろ。
俺って天才(copyright)
0879ご冗談でしょう?名無しさん
垢版 |
2022/10/13(木) 17:50:48.03ID:/x8d8vQH
一般より特殊の方が難しいのを知らないな小僧乙です
0881ご冗談でしょう?名無しさん
垢版 |
2022/10/13(木) 19:28:13.06ID:???
>>876
> 直角レバーうんぬんは書いてないから、粘着アスペがそれを使って荒らしだしただけだ
パウリ本や江沢本に書いてる馬鹿

>>877
> アホパラドックスは全て一掃された!
アホ
0883ご冗談でしょう?名無しさん
垢版 |
2022/10/13(木) 20:05:26.75ID:???
>>877
> ローレンツ変換で運動量は定義式から違う値になるが、

これさあ、
座標変換で物理法則が変わってしまうってだけなら、相対性原理に反してるから相間だね、ニュー間だねって話で済む話だけど、
座標変換で定義式が成り立たなくなるとか言い出したら、もう物理学として成立してないだろがよ・・・
0884ご冗談でしょう?名無しさん
垢版 |
2022/10/13(木) 23:12:45.59ID:???
>>833
>座標変換で物理法則が変わってしまうってだけなら、相対性原理に反してるから相間だね
頭悪いやつだな

相対性原理とは、力学の法則(方程式)が座標変換で変化しないという意味だ
ローレンツ変換で運動量と運動エネルギーの値が変っても当たり前だ。
マックスウェル方程式がローレンツ変換で不変でも、電磁場の値は変わる。
0885ご冗談でしょう?名無しさん
垢版 |
2022/10/13(木) 23:19:48.88ID:???
>>883
>パウリ本や江沢本に書いてる

粘着アスペ症のワンパターン参照で逃げる手口
オマエが、5ちゃん住人に分かる様に解決法を簡単に説明してみせればいいだけだ
4−5年前からおんなじパターンで逃げて荒らしてるだけだろが
アスペ症だから他人に説明できないんだろ、出来ないならさっさと消えろ
0886ご冗談でしょう?名無しさん
垢版 |
2022/10/13(木) 23:30:59.84ID:???
>>884
そんなことはわかっとるわ脳足りん。
おまえは力学の法則どころか力や運動量の定義式までも変えてしまってるだろってこった。
0889ご冗談でしょう?名無しさん
垢版 |
2022/10/13(木) 23:44:29.37ID:???
>>877
つまり、ローレンツ変換で運動量保存の法則と角運動量保存の法則が成り立ってれば
パラドックスは起こらない。で解決済み
「運動してる物体の電気力学」(特殊相対論力学)ではローレンツ力の式が不変だから
せっかく苦労して勉強したニュートン力学の力のつり合いや、F=maがそのまま使えな
くなりパラドックス・モドキになるだけだ。 おまけに質量が増える?解釈が蔓延る

もしも特殊相対論を変えないで、ニュートン力学の質量が不変で力のつり合、F=maの式
の計算が使える様に「一種の数学変換」が出来ないだろうか?
それが可能なら、凡人でもニュートン力学の計算で特殊相対論の問題を解くことが出来る!
0890ご冗談でしょう?名無しさん
垢版 |
2022/10/13(木) 23:58:21.24ID:???
>>889
> F=ma
そりゃ相対論では成り立たん式だ。
いい加減、相対論の力の定義くらい覚えてくれ。

> おまけに質量が増える?解釈が蔓延る
いまだに質量が増えるとか言ってんのはおまえだけ。

ずっと誤魔化してんな。
おまえは力学の法則どころか力や運動量の定義式までも変えてしまってるだろ。
登記の話などまったくナンセンスで聞くに値せん。
物理ですらない。
0891ご冗談でしょう?名無しさん
垢版 |
2022/10/14(金) 00:07:12.80ID:???
>>890
>いまだに質量が増えるとか言ってんの
お前か誰かが貼ったPDF論文に、言葉だけでなく式で質量増大と書いてるだろが!
(参考書)太田浩一「電磁気学の基礎II」東京大学出版会, 2012 がネタらしい

つまり、実際には「質量が増える」を禁止用語のような扱いにして誤魔化してるだけだ
0892ご冗談でしょう?名無しさん
垢版 |
2022/10/14(金) 00:16:42.09ID:???
>>891
これおまえだろ?

> 630 名前:ご冗談でしょう?名無しさん[sage] 投稿日:2022/10/05(水) 16:43:44.85 ID:???
> アインシュタイン論文の定義した縦質量と横質量の式を使って何が悪い。

PDFに歴史的な経緯として出てくるのは問題ないが、いまだに使おうとしてるのは登記爺のみ。
まったく逆の立場をでっち上げんな痴呆爺。
0893ご冗談でしょう?名無しさん
垢版 |
2022/10/14(金) 00:29:41.70ID:???
>>889
>それが可能なら、凡人でもニュートン力学の計算で特殊相対論の問題を解くことが出来る!
>>アインシュタイン論文の定義した縦質量と横質量の式を使って何が悪い。
の意味は
俺が試行錯誤の「一種の数学変換」のネタで F'=AF A:行列 -> F'=ma と変換する
方法だ
成功すれば高校物理レベルでも特殊相対論の問題を解くことができるのだよ。

>>892 のバカ読みは教科書コピペしかできないアスペ症とは違うのだよ。
0895ご冗談でしょう?名無しさん
垢版 |
2022/10/14(金) 06:26:22.30ID:???
>>893
結局おまえも縦質量横質量使おうとしたって事じゃねーかw
自分で使うのはよくて、他人が使ってたら文句付けるというのは、どういう頭の構造だ。

> 成功すれば高校物理レベルでも特殊相対論の問題を解くことができるのだよ。

どうぞどうぞ。
いつものように式を違うものに変えてつじつまがあったことにするだけだから簡単でしょうよ。
呆け老人の無意味な作業見せ続けられてもしょうがないからここには書くなよ。
自己満足で終わらせてくれ。
0896ご冗談でしょう?名無しさん
垢版 |
2022/10/14(金) 08:00:40.00ID:???
>>885
>粘着アスペ症のワンパターン参照で逃げる手口
逃げてるのはお前だろうがハゲ

>>889
>つまり、ローレンツ変換で運動量保存の法則と角運動量保存の法則が成り立ってれば
>パラドックスは起こらない。で解決済み
アホ、ローレンツ変換だけじゃ解決できないから「パラドックス」になってんだよ

>>891
>お前か誰かが貼ったPDF論文に、言葉だけでなく式で質量増大と書いてるだろが!
「・・・の割合だけ重く見える。」p.7
の何が問題なんだよ、馬鹿。お前みたいにその後の計算になんか使ってないし

>>893
>成功すれば高校物理レベルでも特殊相対論の問題を解くことができるのだよ。
未だに4元力のローレンツ変換すら出来ない馬鹿だもんな、腹イテェ

>>894
>PDFではしっかり式の計算に使ってる
また嘘ついてら。登記の馬鹿は式読めんのだな
0897ご冗談でしょう?名無しさん
垢版 |
2022/10/14(金) 09:50:35.15ID:???
>>896
オマエの粘着アスペ症は死ぬまで治らんようだな
さっさと
パウリ、江頭が解いたという方法を簡単に説明しろや アスペ症で説明出来ないのか?
>4元力のローレンツ変換
オマエがコピペした式で書いてみろやそれでだけで解けるんだろ アスペ症で説明出来ないのか?

他の5ちゃん住人もアスペ症の説明を期待してるぞ。
0898ご冗談でしょう?名無しさん
垢版 |
2022/10/14(金) 10:19:25.00ID:???
>>889
>「運動してる物体の電気力学」(特殊相対論力学)ではローレンツ力の式が不変
他の5ちゃん住民のために誰でも知ってる式を示そう
静止座標で2つの点電荷qがx方向に距離rで静止した状態とする
   Fx
 q ・←・q
だれでも知ってるように Fx = q^2/4πε0r^2  クーロンの法則そのまんま
問題は、2つの点電荷qが速度vにみえる慣性座標で力Fx'をローレンツ変換で計算せよ。
解くまえに予備の問として (1) Fx' < Fx (2) Fx' = Fx (3) Fx' > Fx
を予想してくれ。
0900ご冗談でしょう?名無しさん
垢版 |
2022/10/14(金) 12:05:09.68ID:???
>>899
俺以外は(2)か(3)が正解だと予想するだろう
力のローレンツ変換の式をコピペ丸暗記(Fx'=Fx)してる人は(2)
(誰かコピペ貼ったPDFにも、添え字が間違ってるが力のローレンツ変換の式が書いてある)
距離rがローレンツ収縮するからと考える人は(3) Fx' > Fx を予想するだろ

ところが、奇妙なことに(1) Fx' < Fx がローレンツ変換の正しい計算結果なのだ!
0901ご冗談でしょう?名無しさん
垢版 |
2022/10/14(金) 13:15:16.37ID:???
>>897-898,900
話逸らすなハゲ。
結局登記の馬鹿は、双子のパラドックスやガレージのパラドックス等日本語の啓蒙書で解説されたモノしか分からんアホだってことだ。自力じゃ英語文献も読めん知障だしなゲラゲラ
0902ご冗談でしょう?名無しさん
垢版 |
2022/10/14(金) 13:31:13.32ID:???
>>901
荒らししかできないなら消えろ!と言ってるだろが
パウリ、江頭どころが力のローレンツ変換の式も自分で説明できないアスペ症だろが
リアルで強制入院させられるだろ
0904ご冗談でしょう?名無しさん
垢版 |
2022/10/14(金) 13:45:16.62ID:dEYDPRAl
相対論 信じるものは 救われる
0906ご冗談でしょう?名無しさん
垢版 |
2022/10/14(金) 13:47:30.97ID:???
>>900
>奇妙なことに(1) Fx' < Fx がローレンツ変換の正しい計算結果なのだ!

俺が「力のローレンツ変換の式」見たこともない云々とかではないのだよ
奇妙なことに、ローレンツ力(またはクーロン力)のモーメント問題に置き換えると
直角レバーが釣り合う
つまり、質量が表面に現れない定義式の計算だとモーメントがしっかり釣り合うが

質量を含んでる「力のローレンツ変換の式」を使うとモーメントが釣り合わなくなる???
直角レバーパラドックスが簡単に解決できなる、解決が簡単でなくなるのは
質量の定義の仕方と関係がある、ということだ。
0907ご冗談でしょう?名無しさん
垢版 |
2022/10/14(金) 15:04:46.03ID:???
>>898

Ex=kq/x^2 (k=1/(4πε_0))とする
x=rの位置にある電荷qに掛かる力はFx=kq^2/r^2

電場Exをローレンツ変換すると
Ex'=kq/{γ(x'-vt')}^2

t'=0のときの電場の強さEx'と力Fx'=qEx'は
Ex'=kq/(γx')^2
Fx'=kq^2/(γx')^2

2つの電荷の距離rはローレンツ収縮してr'=r/γになるから、
Fx'=kq^2/(γr')^2=kq^2/r^2

以上によりFx=Fx'である。正解は(2)
0909ご冗談でしょう?名無しさん
垢版 |
2022/10/14(金) 16:59:05.89ID:???
>>907
マトモに計算する人がいた、最後の行までなら(2)が確かに正しい

>2つの電荷の距離rはローレンツ収縮してr'=r/γになる
静止系で観測してる場合、静止系の長さでr を測定するとすれば
>Ex'=kq/(γx')^2
のように電場が弱くなっていると解釈できるだろう
0911ご冗談でしょう?名無しさん
垢版 |
2022/10/14(金) 20:27:33.06ID:uub8wDEQ
特殊相対性理論と量子テレポーテーションが矛盾しないって本当ですか?またその理由は?
0913ご冗談でしょう?名無しさん
垢版 |
2022/10/14(金) 21:51:22.82ID:???
結局、くっくっくが一番正しいかった。

真空中に孤立した電荷間にはそれらがどのように運動してようが
最初からクーロン力しか働かないと考えればいいだけ。
電磁波も発生しない。

ローレンツ力や電磁誘導や変位電流や電磁波の発生には
物質(の中の電荷)との干渉が必要であって、この物質が物理現象の根源ということ。
これが宇宙の真理であって、観測者による違いが発生しない。

物質の存在を抜きにして
孤立電荷でもすべての電磁現象が発生すると考えている現代物理が異常すぎるだけ。

孤立電荷だけではクーロン力しか作用しない。これが真相。
0914ご冗談でしょう?名無しさん
垢版 |
2022/10/14(金) 22:01:04.19ID:???
孤立電荷が複数あって、観測者によっては静止してたり運動してたり見える。
クーロン力だけだったり、クーロン力とローレンツ力、あるいは電磁波も発生しているように見える。

しかし真相はクーロン力のみであって、観測者の違いによる差など当然ない。

物質がなければローレンツ力や電磁誘導や変位電流や電磁波は発生しない。
物質の物性によってクーロン力以外の電磁現象が発生するので
観測の基準は物質であって観測者ではない。だから観測者の違いうんぬんは最初からない。

たったこれだけの基本法則にいまだに気づかないのが現代物理学。

くっくっくに教えてもらってから今は完全に目が覚めてる。
0918ご冗談でしょう?名無しさん
垢版 |
2022/10/14(金) 22:14:02.61ID:dEYDPRAl
危篤な方と呼んであげてください
0919ご冗談でしょう?名無しさん
垢版 |
2022/10/14(金) 22:14:09.49ID:???
>>914
アンテナという物体がなければ電磁波は出ないってことか。
空間にある孤立電荷だけではどんな加速運動しようが電磁波は出ないってわけね。

片方に物体があればいいわけで、アンテナと孤立電荷の間なら
電磁波が作用反作用するわけね、なるほど。
0920ご冗談でしょう?名無しさん
垢版 |
2022/10/14(金) 22:23:45.75ID:???
>>919
そういうことだな。

加速器では、高速で飛行する電荷と
観測装置という物体との間で電磁現象が発生するので
あたかも電荷が電磁波を放射しているように測定される。

正しくは、観測装置という物体がなければ
高速電荷は何も放射していない。

しかし、磁場をかける加速器全体とも
高速電荷は作用し合っているので、加速器の中の高速電荷はそもそも孤立電荷ではなくて
作用しまくりってこと。

孤立電荷は、観測者が見た瞬間に作用し合う関係になるから
シュレーディンガーの猫みたいなもの。
0921ご冗談でしょう?名無しさん
垢版 |
2022/10/14(金) 22:26:11.01ID:???
>>914
なんでこんな自然な原理に気づかなったんだろうね。
20世紀の物理学者ってアインシュタイン筆頭にマジもんの馬鹿ぞろいだったのか...
0923ご冗談でしょう?名無しさん
垢版 |
2022/10/14(金) 22:31:02.75ID:dEYDPRAl
世界最高の知性の香りがするね
0924ご冗談でしょう?名無しさん
垢版 |
2022/10/14(金) 22:37:09.22ID:???
特殊・一般相対性理論は数学的には間違っていない。

その土台とする
「光速不変」
「ローレンツ変換による特殊相対性原理」
「等価原理」
「一般相対性原理」
が間違っている。

重力で光が曲がるという「等価原理」による思考実験では光源もその重力源にある場合であって、
光源が重力源から遠く離れている場合には説明不可能。

これって、みんな当たり前に気づいてることだよね?
0925ご冗談でしょう?名無しさん
垢版 |
2022/10/14(金) 22:41:40.29ID:???
>>917
少なくとも「光速不変」は誰でも疑ってるだろうね。
加速し続ければ普通に光速超えないとおかしいって、みんな思ってるでしょ。
0928ご冗談でしょう?名無しさん
垢版 |
2022/10/15(土) 01:59:42.75ID:???
>>924
ほんとこれ。
光が曲がる思考実験は2つあって

・自由落下する系は重力を感じないので慣性系と区別できないからその系内では光は直進するはずで、
 それを地上系から見ると光の先端は放物線を描くことになるため、光は重力で曲がることになる。

・宇宙空間で加速する大きな箱の中で光を発射すると光は取り残されて箱の中で曲がるはずだが、
 この加速力は重力と区別できないので重力でも同じように曲がることになる。

アインシュタインがやったとされるこの思考実験の重大な欠点は、どちらも光源が同じ系内にあること。
光源が系外にあって重力や加速力の影響をまったく受けていない場合には適用できない。

結局、特殊な装置を作って強力な加速系で光が曲がる実験をやったとしても、それは
本当に光が曲がっているのか、あるいは光源が原子レベルで加速の影響を受けて
斜めに放射してしまうからなのか区別できないだろうね。
0930ご冗談でしょう?名無しさん
垢版 |
2022/10/15(土) 04:43:49.74ID:???
>>928
> アインシュタインがやったとされるこの思考実験の重大な欠点は、どちらも光源が同じ系内にあること。

アホ、デタラメ書くな
0931ご冗談でしょう?名無しさん
垢版 |
2022/10/15(土) 06:16:13.08ID:???
>>902
荒らしてんのはお前。登記の馬鹿だってバレてんじゃん。

>>903
>(1)を正しいというのは現代の相対論だけ。もちろんそれは間違っている。
おお、漸く相間を自認したか。腹イテェ

>>906,909
馬鹿
0935ご冗談でしょう?名無しさん
垢版 |
2022/10/15(土) 06:52:44.30ID:???
>>931
>>(1)を正しいというのは現代の相対論だけ。もちろんそれは間違っている。
>おお、漸く相間を自認したか。腹イテェ

悪い。その「現代」ってのは、俺が付けた奴の名前だ
0936ご冗談でしょう?名無しさん
垢版 |
2022/10/15(土) 07:34:54.52ID:???
>>928
系外の光源の光もそのまま進んで系内に入ればどちらも同じこと。
重力場の影響を受けずに直進する光(時空の歪みに関係なく直進する光)が存在するならあなたの言うことは正しいかもしれないがそんな光は存在しない。

>925
>加速し続ければ普通に光速超えないとおかしいって、みんな思ってるでしょ
物質が光(電磁結合)で作られてることを考えれば何も不思議ではないでしょう?
光は空間という名の媒質に対して一定にすすむ波。
その一定の速度しか出せない波で作られる物質がその波の速度を超えて成り立つはずが無い。
もし光に関係なく存在する何かあれば可能かもしれないけど。
0937ご冗談でしょう?名無しさん
垢版 |
2022/10/15(土) 08:09:23.15ID:???
>>928
>アインシュタインがやったとされるこの思考実験の重大な欠点は、どちらも光源が同じ系内にあること。

qqq、久しぶりのネタも底が浅過ぎ
つうか馬鹿過ぎて
0939ご冗談でしょう?名無しさん
垢版 |
2022/10/15(土) 12:08:54.77ID:ACMH8/pe
問題は999と666の関連付けをしているような気しかせんから
恐ろしいことにqqqはカルト教系ポスドクである予感がすることw
0940ご冗談でしょう?名無しさん
垢版 |
2022/10/15(土) 12:21:09.37ID:???
>>932
>光源が遠方にある場合には論理破綻するな
正しくは
論理破綻するのは一般相対論の解説本に書いてあるエレベータ箱の話だ!
つまり、箱は足場のように使ってるだけ

アインシュタインの解説によれば等価原理は
慣性系からみた(等)加速度系では全空間に(一定の)重力場が有るのと等価
という意味だ。
頭が良くなりたい人は解説本など読まずに、アインシュタインを苦労してでも読むべきだ
(copyright)
0949ご冗談でしょう?名無しさん
垢版 |
2022/10/15(土) 21:15:42.75ID:???
エレベータや車の加速の場合は運動系から見た空間と静止系から見た空間、その2つの空間が重なって存在してる。
2つと言わず無数の観測者を考えれば無数の空間が同じ位置に重なってる存在することになる。
これは単に観測者ごとに時間と距離の見え方が違うだけで実際に無数の空間が重なってるわけではなく、あたりまえだがあくまで空間は一つしか無い。
ちなみによく勘違いする人がいるが光の媒質とも呼べる電磁場(=空間)も一つしか無い。なので無数の観測者を用意したとしてもすべての観測者は共通の光を違う視点で見てるだけ。

これに対して重力による時空の歪みは実際に空間が変形してるため、異なる加速度による異なる観測者による異なる時空が同じ位置に重なって存在することができない。
なので系外の光源も系内の光源もどちらも同じ結果になる。

慣性質量と重力質量の違いはそれ。
慣性質量と重力質量は等価に扱えるが完全に同じというわけではない。
0951ご冗談でしょう?名無しさん
垢版 |
2022/10/16(日) 00:31:20.15ID:???
重力場のとある1点に物体を置けば必ず決まった加速度になる。
それに対して慣性質量はとある1点に物体を置いてもどのようにでも加速できる。
0953ご冗談でしょう?名無しさん
垢版 |
2022/10/16(日) 05:43:13.60ID:???
>>951
>重力場のとある1点に物体を置けば必ず決まった加速度になる
のは重力質量が慣性質量に比例してるから。
一体何を言いたいのか?
0955ご冗談でしょう?名無しさん
垢版 |
2022/10/16(日) 14:43:20.23ID:???
>>953
アスペ症でないなら、少しは他人を分かってやれよ

>>951
の説では「重力場」がニュートンの万有引力と同じと勘違いしてるだけ
それをオレ様説でスレしただけ、現在でも殆どの一般人は似たような考えだ。

アインシュタインはニュートン力学の加速度系の慣性力が見かけの力ではなく
空間に有る「重力場」による近接作用と考えたのだよ。
つまり、「重力場」は万有引力と慣性力の概念を両方含んでいるから、
加速度系の方向で現れたり消えたりする。(等価原理から)

地球上で自由落下する座標系(近傍)では万有引力と慣性力が釣り合うのではなく
重力場0の無重力状態になる。
その一般相対論が現実の宇宙で正しいことは現代の重力波観測、スカイツリー実験
重力レンズを含め多くの実験で検証されている。
(copyright)
0956ご冗談でしょう?名無しさん
垢版 |
2022/10/16(日) 14:54:42.03ID:???
そういうあなたはもしや、聞いてもいない頓珍漢なレスを繰り返しては粘着アスペと評されてる登記爺では?
0958ご冗談でしょう?名無しさん
垢版 |
2022/10/16(日) 15:18:18.01ID:???
>>957
粘着アスペのお前は貶しか出来ないキチガイだからな

他人が説明してるのが悔しいんだろ、精神病院で治してもらえ
0959ご冗談でしょう?名無しさん
垢版 |
2022/10/16(日) 15:26:39.19ID:???
頭のいい人とは他人の異なる説を対等に比較することが出来る人をいうのだよ

アスペ症、偏執病者にはそれが出来ない、つまり現実社会では使い物にならない。
0962ご冗談でしょう?名無しさん
垢版 |
2022/10/16(日) 16:01:39.49ID:???
粘着アスペが4年以上荒らしてるだけだろが

ローレンツ変換しても座標のレバーの角度が時間変化しないのが誰でも計算できる
レバーが回転する???という意味のレバーパラドックスは解決済みなのだよ。

簡単に解決できないのは定義で変わる力のつり合いの問題だということだ
>力のローレンツ変換
既に知っててもそれを使って直角レバーのパラドックスが簡単に解けないだけの話
それを使っで自分だけの推論計算で解ける奴などこの5chスレにはいない。
URL貼り付け馬鹿がいるだけ (困った時のEMANもできないらしい!)

俺は電気力だけの直角レバーパラドックスに変更して既に解いてるから満足だよ。
0964ご冗談でしょう?名無しさん
垢版 |
2022/10/16(日) 16:14:31.77ID:???
現実形状の平板コンデンサの容量(または電場分布)を解析計算で解け

と似たような問題だが、計算できなくても電磁気学が間違っていると誰も言わない。

>粘着アスペが4年以上荒らしてるだけ だから精神病院に池と言ってるだろ
0965ご冗談でしょう?名無しさん
垢版 |
2022/10/16(日) 16:19:54.18ID:???
>>963
オマエは俺がスレしたのを何でも検索して、粘着荒らしを始める
最後はワンパターンのレバー話を蒸し返えす アスペ症状
>粘着アスペが4年以上荒らしてるだけ だから精神病院に池と言ってるだろ
0968ご冗談でしょう?名無しさん
垢版 |
2022/10/16(日) 16:29:18.14ID:???
>>965
今回の直角レバーの話は>>590以降、どう見ても登記爺が発端だよね?

レバー話を蒸し返してさんざん粘着してデタラメふりまきまくった挙句、
さも他人がやったような言いぐさになるのはなぜか説明してくれるかな?
0969ご冗談でしょう?名無しさん
垢版 |
2022/10/16(日) 16:48:34.20ID:???
登記の馬鹿なりに解けた(>>598)から披露したかったんじゃないかな。
ルイス・トルマン (1909) の間違った解法のまんまだけど。
0970ご冗談でしょう?名無しさん
垢版 |
2022/10/16(日) 16:58:51.17ID:???
それなら「レバー話を蒸し返えしたのは私です、聞いてほしくてさんざん粘着してたんです」と言えという話だな
なんで他人が蒸し返して他人が粘着してたようにいうねん
0971ご冗談でしょう?名無しさん
垢版 |
2022/10/16(日) 16:59:42.33ID:???
5ちゃん馬鹿は >>962
>ローレンツ変換しても座標のレバーの角度が時間変化しないのが誰でも計算できる
>レバーが回転する???という意味のレバーパラドックスは解決済みなのだよ。

誰でも計算結果で解ることさえ認めないらしい(どんな形状・密度分布でも関係ない)
0974ご冗談でしょう?名無しさん
垢版 |
2022/10/16(日) 19:33:52.58ID:???
粘着アスペと相間サイトの相間はコピペの解釈が逆だけで同類の思考停止

マトモな人なら力のモーメント、角運動量などが特殊相対論で本当に計算困難なら
凡人には使い物にならない。
ニュートン力学の力のモーメントの定義は r x F だがローレンツ変換の力が前提
で釣り合わなくのはパラドックスの通りだ。

特殊相対論の力のモーメントの定義が変るからだとすれば 2次元で関数 fx , fy
fx(ry x Fx) , fy(rx x Fy)  の形式だと仮定しよう
fx と fy との比が相対論の因子γ^2 であることは今までの計算から誰でも解る。

これが直角以外の任意の方向成分でも一致すれば計算的には辻褄が合うといえる。
0975ご冗談でしょう?名無しさん
垢版 |
2022/10/16(日) 19:39:10.55ID:???
「マトモな人なら力のモーメント、角運動量などが特殊相対論で本当に計算困難なら
凡人には使い物にならない。」

すまんが、この意味不明な文章は誰に向かって何を説明したいのかさっぱりわからん。
「アスペ」でもわかるように文章を推敲してから再度書き込んでみてくれ。
0976ご冗談でしょう?名無しさん
垢版 |
2022/10/16(日) 19:46:54.11ID:???
>>975
特殊相対論の定番思考実験で列車と線路では車輪が回転している
列車の車輪の回転運動が計算困難だと凡人(学生)には大問題だろ
0977ご冗談でしょう?名無しさん
垢版 |
2022/10/16(日) 19:47:06.25ID:???
論理学だと
AならB
とか
AならBではない
といった論理を展開するが
登記論理学では
「AならBならCではない
などという登記本人にしか理解不能な論理を書き散らして
その意味不明な論理を指摘する人間を「アスペ」と断定するから痛々しい。
アスペはお前だろがw
0980ご冗談でしょう?名無しさん
垢版 |
2022/10/16(日) 20:10:27.90ID:???
>>955
アインシュタインの考えでは重力なんてものは存在しない。
遠方から見て曲がって進む物体や光は、時空の歪みの中をまっすぐに進んでるだけという風に解釈するのが正解。

>つまり、「重力場」は万有引力と慣性力の概念を両方含んでいる
それは【座標系】の説明。相対性理論の説明じゃない。
物体や光は【力】で曲がるわでじゃない。物体や光が曲がる時、何の【力】も発生しない。
ただ曲がった空間の中をまっすぐそのまま直進してるだけ。
その曲がって進む様子を静止系では重力加速度という風に解釈する。
その重力加速度の集まりを重力場という風に解釈してるだけ。

>の説では「重力場」がニュートンの万有引力と同じと勘違いしてるだけ
私は勘違いしてませんよ。むしろあなたの説明の方が勘違いしてるように見える。
0981ご冗談でしょう?名無しさん
垢版 |
2022/10/16(日) 21:05:32.08ID:???
1、重力質量による時空の歪み
2、慣性質量による時空の歪み
どちらも同じだと思ってる人が多いようだけど、
アインシュタインは2つを等価に扱えるとは言ってるが、完全に同じだとは言ってない。

1、は実際に時空が歪んでる。曲がった時空の中をまっすぐ進む。
2、は実際には時空は歪んでない。観測者ごとに見え方(距離と時間)が異なりそれを時空と解釈してるだけ。

なぜならあたりまえだが、現実には1つの空間しか存在しないから。
ただし計算上は無数の観測者ごとに無数の時空を考えることはできるし、無数の空間(座標系)が重なって存在するように考えることは可能。

加速度についての解釈も注意が必要。
1、は静止系から見て曲がって進む様子を重力加速度という風に解釈してるだけ。実際には曲がった時空の中をまっすぐ進んでるだけ。自分では加速度を調整できない。
   ↑↓まるで違う。
2、は加速度を自分で自由に調整できるし、その結果として時空が歪んでるように見えてるだけ。

計算上は同じように扱えるが、そもそも異なる現象なのだと言うことを忘れてはいけない。
0983ご冗談でしょう?名無しさん
垢版 |
2022/10/16(日) 21:10:49.21ID:11J4YVin
二重計量モデルってやつだね。重力質量と慣性質量とで時空の曲がり方が違うんだよ。多分最新理論
0984ご冗談でしょう?名無しさん
垢版 |
2022/10/16(日) 21:17:53.72ID:???
>>980
>その重力加速度の集まりを重力場という風に解釈してるだけ。
が意味不明だが
>>949 のような全部意味不明でなく、始めから時空が歪んでるぽく書けばいいだけ

電磁気学でいえばベクトルポテンシャルで説明するか、電磁場で説明するかの違い

>>981 また意味不明にもどしたいようだな
0985ご冗談でしょう?名無しさん
垢版 |
2022/10/16(日) 21:19:22.63ID:???
>>982
重力質量で時空が歪んでるのか、
慣性質量で時空が歪んでるのか、
加速してる本人から見ると見分けがつかない。
自分の周りに比較対象がある場合は見分けがつくかもしれませんが、
重力加速の場合は周囲のすべてが空間丸ごと同じ重力加速で一緒に動くから比較対象が無い。
0986ご冗談でしょう?名無しさん
垢版 |
2022/10/16(日) 21:23:20.78ID:???
>>983
アインシュタインの一般相対論は完結してる。
後世学者は後から数学でいじくるしかできない、失敗すると物理解釈が
意味不明になる
0987ご冗談でしょう?名無しさん
垢版 |
2022/10/16(日) 21:40:26.43ID:???
他の理論と同様に一般相対性理論は完結なんか全くしてない。
斥力項の取り扱いとか、実験的フィードバックによる修正などは日常茶飯事。
すべての科学の理論は常に修正を受け付ける準備がある。
0988ご冗談でしょう?名無しさん
垢版 |
2022/10/16(日) 21:43:52.32ID:???
ふと思ったんだけど。
重力質量による時空の歪みも実は空間は歪んでない。
光は曲がった空間の中を進むのではなく実際に曲げられてるだけと解釈すれば。
慣性質量と重力質量は完全に同じ現象として考えることが可能になるんじゃないだろうか。
見分けがつかないだけではなく、本当に同じ現象なのだと考えられる。

将来、重力加速に関係なく動くことが可能な物体か技術か何かが発見されれば、
空間が実際に曲がってるなんて考え方は将来笑い話になる可能性すらある。
そうなったとしても相対性理論の計算方法だけは変わらず残るだろうけど、時空という概念の解釈が変わる。
0990ご冗談でしょう?名無しさん
垢版 |
2022/10/16(日) 21:48:56.62ID:???
>>988
光子の質量が0であることは現在の高精度測定で実証されています。
その上でなおしつこく
「実際に曲げられてるだけと解釈」
する理由は今のところ何一つありません。
0991ご冗談でしょう?名無しさん
垢版 |
2022/10/16(日) 21:56:21.44ID:11J4YVin
>>990
>実証されています。
されてません。永遠に無理です。
0992ご冗談でしょう?名無しさん
垢版 |
2022/10/16(日) 22:01:42.35ID:???
>>988
まあコペルニクス以前は重力とか知らんから勝手に曲がってると思ってたしな
まったく知識のない人間の発想ってだいたい同じなんだな
0994ご冗談でしょう?名無しさん
垢版 |
2022/10/16(日) 22:13:25.51ID:11J4YVin
どこで0が実証されてるの?
0995ご冗談でしょう?名無しさん
垢版 |
2022/10/16(日) 22:16:23.67ID:???
>>994
0でない実証はどこでされてるの?
いまのところ最も正確な実証理論である量子電磁力学理論では0とされていて、
それに反する実証結果は再現性が無いものばかりだけど、どれを実証と言っているの?
0996ご冗談でしょう?名無しさん
垢版 |
2022/10/16(日) 22:19:54.32ID:11J4YVin
じぶんの言には責任とってね。あほらし
0998ご冗談でしょう?名無しさん
垢版 |
2022/10/16(日) 23:01:49.09ID:???
>>988
そもそも時空の歪みは、直接観測不可能だのだよ!

アインシュタインの一般相対性理論で「時空の歪」(または重力ポテンシャル)を定義している
同様に「光が歪んだ時空間の中を直進する」というのも一般相対性理論の中だけの話だ。
(その光直進ではそもそも重力自体が無いとおなじ)
”人類は時空の歪に閉じ込められている” 凡人には理解不能

理論を理解する知能レベルがない凡人は物理観測可能な重力(場)の作用で理解するしかない
相対論によれば重力はエネルギーに作用すると物理解釈するのが普通のやりかただ。
(ワインバーグの一般人向け本にもそう書いてある)
つまり、質量物体や光はエネルギーを持つから重力(場)で曲がるという説明になる。
”人類は重力で閉じ込められている” 凡人でも理解可能

万有引力理論は質量にしか作用しないから光は曲がらない、現実の実験で比較検証できる。
めでたし、めでたし
10011001
垢版 |
Over 1000Thread
このスレッドは1000を超えました。
新しいスレッドを立ててください。
life time: 119日 20時間 48分 32秒
10021002
垢版 |
Over 1000Thread
5ちゃんねるの運営はプレミアム会員の皆さまに支えられています。
運営にご協力お願いいたします。


───────────────────
《プレミアム会員の主な特典》
★ 5ちゃんねる専用ブラウザからの広告除去
★ 5ちゃんねるの過去ログを取得
★ 書き込み規制の緩和
───────────────────

会員登録には個人情報は一切必要ありません。
月300円から匿名でご購入いただけます。

▼ プレミアム会員登録はこちら ▼
https://premium.5ch.net/

▼ 浪人ログインはこちら ▼
https://login.5ch.net/login.php
レス数が1000を超えています。これ以上書き込みはできません。

ニューススポーツなんでも実況